Contenu | Rechercher | Menus

Annonce

Si vous avez des soucis pour rester connecté, déconnectez-vous puis reconnectez-vous depuis ce lien en cochant la case
Me connecter automatiquement lors de mes prochaines visites.

À propos de l'équipe du forum.

#1 Le 06/09/2015, à 22:09

jedineofr

[RESOLU]Dual-Boot Windows 8.1 et Ubuntu 14.04 sur Acer Aspire V5-122P

Bonjour à tous

Après avoir installé Ubuntu en UEFI (comme l'était Windows) depuis la version international, l'ordinateur boote directement sur la partition Windows.

J'ai testé boot repair mais cela n'a pas résolu mon problème.
Je vous poste donc la fin de mon rapport (http://paste.ubuntu.com/12300288/)

Error: no grub*.efi generated. Please report this message to boot.repair@gmail.com

Add /mnt/boot-sav/sda6/boot/efi efi entries in /mnt/boot-sav/sda6/etc/grub.d/25_custom
Adding custom /mnt/boot-sav/sda6/boot/efi/EFI/Microsoft/Boot/bootmgfw.efi
Adding custom /mnt/boot-sav/sda6/boot/efi/EFI/Boot/bootx64.efi
sda2/bootx64.efi already added
sda2/bootmgfw.efi already added
Adding custom /mnt/boot-sav/sda6/boot/efi/EFI/OEM/Boot/bootmgfw.efi

---- Grub-install verbose
/usr/sbin/grub-install: 2: /usr/sbin/grub-install: Syntax error: "(" unexpected
--------
/usr/sbin/grub-install --efi-directory=/boot/efi --target=x86_64-efi --uefi-secure-boot : exit code of grub-install :2
---- End of grub-install verbose


chroot /mnt/boot-sav/sda6 efibootmgr -v
BootCurrent: 0001
Timeout: 2 seconds
BootOrder: 2001,0000,2002,2003
Boot0000* Windows Boot Manager	HD(2,c8800,96000,7e77172a-71dd-41ec-a543-f688fe140041)File(EFIMicrosoftBootbootmgfw.efi)WINDOWS.........x...B.C.D.O.B.J.E.C.T.=.{.9.d.e.a.8.6.2.c.-.5.c.d.d.-.4.e.7.0.-.a.c.c.1.-.f.3.2.b.3.4.4.d.4.7.9.5.}....................
Boot0001* USB HDD: General USB Flash Disk	ACPI(a0341d0,0)PCI(12,2)USB(0,0)HD(1,3e,3bb5aa,00000000)RC
Boot0002* WDC WD5000LPVX-22V0TT0          	BIOS(2,500,00)................-.d.......d.A.d........................................
Boot0003* General USB Flash Disk  	BIOS(6,500,00).......................................................................
Boot2001* EFI USB Device	RC
Boot2002* EFI DVD/CDROM	RC
Boot2003* EFI Network	RC

chroot /mnt/boot-sav/sda6 update-grub
Generating grub configuration file ...
Found linux image: /boot/vmlinuz-3.19.0-26-generic
Found initrd image: /boot/initrd.img-3.19.0-26-generic
Found linux image: /boot/vmlinuz-3.19.0-25-generic
Found initrd image: /boot/initrd.img-3.19.0-25-generic
Found Windows Boot Manager on /dev/sda2@/EFI/Microsoft/Boot/bootmgfw.efi
Adding boot menu entry for EFI firmware configuration
Unhide GRUB boot menu in sda6/boot/grub/grub.cfg

An error occurred during the repair.

Locked-ESP detected. You may want to retry after creating a /boot/efi partition (FAT32, 100MB~250MB, start of the disk, boot flag). This can be performed via tools such as gParted. Then select this partition via the [Separate /boot/efi partition:] option of [Boot Repair].

Download as text

Merci d'avande de votre aide

Dernière modification par jedineofr (Le 07/09/2015, à 13:35)


Ubuntu 8.04 LTS -> Ubuntu 8.10 -> Ubuntu 9.04 -> Ubuntu 9.10 -> Ubuntu 10.04 LTS -> Lubuntu 14.04 > Ubuntu 14.04 > Ubuntu 16.04 LTS > Ubuntu 18.04 LTS
Asus ROG G551JW-DM379T (i7 4750HQ, GTX960M, Dd 1 To + SSD 24 Go, Windows 10 + Ubuntu)

Hors ligne

#2 Le 06/09/2015, à 23:17

Bougron

Re : [RESOLU]Dual-Boot Windows 8.1 et Ubuntu 14.04 sur Acer Aspire V5-122P

Bonjour.
Je ne sais pas comment tu as fait ton installation efi mais elle s'est mal déroulée.
en effet , le logiciel de boot de ubuntu ne s'est pas installé dans la partition de boot.

sda2: __________________________________________________________________________
    File system:       vfat
    Boot sector type:  Windows 8/2012: FAT32
    Boot sector info:  No errors found in the Boot Parameter Block.
    Operating System:  
    Boot files:        /EFI/Boot/bootx64.efi /EFI/Microsoft/Boot/bootmgfw.efi 
                       /EFI/Microsoft/Boot/bootmgr.efi 
                       /EFI/Microsoft/Boot/memtest.efi 
                       /EFI/OEM/Boot/bootmgfw.efi /EFI/OEM/Boot/bootmgr.efi 
                       /EFI/OEM/Boot/memtest.efi

en conséquence, il ne s'est pas installé dans le bios EFI

/boot/efi detected in the fstab of sda6: UUID=86C4-3BF4	 (sda2)
=================== efibootmgr -v
BootCurrent: 0001
Timeout: 2 seconds
BootOrder: 2001,0000,2002,2003
Boot0000* Windows Boot Manager	HD(2,c8800,96000,7e77172a-71dd-41ec-a543-f688fe140041)File(EFIMicrosoftBootbootmgfw.efi)WINDOWS.........x...B.C.D.O.B.J.E.C.T.=.{.9.d.e.a.8.6.2.c.-.5.c.d.d.-.4.e.7.0.-.a.c.c.1.-.f.3.2.b.3.4.4.d.4.7.9.5.}....................
Boot0001* USB HDD: General USB Flash Disk	ACPI(a0341d0,0)PCI(12,2)USB(0,0)HD(1,3e,3bb5aa,00000000)RC
Boot0002* WDC WD5000LPVX-22V0TT0          	BIOS(2,500,00)................-.d.......d.A.d........................................
Boot0003* General USB Flash Disk  	BIOS(6,500,00).......................................................................
Boot2001* EFI USB Device	RC
Boot2002* EFI DVD/CDROM	RC
Boot2003* EFI Network	RC
=================== UEFI/Legacy mode:
BIOS is EFI-compatible, and is setup in EFI-mode for this live-session.
SecureBoot disabled. (maybe sec-boot, Please report this message to boot.repair@gmail.com)

Mais tu es excusé car la réparation de boot-repair n'arrive pas non plus à le faire à cause de ce message:
Locked-ESP detected You may want to retry after creating a /boot/efi partition (FAT32, 100MB~250MB, start of the disk, boot flag). This can be performed via tools such as gParted. Then select this partition via the [Separate /boot/efi partition:] option of [Boot Repair].
J'ai déjà rencontré ce cas une fois.  Il y a un constructeur qui verrouille la partition de boot.
Une solution est de créer une partition supplémentaire. C'est ce qu'on avait fait. Mais peux-tu lancer  gparted en liveUSB et poster ce qu'il affiche pour la description du disque SDA   car il est peut-être plus simple de casser cette protection.

Car il suffirait probablement de mettre le flag BOOT et d'enlever le flag HIDDEN à moins qu'il y ait encore d'autres flags!
Je sais: c'est stupide: En EFI il n'y a pas besoin d'un flag boot. Mais que veux-tu avec ubuntu tout est possible !!!!

AJOUT.
    Sinon prends d'entrée ce  que boot-repair propose.
                     Lances une "livesession" avec la clé USB d'installation"
                     Choisis  "try ubuntu before installing"
                    Lances Gparted
                    Fabriques une partition FAT32 d'environ 128 Mo
                    Mets le flag-boot sur cette partition.                 
                       Attention c'est le drapeau BOOT et surtout pas  "'legacy_boot" ni "bios-grub"
                Retentes un boot-repair,
                1) Choisir "options avancées" au lieu de "réparation recommandée"
                2) choisir "emplacement de grub" au lieu de "options principales"
                3) choisir la nouvelle partition que tu viens de créer
                4) et Lancer la réparation 
                       soit poster le boot-info poroduit afin de voir si cela a été bien réeparé
                       soit  faire la commande efibootmgr
                             Installer, si nécessaire, le logiciel avec la ligne de commande qui a été indiquée.
                             Puis faire la commande 

sudo efibootmgr -v

afin de vérifier que tout est OK.
En première position, tu devrais avoir la ligne de ubuntu avecle numéro de laartition nouvelle tel qu'on le voit ici.

usb14-04-3@usb14043:~$ sudo efibootmgr -v
BootCurrent: 0000
Timeout: 0 seconds
BootOrder: 0003,2001,0002,0008,000A,0001,2003,2002
Boot0000* EFI USB Device (WD      Elements 1023)	ACPI(a0341d0,0)PCI(1d,0)USB(0,0)USB(0,0)HD(1,800,ff800,939de71b-766d-4050-9ee4-b868d3e3e668)RC
Boot0001* Windows Boot Manager	HD(2,96800,32000,de40e452-2c7c-4617-9b15-15b7ddf168d0)File(\EFI\Microsoft\Boot\bootmgfw.efi)RC
Boot0002* Windows Boot Manager	HD(1,800,40000,4896ebaf-3832-4ee3-b6b3-2d9c77917c1b)File(\EFI\Microsoft\Boot\bootmgfw.efi)RC
Boot0003* ubuntu	HD(b,14108800,3f40000,fcfadc16-94ac-43d9-8998-0de8fc2175bd)File(\EFI\ubuntu\shimx64.efi)
Boot0004* EFI Network 0 for IPv6 (08-9E-01-B7-1C-5D) 	ACPI(a0341d0,0)PCI(1c,3)PCI(0,0)MAC(089e01b71c5d,0)030d3c000000000000000000000000000000000000000000000000000000000000000000000000000000004000000000000000000000000000000000RC
Boot0005* EFI Network 0 for IPv4 (08-9E-01-B7-1C-5D) 	ACPI(a0341d0,0)PCI(1c,3)PCI(0,0)MAC(089e01b71c5d,0)IPv4(0.0.0.0:0<->0.0.0.0:0,0, 0RC
Boot0007  rEFInd Boot Manager	HD(2,96800,32000,de40e452-2c7c-4617-9b15-15b7ddf168d0)File(\EFI\refind\refind_x64.efi)
Boot0008* EFI HDD Device (TOSHIBA MQ01ABD100)	ACPI(a0341d0,0)PCI(1f,2)03120a00000000000000HD(15,735d1800,1135000,75fb9d05-9b88-494b-82f8-abd0d5fbf8f8)RC
Boot000A* Windows Boot Manager	HD(2,96800,32000,de40e452-2c7c-4617-9b15-15b7ddf168d0)File(\EFI\Microsoft\Boot\bootmgfworigine.efi)WINDOWS.........x...B.C.D.O.B.J.E.C.T.=.{.9.d.e.a.8.6.2.c.-.5.c.d.d.-.4.e.7.0.-.a.c.c.1.-.f.3.2.b.3.4.4.d.4.7.9.5.}....................
Boot2001* EFI USB Device	RC
Boot2002* EFI DVD/CDROM	RC
Boot2003* EFI Network	RC
usb14-04-3@usb14043:~$ 

Pour Malbo
Bonjour.
J'ai lu ta réponse, il me semble que boot-repair  va aussi faire la commande que tu indiques....
           et qu'il saura utiliser la nouvelle partition fat32  avec l'option "separate /boot/efi" que je découvre.

   
Conclusion:  Tu auras une partition permettant de booter windows et une autre permettant de booter ubuntu.
      Sous  ubuntu, lorsque tu accéderas à la partition /boot/efi/EFI, tu ne verras pas les fichiers de boot de windows. Cela t'évitera de changer leur nom pour qu'ils ne commencent pas par BKP...
Il est possible que cette protection ait été installée lorsque les constructeurs ont constaté que leur nom de fichiers de boot étaient modifié sans leur accord.

Dernière modification par Bougron (Le 07/09/2015, à 10:44)

Hors ligne

#3 Le 07/09/2015, à 06:38

malbo

Re : [RESOLU]Dual-Boot Windows 8.1 et Ubuntu 14.04 sur Acer Aspire V5-122P

Ton Boot-Info complet est là :

 Boot Info Script e7fc706 + Boot-Repair extra info      [Boot-Info 9Feb2015]


============================= Boot Info Summary: ===============================

 => No boot loader is installed in the MBR of /dev/sda.
 => Syslinux MBR (4.04 and higher) is installed in the MBR of /dev/sdb.

sda1: __________________________________________________________________________

    File system:       ntfs
    Boot sector type:  Windows 8/2012: NTFS
    Boot sector info:  No errors found in the Boot Parameter Block.
    Operating System:  
    Boot files:        

sda2: __________________________________________________________________________

    File system:       vfat
    Boot sector type:  Windows 8/2012: FAT32
    Boot sector info:  No errors found in the Boot Parameter Block.
    Operating System:  
    Boot files:        /EFI/Boot/bootx64.efi /EFI/Microsoft/Boot/bootmgfw.efi 
                       /EFI/Microsoft/Boot/bootmgr.efi 
                       /EFI/Microsoft/Boot/memtest.efi 
                       /EFI/OEM/Boot/bootmgfw.efi /EFI/OEM/Boot/bootmgr.efi 
                       /EFI/OEM/Boot/memtest.efi

sda3: __________________________________________________________________________

    File system:       
    Boot sector type:  -
    Boot sector info: 
    Mounting failed:   mount: unknown filesystem type ''

sda4: __________________________________________________________________________

    File system:       ntfs
    Boot sector type:  Windows 8/2012: NTFS
    Boot sector info:  No errors found in the Boot Parameter Block.
    Operating System:  Windows 8
    Boot files:        /Windows/System32/winload.exe

sda5: __________________________________________________________________________

    File system:       ntfs
    Boot sector type:  Windows 8/2012: NTFS
    Boot sector info:  No errors found in the Boot Parameter Block.
    Operating System:  
    Boot files:        

sda6: __________________________________________________________________________

    File system:       ext4
    Boot sector type:  -
    Boot sector info: 
    Operating System:  Ubuntu 14.04.3 LTS 
    Boot files:        /boot/grub/grub.cfg /etc/fstab

sda7: __________________________________________________________________________

    File system:       swap
    Boot sector type:  -
    Boot sector info: 

sdb1: __________________________________________________________________________

    File system:       vfat
    Boot sector type:  SYSLINUX 6.03 20141020..................................................2....0............A20 gate n
    Boot sector info:  Syslinux looks at sector 2122308 of /dev/sdb1 for its 
                       second stage. SYSLINUX is installed in the  directory. 
                       No errors found in the Boot Parameter Block.
    Operating System:  
    Boot files:        /boot/grub/grub.cfg /syslinux.cfg /casper/vmlinuz.efi 
                       /EFI/BOOT/grubx64.efi /ldlinux.sys

============================ Drive/Partition Info: =============================

Drive: sda _____________________________________________________________________

Disk /dev/sda: 500.1 GB, 500107862016 bytes
255 heads, 63 sectors/track, 60801 cylinders, total 976773168 sectors
Units = sectors of 1 * 512 = 512 bytes
Sector size (logical/physical): 512 bytes / 4096 bytes

Partition  Boot  Start Sector    End Sector  # of Sectors  Id System

/dev/sda1                   1   976,773,167   976,773,167  ee GPT


GUID Partition Table detected.

Partition    Start Sector    End Sector  # of Sectors System
/dev/sda1           2,048       821,247       819,200 Windows Recovery Environment (Windows)
/dev/sda2         821,248     1,435,647       614,400 EFI System partition
/dev/sda3       1,435,648     1,697,791       262,144 Microsoft Reserved Partition (Windows)
/dev/sda4       1,697,792   392,410,661   390,712,870 Data partition (Windows/Linux)
/dev/sda5     947,156,992   976,773,119    29,616,128 Windows Recovery Environment (Windows)
/dev/sda6     392,411,136   935,745,535   543,334,400 Data partition (Linux)
/dev/sda7     935,745,536   947,156,991    11,411,456 Swap partition (Linux)

Drive: sdb _____________________________________________________________________

Disk /dev/sdb: 2004 MB, 2004877312 bytes
62 heads, 62 sectors/track, 1018 cylinders, total 3915776 sectors
Units = sectors of 1 * 512 = 512 bytes
Sector size (logical/physical): 512 bytes / 512 bytes

Partition  Boot  Start Sector    End Sector  # of Sectors  Id System

/dev/sdb1    *             62     3,913,191     3,913,130   c W95 FAT32 (LBA)


"blkid" output: ________________________________________________________________

Device           UUID                                   TYPE       LABEL

/dev/loop0                                              squashfs   
/dev/sda1        FA92C1CB92C18D17                       ntfs       Recovery
/dev/sda2        86C4-3BF4                              vfat       ESP
/dev/sda4        C69C86AD9C869817                       ntfs       Acer
/dev/sda5        6A80C79F80C76FDD                       ntfs       Push Button Reset
/dev/sda6        d2a5ed22-3bda-4625-9f3b-069625b0ebba   ext4       
/dev/sda7        23940c6e-bf1d-426d-98f7-775a96248814   swap       
/dev/sdb1        8606-DE46                              vfat       

========================= "ls -l /dev/disk/by-id" output: ======================

total 0
lrwxrwxrwx 1 root root  9 Sep  6 20:01 ata-WDC_WD5000LPVX-22V0TT0_WD-WX71E23VTY10 -> ../../sda
lrwxrwxrwx 1 root root 10 Sep  6 20:03 ata-WDC_WD5000LPVX-22V0TT0_WD-WX71E23VTY10-part1 -> ../../sda1
lrwxrwxrwx 1 root root 10 Sep  6 20:01 ata-WDC_WD5000LPVX-22V0TT0_WD-WX71E23VTY10-part2 -> ../../sda2
lrwxrwxrwx 1 root root 10 Sep  6  2015 ata-WDC_WD5000LPVX-22V0TT0_WD-WX71E23VTY10-part3 -> ../../sda3
lrwxrwxrwx 1 root root 10 Sep  6 20:03 ata-WDC_WD5000LPVX-22V0TT0_WD-WX71E23VTY10-part4 -> ../../sda4
lrwxrwxrwx 1 root root 10 Sep  6 20:03 ata-WDC_WD5000LPVX-22V0TT0_WD-WX71E23VTY10-part5 -> ../../sda5
lrwxrwxrwx 1 root root 10 Sep  6  2015 ata-WDC_WD5000LPVX-22V0TT0_WD-WX71E23VTY10-part6 -> ../../sda6
lrwxrwxrwx 1 root root 10 Sep  6  2015 ata-WDC_WD5000LPVX-22V0TT0_WD-WX71E23VTY10-part7 -> ../../sda7
lrwxrwxrwx 1 root root  9 Sep  6 20:01 usb-General_USB_Flash_Disk_0000000000001379-0:0 -> ../../sdb
lrwxrwxrwx 1 root root 10 Sep  6  2015 usb-General_USB_Flash_Disk_0000000000001379-0:0-part1 -> ../../sdb1
lrwxrwxrwx 1 root root  9 Sep  6 20:01 wwn-0x50014ee25db8ef41 -> ../../sda
lrwxrwxrwx 1 root root 10 Sep  6 20:03 wwn-0x50014ee25db8ef41-part1 -> ../../sda1
lrwxrwxrwx 1 root root 10 Sep  6 20:01 wwn-0x50014ee25db8ef41-part2 -> ../../sda2
lrwxrwxrwx 1 root root 10 Sep  6  2015 wwn-0x50014ee25db8ef41-part3 -> ../../sda3
lrwxrwxrwx 1 root root 10 Sep  6 20:03 wwn-0x50014ee25db8ef41-part4 -> ../../sda4
lrwxrwxrwx 1 root root 10 Sep  6 20:03 wwn-0x50014ee25db8ef41-part5 -> ../../sda5
lrwxrwxrwx 1 root root 10 Sep  6  2015 wwn-0x50014ee25db8ef41-part6 -> ../../sda6
lrwxrwxrwx 1 root root 10 Sep  6  2015 wwn-0x50014ee25db8ef41-part7 -> ../../sda7

================================ Mount points: =================================

Device           Mount_Point              Type       Options

/dev/loop0       /rofs                    squashfs   (ro,noatime)
/dev/sdb1        /cdrom                   vfat       (ro,noatime,fmask=0022,dmask=0022,codepage=437,iocharset=iso8859-1,shortname=mixed,errors=remount-ro)


=========================== sda6/boot/grub/grub.cfg: ===========================

--------------------------------------------------------------------------------
#
# DO NOT EDIT THIS FILE
#
# It is automatically generated by grub-mkconfig using templates
# from /etc/grub.d and settings from /etc/default/grub
#

### BEGIN /etc/grub.d/00_header ###
if [ -s $prefix/grubenv ]; then
  set have_grubenv=true
  load_env
fi
if [ "${next_entry}" ] ; then
   set default="${next_entry}"
   set next_entry=
   save_env next_entry
   set boot_once=true
else
   set default="0"
fi

if [ x"${feature_menuentry_id}" = xy ]; then
  menuentry_id_option="--id"
else
  menuentry_id_option=""
fi

export menuentry_id_option

if [ "${prev_saved_entry}" ]; then
  set saved_entry="${prev_saved_entry}"
  save_env saved_entry
  set prev_saved_entry=
  save_env prev_saved_entry
  set boot_once=true
fi

function savedefault {
  if [ -z "${boot_once}" ]; then
    saved_entry="${chosen}"
    save_env saved_entry
  fi
}
function recordfail {
  set recordfail=1
  if [ -n "${have_grubenv}" ]; then if [ -z "${boot_once}" ]; then save_env recordfail; fi; fi
}
function load_video {
  if [ x$feature_all_video_module = xy ]; then
    insmod all_video
  else
    insmod efi_gop
    insmod efi_uga
    insmod ieee1275_fb
    insmod vbe
    insmod vga
    insmod video_bochs
    insmod video_cirrus
  fi
}

if [ x$feature_default_font_path = xy ] ; then
   font=unicode
else
insmod part_gpt
insmod ext2
set root='hd0,gpt6'
if [ x$feature_platform_search_hint = xy ]; then
  search --no-floppy --fs-uuid --set=root --hint-bios=hd0,gpt6 --hint-efi=hd0,gpt6 --hint-baremetal=ahci0,gpt6  d2a5ed22-3bda-4625-9f3b-069625b0ebba
else
  search --no-floppy --fs-uuid --set=root d2a5ed22-3bda-4625-9f3b-069625b0ebba
fi
    font="/usr/share/grub/unicode.pf2"
fi

if loadfont $font ; then
  set gfxmode=auto
  load_video
  insmod gfxterm
  set locale_dir=$prefix/locale
  set lang=en_US
  insmod gettext
fi
terminal_output gfxterm
if [ "${recordfail}" = 1 ] ; then
  set timeout=10
else
  if [ x$feature_timeout_style = xy ] ; then
    set timeout_style=menu
    set timeout=10
  # Fallback normal timeout code in case the timeout_style feature is
  # unavailable.
  else
    set timeout=10
  fi
fi
### END /etc/grub.d/00_header ###

### BEGIN /etc/grub.d/05_debian_theme ###
set menu_color_normal=white/black
set menu_color_highlight=black/light-gray
if background_color 44,0,30,0; then
  clear
fi
### END /etc/grub.d/05_debian_theme ###

### BEGIN /etc/grub.d/10_linux ###
function gfxmode {
	set gfxpayload="${1}"
	if [ "${1}" = "keep" ]; then
		set vt_handoff=vt.handoff=7
	else
		set vt_handoff=
	fi
}
if [ "${recordfail}" != 1 ]; then
  if [ -e ${prefix}/gfxblacklist.txt ]; then
    if hwmatch ${prefix}/gfxblacklist.txt 3; then
      if [ ${match} = 0 ]; then
        set linux_gfx_mode=keep
      else
        set linux_gfx_mode=text
      fi
    else
      set linux_gfx_mode=text
    fi
  else
    set linux_gfx_mode=keep
  fi
else
  set linux_gfx_mode=text
fi
export linux_gfx_mode
menuentry 'Ubuntu' --class ubuntu --class gnu-linux --class gnu --class os $menuentry_id_option 'gnulinux-simple-d2a5ed22-3bda-4625-9f3b-069625b0ebba' {
	recordfail
	load_video
	gfxmode $linux_gfx_mode
	insmod gzio
	insmod part_gpt
	insmod ext2
	set root='hd0,gpt6'
	if [ x$feature_platform_search_hint = xy ]; then
	  search --no-floppy --fs-uuid --set=root --hint-bios=hd0,gpt6 --hint-efi=hd0,gpt6 --hint-baremetal=ahci0,gpt6  d2a5ed22-3bda-4625-9f3b-069625b0ebba
	else
	  search --no-floppy --fs-uuid --set=root d2a5ed22-3bda-4625-9f3b-069625b0ebba
	fi
	linux	/boot/vmlinuz-3.19.0-26-generic.efi.signed root=UUID=d2a5ed22-3bda-4625-9f3b-069625b0ebba ro  quiet splash $vt_handoff
	initrd	/boot/initrd.img-3.19.0-26-generic
}
submenu 'Advanced options for Ubuntu' $menuentry_id_option 'gnulinux-advanced-d2a5ed22-3bda-4625-9f3b-069625b0ebba' {
	menuentry 'Ubuntu, with Linux 3.19.0-26-generic' --class ubuntu --class gnu-linux --class gnu --class os $menuentry_id_option 'gnulinux-3.19.0-26-generic-advanced-d2a5ed22-3bda-4625-9f3b-069625b0ebba' {
		recordfail
		load_video
		gfxmode $linux_gfx_mode
		insmod gzio
		insmod part_gpt
		insmod ext2
		set root='hd0,gpt6'
		if [ x$feature_platform_search_hint = xy ]; then
		  search --no-floppy --fs-uuid --set=root --hint-bios=hd0,gpt6 --hint-efi=hd0,gpt6 --hint-baremetal=ahci0,gpt6  d2a5ed22-3bda-4625-9f3b-069625b0ebba
		else
		  search --no-floppy --fs-uuid --set=root d2a5ed22-3bda-4625-9f3b-069625b0ebba
		fi
		echo	'Loading Linux 3.19.0-26-generic ...'
		linux	/boot/vmlinuz-3.19.0-26-generic.efi.signed root=UUID=d2a5ed22-3bda-4625-9f3b-069625b0ebba ro  quiet splash $vt_handoff
		echo	'Loading initial ramdisk ...'
		initrd	/boot/initrd.img-3.19.0-26-generic
	}
	menuentry 'Ubuntu, with Linux 3.19.0-26-generic (recovery mode)' --class ubuntu --class gnu-linux --class gnu --class os $menuentry_id_option 'gnulinux-3.19.0-26-generic-recovery-d2a5ed22-3bda-4625-9f3b-069625b0ebba' {
		recordfail
		load_video
		insmod gzio
		insmod part_gpt
		insmod ext2
		set root='hd0,gpt6'
		if [ x$feature_platform_search_hint = xy ]; then
		  search --no-floppy --fs-uuid --set=root --hint-bios=hd0,gpt6 --hint-efi=hd0,gpt6 --hint-baremetal=ahci0,gpt6  d2a5ed22-3bda-4625-9f3b-069625b0ebba
		else
		  search --no-floppy --fs-uuid --set=root d2a5ed22-3bda-4625-9f3b-069625b0ebba
		fi
		echo	'Loading Linux 3.19.0-26-generic ...'
		linux	/boot/vmlinuz-3.19.0-26-generic.efi.signed root=UUID=d2a5ed22-3bda-4625-9f3b-069625b0ebba ro recovery nomodeset 
		echo	'Loading initial ramdisk ...'
		initrd	/boot/initrd.img-3.19.0-26-generic
	}
	menuentry 'Ubuntu, with Linux 3.19.0-25-generic' --class ubuntu --class gnu-linux --class gnu --class os $menuentry_id_option 'gnulinux-3.19.0-25-generic-advanced-d2a5ed22-3bda-4625-9f3b-069625b0ebba' {
		recordfail
		load_video
		gfxmode $linux_gfx_mode
		insmod gzio
		insmod part_gpt
		insmod ext2
		set root='hd0,gpt6'
		if [ x$feature_platform_search_hint = xy ]; then
		  search --no-floppy --fs-uuid --set=root --hint-bios=hd0,gpt6 --hint-efi=hd0,gpt6 --hint-baremetal=ahci0,gpt6  d2a5ed22-3bda-4625-9f3b-069625b0ebba
		else
		  search --no-floppy --fs-uuid --set=root d2a5ed22-3bda-4625-9f3b-069625b0ebba
		fi
		echo	'Loading Linux 3.19.0-25-generic ...'
		linux	/boot/vmlinuz-3.19.0-25-generic root=UUID=d2a5ed22-3bda-4625-9f3b-069625b0ebba ro  quiet splash $vt_handoff
		echo	'Loading initial ramdisk ...'
		initrd	/boot/initrd.img-3.19.0-25-generic
	}
	menuentry 'Ubuntu, with Linux 3.19.0-25-generic (recovery mode)' --class ubuntu --class gnu-linux --class gnu --class os $menuentry_id_option 'gnulinux-3.19.0-25-generic-recovery-d2a5ed22-3bda-4625-9f3b-069625b0ebba' {
		recordfail
		load_video
		insmod gzio
		insmod part_gpt
		insmod ext2
		set root='hd0,gpt6'
		if [ x$feature_platform_search_hint = xy ]; then
		  search --no-floppy --fs-uuid --set=root --hint-bios=hd0,gpt6 --hint-efi=hd0,gpt6 --hint-baremetal=ahci0,gpt6  d2a5ed22-3bda-4625-9f3b-069625b0ebba
		else
		  search --no-floppy --fs-uuid --set=root d2a5ed22-3bda-4625-9f3b-069625b0ebba
		fi
		echo	'Loading Linux 3.19.0-25-generic ...'
		linux	/boot/vmlinuz-3.19.0-25-generic root=UUID=d2a5ed22-3bda-4625-9f3b-069625b0ebba ro recovery nomodeset 
		echo	'Loading initial ramdisk ...'
		initrd	/boot/initrd.img-3.19.0-25-generic
	}
}

### END /etc/grub.d/10_linux ###

### BEGIN /etc/grub.d/20_linux_xen ###

### END /etc/grub.d/20_linux_xen ###

### BEGIN /etc/grub.d/20_memtest86+ ###
### END /etc/grub.d/20_memtest86+ ###

### BEGIN /etc/grub.d/25_custom ###

menuentry "Windows UEFI bootmgfw.efi" {
search --fs-uuid --no-floppy --set=root 86C4-3BF4
chainloader (${root})/EFI/Microsoft/Boot/bootmgfw.efi
}

menuentry "Windows Boot UEFI loader" {
search --fs-uuid --no-floppy --set=root 86C4-3BF4
chainloader (${root})/EFI/Boot/bootx64.efi
}

menuentry "EFI/OEM/Boot/bootmgfw.efi" {
search --fs-uuid --no-floppy --set=root 86C4-3BF4
chainloader (${root})/EFI/OEM/Boot/bootmgfw.efi
}
### END /etc/grub.d/25_custom ###

### BEGIN /etc/grub.d/30_os-prober ###
menuentry 'Windows Boot Manager (on /dev/sda2)' --class windows --class os $menuentry_id_option 'osprober-efi-86C4-3BF4' {
	insmod part_gpt
	insmod fat
	set root='hd0,gpt2'
	if [ x$feature_platform_search_hint = xy ]; then
	  search --no-floppy --fs-uuid --set=root --hint-bios=hd0,gpt2 --hint-efi=hd0,gpt2 --hint-baremetal=ahci0,gpt2  86C4-3BF4
	else
	  search --no-floppy --fs-uuid --set=root 86C4-3BF4
	fi
	chainloader /EFI/Microsoft/Boot/bootmgfw.efi
}
set timeout_style=menu
if [ "${timeout}" = 0 ]; then
  set timeout=10
fi
### END /etc/grub.d/30_os-prober ###

### BEGIN /etc/grub.d/30_uefi-firmware ###
menuentry 'System setup' $menuentry_id_option 'uefi-firmware' {
	fwsetup
}
### END /etc/grub.d/30_uefi-firmware ###

### BEGIN /etc/grub.d/40_custom ###
# This file provides an easy way to add custom menu entries.  Simply type the
# menu entries you want to add after this comment.  Be careful not to change
# the 'exec tail' line above.
### END /etc/grub.d/40_custom ###

### BEGIN /etc/grub.d/41_custom ###
if [ -f  ${config_directory}/custom.cfg ]; then
  source ${config_directory}/custom.cfg
elif [ -z "${config_directory}" -a -f  $prefix/custom.cfg ]; then
  source $prefix/custom.cfg;
fi
### END /etc/grub.d/41_custom ###
--------------------------------------------------------------------------------

=============================== sda6/etc/fstab: ================================

--------------------------------------------------------------------------------
# /etc/fstab: static file system information.
#
# Use 'blkid' to print the universally unique identifier for a
# device; this may be used with UUID= as a more robust way to name devices
# that works even if disks are added and removed. See fstab(5).
#
# <file system> <mount point>   <type>  <options>       <dump>  <pass>
# / was on /dev/sda6 during installation
UUID=d2a5ed22-3bda-4625-9f3b-069625b0ebba /               ext4    errors=remount-ro 0       1
# /boot/efi was on /dev/sda2 during installation
#UUID=86C4-3BF4  /boot/efi       vfat    defaults        0       1
# swap was on /dev/sda7 during installation
UUID=23940c6e-bf1d-426d-98f7-775a96248814 none            swap    sw              0       0
UUID=86C4-3BF4	/boot/efi	vfat	defaults	0	1
--------------------------------------------------------------------------------

=========================== sdb1/boot/grub/grub.cfg: ===========================

--------------------------------------------------------------------------------

if loadfont /boot/grub/font.pf2 ; then
	set gfxmode=auto
	insmod efi_gop
	insmod efi_uga
	insmod gfxterm
	terminal_output gfxterm
fi

set menu_color_normal=white/black
set menu_color_highlight=black/light-gray

menuentry "Try Ubuntu without installing" {
	set gfxpayload=keep
	linux	/casper/vmlinuz.efi  file=/cdrom/preseed/ubuntu.seed boot=casper quiet splash --
	initrd	/casper/initrd.lz
}
menuentry "Install Ubuntu" {
	set gfxpayload=keep
	linux	/casper/vmlinuz.efi  file=/cdrom/preseed/ubuntu.seed boot=casper only-ubiquity quiet splash --
	initrd	/casper/initrd.lz
}
menuentry "OEM install (for manufacturers)" {
	set gfxpayload=keep
	linux	/casper/vmlinuz.efi  file=/cdrom/preseed/ubuntu.seed boot=casper only-ubiquity quiet splash oem-config/enable=true --
	initrd	/casper/initrd.lz
}
menuentry "Check disc for defects" {
	set gfxpayload=keep
	linux	/casper/vmlinuz.efi  boot=casper integrity-check quiet splash --
	initrd	/casper/initrd.lz
}
--------------------------------------------------------------------------------

============================== sdb1/syslinux.cfg: ==============================

--------------------------------------------------------------------------------
default menu.c32
prompt 0
menu title UNetbootin
timeout 100

label unetbootindefault
menu label Default
kernel /ubnkern
append initrd=/ubninit file=/cdrom/preseed/ubuntu.seed boot=casper quiet splash --

label ubnentry0
menu label ^Help
kernel /ubnkern
append initrd=/ubninit 

label ubnentry1
menu label ^Try Ubuntu without installing
kernel /casper/vmlinuz.efi
append initrd=/casper/initrd.lz file=/cdrom/preseed/ubuntu.seed boot=casper  quiet splash --

label ubnentry2
menu label ^Install Ubuntu
kernel /casper/vmlinuz.efi
append initrd=/casper/initrd.lz file=/cdrom/preseed/ubuntu.seed boot=casper only-ubiquity  quiet splash --

label ubnentry3
menu label ^Check disc for defects
kernel /casper/vmlinuz.efi
append initrd=/casper/initrd.lz boot=casper integrity-check  quiet splash --

label ubnentry4
menu label Test ^memory
kernel /install/mt86plus
append initrd=/ubninit 

label ubnentry5
menu label ^Boot from first hard disk
kernel /ubnkern
append initrd=/ubninit 

label ubnentry6
menu label Try Ubuntu without installing
kernel /casper/vmlinuz.efi
append initrd=/casper/initrd.lz file=/cdrom/preseed/ubuntu.seed boot=casper quiet splash --

label ubnentry7
menu label Install Ubuntu
kernel /casper/vmlinuz.efi
append initrd=/casper/initrd.lz file=/cdrom/preseed/ubuntu.seed boot=casper only-ubiquity quiet splash --

label ubnentry8
menu label OEM install (for manufacturers)
kernel /casper/vmlinuz.efi
append initrd=/casper/initrd.lz file=/cdrom/preseed/ubuntu.seed boot=casper only-ubiquity quiet splash oem-config/enable=true --

label ubnentry9
menu label Check disc for defects
kernel /casper/vmlinuz.efi
append initrd=/casper/initrd.lz boot=casper integrity-check quiet splash --

--------------------------------------------------------------------------------

============== sdb1: Version of COM32(R) files used by Syslinux: ===============

 ldlinux.c32                        :  not a COM32/COM32R module
 libcom32.c32                       :  not a COM32/COM32R module
 libutil.c32                        :  not a COM32/COM32R module
 menu.c32                           :  not a COM32/COM32R module

=============================== StdErr Messages: ===============================

File descriptor 9 (/proc/4233/mounts) leaked on lvs invocation. Parent PID 28593: bash
File descriptor 63 (pipe:[53252]) leaked on lvs invocation. Parent PID 28593: bash
  No volume groups found

ADDITIONAL INFORMATION :
=================== log of boot-repair 2015-09-06__20h01 ===================
boot-repair version : 4ppa33
boot-sav version : 4ppa33
glade2script version : 3.2.2~ppa47~saucy
boot-sav-extra version : 4ppa33
boot-repair is executed in live-session (Ubuntu 14.04.3 LTS, trusty, Ubuntu, x86_64)
CPU op-mode(s):        32-bit, 64-bit
BOOT_IMAGE=/casper/vmlinuz.efi file=/cdrom/preseed/ubuntu.seed boot=casper quiet splash --
ls: cannot access /home/usr/.config: No such file or directory

WARNING: GPT (GUID Partition Table) detected on '/dev/sda'! The util fdisk doesn't support GPT. Use GNU Parted.


=================== os-prober:
/dev/sda2@/efi/Microsoft/Boot/bootmgfw.efi:Windows Boot Manager:Windows:efi
/dev/sda6:Ubuntu 14.04.3 LTS (14.04):Ubuntu:linux

=================== blkid:
/dev/loop0: TYPE="squashfs"
/dev/sda1: LABEL="Recovery" UUID="FA92C1CB92C18D17" TYPE="ntfs"
/dev/sda2: LABEL="ESP" UUID="86C4-3BF4" TYPE="vfat"
/dev/sda4: LABEL="Acer" UUID="C69C86AD9C869817" TYPE="ntfs"
/dev/sda5: LABEL="Push Button Reset" UUID="6A80C79F80C76FDD" TYPE="ntfs"
/dev/sda6: UUID="d2a5ed22-3bda-4625-9f3b-069625b0ebba" TYPE="ext4"
/dev/sda7: UUID="23940c6e-bf1d-426d-98f7-775a96248814" TYPE="swap"
/dev/sdb1: UUID="8606-DE46" TYPE="vfat"


1 disks with OS, 2 OS : 1 Linux, 0 MacOS, 1 Windows, 0 unknown type OS.

Windows not detected by os-prober on sda4.

WARNING: GPT (GUID Partition Table) detected on '/dev/sda'! The util sfdisk doesn't support GPT. Use GNU Parted.


WARNING: GPT (GUID Partition Table) detected on '/dev/sda'! The util fdisk doesn't support GPT. Use GNU Parted.

Presence of EFI/Microsoft file detected: /mnt/boot-sav/sda2/EFI/Microsoft/Boot/bootmgfw.efi
Presence of EFI/Boot file detected: /mnt/boot-sav/sda2/EFI/Boot/bootx64.efi

=================== sda6/etc/grub.d/ :
drwxr-xr-x  2 root root    4096 Sep  6 19:23 grub.d
total 80
-rwxr-xr-x 1 root root  9424 Jun 26 11:16 00_header
-rwxr-xr-x 1 root root  6058 May 13 14:51 05_debian_theme
-rwxr-xr-x 1 root root 11608 Jun 26 11:16 10_linux
-rwxr-xr-x 1 root root 10412 Jun 26 11:16 20_linux_xen
-rwxr-xr-x 1 root root  1992 Mar 12  2014 20_memtest86+
-rwxr-xr-x 1 root root   452 Sep  6 19:23 25_custom
-rwxr-xr-x 1 root root 11692 Jun 26 11:16 30_os-prober
-rwxr-xr-x 1 root root  1416 Jun 26 11:16 30_uefi-firmware
-rwxr-xr-x 1 root root   214 Jun 26 11:16 40_custom
-rwxr-xr-x 1 root root   216 Jun 26 11:16 41_custom
-rw-r--r-- 1 root root   483 Jun 26 11:16 README




=================== sda6/etc/default/grub :

# If you change this file, run 'update-grub' afterwards to update
# /boot/grub/grub.cfg.
# For full documentation of the options in this file, see:
#   info -f grub -n 'Simple configuration'

GRUB_DEFAULT=0
#GRUB_HIDDEN_TIMEOUT=0
GRUB_HIDDEN_TIMEOUT_QUIET=true
GRUB_TIMEOUT=10
GRUB_DISTRIBUTOR=`lsb_release -i -s 2> /dev/null || echo Debian`
GRUB_CMDLINE_LINUX_DEFAULT="quiet splash"
GRUB_CMDLINE_LINUX=""

# Uncomment to enable BadRAM filtering, modify to suit your needs
# This works with Linux (no patch required) and with any kernel that obtains
# the memory map information from GRUB (GNU Mach, kernel of FreeBSD ...)
#GRUB_BADRAM="0x01234567,0xfefefefe,0x89abcdef,0xefefefef"

# Uncomment to disable graphical terminal (grub-pc only)
#GRUB_TERMINAL=console

# The resolution used on graphical terminal
# note that you can use only modes which your graphic card supports via VBE
# you can see them in real GRUB with the command `vbeinfo'
#GRUB_GFXMODE=640x480

# Uncomment if you don't want GRUB to pass "root=UUID=xxx" parameter to Linux
#GRUB_DISABLE_LINUX_UUID=true

# Uncomment to disable generation of recovery mode menu entries
#GRUB_DISABLE_RECOVERY="true"

# Uncomment to get a beep at grub start
#GRUB_INIT_TUNE="480 440 1"



/boot/efi detected in the fstab of sda6: UUID=86C4-3BF4	 (sda2)

=================== efibootmgr -v
BootCurrent: 0001
Timeout: 2 seconds
BootOrder: 2001,0000,2002,2003
Boot0000* Windows Boot Manager	HD(2,c8800,96000,7e77172a-71dd-41ec-a543-f688fe140041)File(EFIMicrosoftBootbootmgfw.efi)WINDOWS.........x...B.C.D.O.B.J.E.C.T.=.{.9.d.e.a.8.6.2.c.-.5.c.d.d.-.4.e.7.0.-.a.c.c.1.-.f.3.2.b.3.4.4.d.4.7.9.5.}....................
Boot0001* USB HDD: General USB Flash Disk	ACPI(a0341d0,0)PCI(12,2)USB(0,0)HD(1,3e,3bb5aa,00000000)RC
Boot0002* WDC WD5000LPVX-22V0TT0          	BIOS(2,500,00)................-.d.......d.A.d........................................
Boot0003* General USB Flash Disk  	BIOS(6,500,00).......................................................................
Boot2001* EFI USB Device	RC
Boot2002* EFI DVD/CDROM	RC
Boot2003* EFI Network	RC

=================== UEFI/Legacy mode:
BIOS is EFI-compatible, and is setup in EFI-mode for this live-session.
SecureBoot disabled. (maybe sec-boot, Please report this message to boot.repair@gmail.com)


=================== PARTITIONS & DISKS:
sda1	: sda,	not-sepboot,	no-grubenv	nogrub,	no-docgrub,	no-update-grub,	32,	no-boot,	no-os,	not--efi--part,	part-has-no-fstab,	part-has-no-fstab,	no-nt,	no-winload,	recovery-or-hidden,	no-bmgr,	notwinboot,	nopakmgr,	nogrubinstall,	no---usr,	part-has-no-fstab,	not-sep-usr,	standard,	not-far,	/mnt/boot-sav/sda1.
sda2	: sda,	not-sepboot,	no-grubenv	nogrub,	no-docgrub,	no-update-grub,	32,	no-boot,	is-os,	is-correct-EFI,	part-has-no-fstab,	part-has-no-fstab,	no-nt,	no-winload,	no-recov-nor-hid,	no-bmgr,	notwinboot,	nopakmgr,	nogrubinstall,	no---usr,	part-has-no-fstab,	not-sep-usr,	standard,	not-far,	/mnt/boot-sav/sda2.
sda4	: sda,	not-sepboot,	no-grubenv	nogrub,	no-docgrub,	no-update-grub,	32,	no-boot,	is-os,	not--efi--part,	part-has-no-fstab,	part-has-no-fstab,	no-nt,	haswinload,	no-recov-nor-hid,	no-bmgr,	notwinboot,	nopakmgr,	nogrubinstall,	no---usr,	part-has-no-fstab,	not-sep-usr,	standard,	farbios,	/mnt/boot-sav/sda4.
sda5	: sda,	not-sepboot,	no-grubenv	nogrub,	no-docgrub,	no-update-grub,	32,	no-boot,	no-os,	not--efi--part,	part-has-no-fstab,	part-has-no-fstab,	no-nt,	no-winload,	recovery-or-hidden,	no-bmgr,	notwinboot,	nopakmgr,	nogrubinstall,	no---usr,	part-has-no-fstab,	not-sep-usr,	standard,	farbios,	/mnt/boot-sav/sda5.
sda6	: sda,	not-sepboot,	grubenv-ok	grub2,	signed grub-efi ,	update-grub,	64,	with-boot,	is-os,	not--efi--part,	fstab-without-boot,	fstab-has-goodEFI,	no-nt,	no-winload,	no-recov-nor-hid,	no-bmgr,	notwinboot,	apt-get,	grub-install,	with--usr,	fstab-without-usr,	not-sep-usr,	standard,	farbios,	/mnt/boot-sav/sda6.

sda	: GPT,	no-BIOS_boot,	has-correctEFI, 	not-usb,	has-os,	2048 sectors * 512 bytes


=================== parted -l:

Model: ATA WDC WD5000LPVX-2 (scsi)
Disk /dev/sda: 500GB
Sector size (logical/physical): 512B/4096B
Partition Table: gpt

Number  Start   End    Size    File system     Name                          Flags
1      1049kB  420MB  419MB   ntfs            Basic data partition          hidden, diag
2      420MB   735MB  315MB   fat32           EFI system partition          boot
3      735MB   869MB  134MB                   Microsoft reserved partition  msftres
4      869MB   201GB  200GB   ntfs            Basic data partition          msftdata
6      201GB   479GB  278GB   ext4
7      479GB   485GB  5843MB  linux-swap(v1)
5      485GB   500GB  15.2GB  ntfs            Basic data partition          hidden, diag


Model: General USB Flash Disk (scsi)
Disk /dev/sdb: 2005MB
Sector size (logical/physical): 512B/512B
Partition Table: msdos

Number  Start   End     Size    Type     File system  Flags
1      31.7kB  2004MB  2004MB  primary  fat32        boot, lba

=================== parted -lm:

BYT;
/dev/sda:500GB:scsi:512:4096:gpt:ATA WDC WD5000LPVX-2;
1:1049kB:420MB:419MB:ntfs:Basic data partition:hidden, diag;
2:420MB:735MB:315MB:fat32:EFI system partition:boot;
3:735MB:869MB:134MB::Microsoft reserved partition:msftres;
4:869MB:201GB:200GB:ntfs:Basic data partition:msftdata;
6:201GB:479GB:278GB:ext4::;
7:479GB:485GB:5843MB:linux-swap(v1)::;
5:485GB:500GB:15.2GB:ntfs:Basic data partition:hidden, diag;

BYT;
/dev/sdb:2005MB:scsi:512:512:msdos:General USB Flash Disk;
1:31.7kB:2004MB:2004MB:fat32::boot, lba;

=================== lsblk:
KNAME TYPE FSTYPE     SIZE LABEL    MODEL    UUID
sda   disk          465.8G          WDC WD50
sda1  part ntfs       400M Recovery          FA92C1CB92C18D17
sda2  part vfat       300M ESP               86C4-3BF4
sda3  part            128M
sda4  part ntfs     186.3G Acer              C69C86AD9C869817
sda5  part ntfs      14.1G Push Button Reset
6A80C79F80C76FDD
sda6  part ext4     259.1G                   d2a5ed22-3bda-4625-9f3b-069625b0ebba
sda7  part swap       5.5G                   23940c6e-bf1d-426d-98f7-775a96248814
sdb   disk            1.9G          USB Flas
sdb1  part vfat       1.9G                   8606-DE46
loop0 loop squashfs 962.1M

KNAME ROTA RO RM STATE   MOUNTPOINT
sda      1  0  0 running
sda1     1  0  0         /mnt/boot-sav/sda1
sda2     1  0  0         /mnt/boot-sav/sda2
sda3     1  0  0
sda4     1  0  0         /mnt/boot-sav/sda4
sda5     1  0  0         /mnt/boot-sav/sda5
sda6     1  0  0         /mnt/boot-sav/sda6
sda7     1  0  0         [SWAP]
sdb      1  0  1 running
sdb1     1  0  1         /cdrom
loop0    1  1  0         /rofs


=================== mount:
/cow on / type overlay (rw)
proc on /proc type proc (rw,noexec,nosuid,nodev)
sysfs on /sys type sysfs (rw,noexec,nosuid,nodev)
udev on /dev type devtmpfs (rw,mode=0755)
devpts on /dev/pts type devpts (rw,noexec,nosuid,gid=5,mode=0620)
tmpfs on /run type tmpfs (rw,noexec,nosuid,size=10%,mode=0755)
/dev/sdb1 on /cdrom type vfat (ro,noatime,fmask=0022,dmask=0022,codepage=437,iocharset=iso8859-1,shortname=mixed,errors=remount-ro)
/dev/loop0 on /rofs type squashfs (ro,noatime)
none on /sys/fs/cgroup type tmpfs (rw)
none on /sys/fs/fuse/connections type fusectl (rw)
none on /sys/kernel/debug type debugfs (rw)
none on /sys/kernel/security type securityfs (rw)
none on /sys/firmware/efi/efivars type efivarfs (rw)
tmpfs on /tmp type tmpfs (rw,nosuid,nodev)
none on /run/lock type tmpfs (rw,noexec,nosuid,nodev,size=5242880)
none on /run/shm type tmpfs (rw,nosuid,nodev)
none on /run/user type tmpfs (rw,noexec,nosuid,nodev,size=104857600,mode=0755)
none on /sys/fs/pstore type pstore (rw)
systemd on /sys/fs/cgroup/systemd type cgroup (rw,noexec,nosuid,nodev,none,name=systemd)
gvfsd-fuse on /run/user/999/gvfs type fuse.gvfsd-fuse (rw,nosuid,nodev,user=ubuntu)
/dev/sda1 on /mnt/boot-sav/sda1 type fuseblk (rw,nosuid,nodev,allow_other,blksize=4096)
/dev/sda2 on /mnt/boot-sav/sda2 type vfat (rw)
/dev/sda4 on /mnt/boot-sav/sda4 type fuseblk (rw,nosuid,nodev,allow_other,blksize=4096)
/dev/sda5 on /mnt/boot-sav/sda5 type fuseblk (rw,nosuid,nodev,allow_other,blksize=4096)
/dev/sda6 on /mnt/boot-sav/sda6 type ext4 (rw)


=================== ls:
/sys/block/sda (filtered):  alignment_offset bdi capability dev device discard_alignment events events_async events_poll_msecs ext_range holders inflight power queue range removable ro sda1 sda2 sda3 sda4 sda5 sda6 sda7 size slaves stat subsystem trace uevent
/sys/block/sdb (filtered):  alignment_offset bdi capability dev device discard_alignment events events_async events_poll_msecs ext_range holders inflight power queue range removable ro sdb1 size slaves stat subsystem trace uevent
/dev (filtered):  autofs block bsg btrfs-control bus char console core cpu cpu_dma_latency cuse disk dri ecryptfs fb0 fd full fuse hidraw0 hpet i2c-0 i2c-1 i2c-2 i2c-3 i2c-4 i2c-5 i2c-6 i2c-7 i2c-8 i2c-9 input kfd kmsg log mapper mcelog media0 mem memory_bandwidth net network_latency network_throughput null port ppp psaux ptmx pts random rfkill rtc rtc0 sda sda1 sda2 sda3 sda4 sda5 sda6 sda7 sdb sdb1 sg0 sg1 shm snapshot snd stderr stdin stdout uhid uinput urandom usb v4l vfio vga_arbiter vhci vhost-net video0 zero
ls /dev/mapper:  control

=================== hexdump -n512 -C /dev/sda1
00000000  eb 52 90 4e 54 46 53 20  20 20 20 00 02 08 00 00  |.R.NTFS    .....|
00000010  00 00 00 00 00 f8 00 00  3f 00 ff 00 00 08 00 00  |........?.......|
00000020  00 00 00 00 80 00 80 00  ff 7f 0c 00 00 00 00 00  |................|
00000030  55 85 00 00 00 00 00 00  02 00 00 00 00 00 00 00  |U...............|
00000040  f6 00 00 00 01 00 00 00  17 8d c1 92 cb c1 92 fa  |................|
00000050  00 00 00 00 fa 33 c0 8e  d0 bc 00 7c fb 68 c0 07  |.....3.....|.h..|
00000060  1f 1e 68 66 00 cb 88 16  0e 00 66 81 3e 03 00 4e  |..hf......f.>..N|
00000070  54 46 53 75 15 b4 41 bb  aa 55 cd 13 72 0c 81 fb  |TFSu..A..U..r...|
00000080  55 aa 75 06 f7 c1 01 00  75 03 e9 dd 00 1e 83 ec  |U.u.....u.......|
00000090  18 68 1a 00 b4 48 8a 16  0e 00 8b f4 16 1f cd 13  |.h...H..........|
000000a0  9f 83 c4 18 9e 58 1f 72  e1 3b 06 0b 00 75 db a3  |.....X.r.;...u..|
000000b0  0f 00 c1 2e 0f 00 04 1e  5a 33 db b9 00 20 2b c8  |........Z3... +.|
000000c0  66 ff 06 11 00 03 16 0f  00 8e c2 ff 06 16 00 e8  |f...............|
000000d0  4b 00 2b c8 77 ef b8 00  bb cd 1a 66 23 c0 75 2d  |K.+.w......f#.u-|
000000e0  66 81 fb 54 43 50 41 75  24 81 f9 02 01 72 1e 16  |f..TCPAu$....r..|
000000f0  68 07 bb 16 68 52 11 16  68 09 00 66 53 66 53 66  |h...hR..h..fSfSf|
00000100  55 16 16 16 68 b8 01 66  61 0e 07 cd 1a 33 c0 bf  |U...h..fa....3..|
00000110  0a 13 b9 f6 0c fc f3 aa  e9 fe 01 90 90 66 60 1e  |.............f`.|
00000120  06 66 a1 11 00 66 03 06  1c 00 1e 66 68 00 00 00  |.f...f.....fh...|
00000130  00 66 50 06 53 68 01 00  68 10 00 b4 42 8a 16 0e  |.fP.Sh..h...B...|
00000140  00 16 1f 8b f4 cd 13 66  59 5b 5a 66 59 66 59 1f  |.......fY[ZfYfY.|
00000150  0f 82 16 00 66 ff 06 11  00 03 16 0f 00 8e c2 ff  |....f...........|
00000160  0e 16 00 75 bc 07 1f 66  61 c3 a1 f6 01 e8 09 00  |...u...fa.......|
00000170  a1 fa 01 e8 03 00 f4 eb  fd 8b f0 ac 3c 00 74 09  |............<.t.|
00000180  b4 0e bb 07 00 cd 10 eb  f2 c3 0d 0a 41 20 64 69  |............A di|
00000190  73 6b 20 72 65 61 64 20  65 72 72 6f 72 20 6f 63  |sk read error oc|
000001a0  63 75 72 72 65 64 00 0d  0a 42 4f 4f 54 4d 47 52  |curred...BOOTMGR|
000001b0  20 69 73 20 63 6f 6d 70  72 65 73 73 65 64 00 0d  | is compressed..|
000001c0  0a 50 72 65 73 73 20 43  74 72 6c 2b 41 6c 74 2b  |.Press Ctrl+Alt+|
000001d0  44 65 6c 20 74 6f 20 72  65 73 74 61 72 74 0d 0a  |Del to restart..|
000001e0  00 00 00 00 00 00 00 00  00 00 00 00 00 00 00 00  |................|
000001f0  00 00 00 00 00 00 8a 01  a7 01 bf 01 00 00 55 aa  |..............U.|
00000200

=================== hexdump -n512 -C /dev/sda2
00000000  eb 58 90 4d 53 44 4f 53  35 2e 30 00 02 08 5e 1b  |.X.MSDOS5.0...^.|
00000010  02 00 00 00 00 f8 00 00  3f 00 ff 00 00 88 0c 00  |........?.......|
00000020  00 60 09 00 51 02 00 00  00 00 00 00 02 00 00 00  |.`..Q...........|
00000030  01 00 06 00 00 00 00 00  00 00 00 00 00 00 00 00  |................|
00000040  80 01 29 f4 3b c4 86 4e  4f 20 4e 41 4d 45 20 20  |..).;..NO NAME  |
00000050  20 20 46 41 54 33 32 20  20 20 33 c9 8e d1 bc f4  |  FAT32   3.....|
00000060  7b 8e c1 8e d9 bd 00 7c  88 56 40 88 4e 02 8a 56  |{......|.V@.N..V|
00000070  40 b4 41 bb aa 55 cd 13  72 10 81 fb 55 aa 75 0a  |@.A..U..r...U.u.|
00000080  f6 c1 01 74 05 fe 46 02  eb 2d 8a 56 40 b4 08 cd  |...t..F..-.V@...|
00000090  13 73 05 b9 ff ff 8a f1  66 0f b6 c6 40 66 0f b6  |.s......f...@f..|
000000a0  d1 80 e2 3f f7 e2 86 cd  c0 ed 06 41 66 0f b7 c9  |...?.......Af...|
000000b0  66 f7 e1 66 89 46 f8 83  7e 16 00 75 39 83 7e 2a  |f..f.F..~..u9.~*|
000000c0  00 77 33 66 8b 46 1c 66  83 c0 0c bb 00 80 b9 01  |.w3f.F.f........|
000000d0  00 e8 2c 00 e9 a8 03 a1  f8 7d 80 c4 7c 8b f0 ac  |..,......}..|...|
000000e0  84 c0 74 17 3c ff 74 09  b4 0e bb 07 00 cd 10 eb  |..t.<.t.........|
000000f0  ee a1 fa 7d eb e4 a1 7d  80 eb df 98 cd 16 cd 19  |...}...}........|
00000100  66 60 80 7e 02 00 0f 84  20 00 66 6a 00 66 50 06  |f`.~.... .fj.fP.|
00000110  53 66 68 10 00 01 00 b4  42 8a 56 40 8b f4 cd 13  |Sfh.....B.V@....|
00000120  66 58 66 58 66 58 66 58  eb 33 66 3b 46 f8 72 03  |fXfXfXfX.3f;F.r.|
00000130  f9 eb 2a 66 33 d2 66 0f  b7 4e 18 66 f7 f1 fe c2  |..*f3.f..N.f....|
00000140  8a ca 66 8b d0 66 c1 ea  10 f7 76 1a 86 d6 8a 56  |..f..f....v....V|
00000150  40 8a e8 c0 e4 06 0a cc  b8 01 02 cd 13 66 61 0f  |@............fa.|
00000160  82 74 ff 81 c3 00 02 66  40 49 75 94 c3 42 4f 4f  |.t.....f@Iu..BOO|
00000170  54 4d 47 52 20 20 20 20  00 00 00 00 00 00 00 00  |TMGR    ........|
00000180  00 00 00 00 00 00 00 00  00 00 00 00 00 00 00 00  |................|
*
000001a0  00 00 00 00 00 00 00 00  00 00 00 00 0d 0a 44 69  |..............Di|
000001b0  73 6b 20 65 72 72 6f 72  ff 0d 0a 50 72 65 73 73  |sk error...Press|
000001c0  20 61 6e 79 20 6b 65 79  20 74 6f 20 72 65 73 74  | any key to rest|
000001d0  61 72 74 0d 0a 00 00 00  00 00 00 00 00 00 00 00  |art.............|
000001e0  00 00 00 00 00 00 00 00  00 00 00 00 00 00 00 00  |................|
000001f0  00 00 00 00 00 00 00 00  ac 01 b9 01 00 00 55 aa  |..............U.|
00000200

=================== hexdump -n512 -C /dev/sda4
00000000  eb 52 90 4e 54 46 53 20  20 20 20 00 02 08 00 00  |.R.NTFS    .....|
00000010  00 00 00 00 00 f8 00 00  3f 00 ff 00 00 e8 19 00  |........?.......|
00000020  00 00 00 00 80 00 80 00  18 ce 49 17 00 00 00 00  |..........I.....|
00000030  00 00 0c 00 00 00 00 00  02 00 00 00 00 00 00 00  |................|
00000040  f6 00 00 00 01 00 00 00  17 98 86 9c ad 86 9c c6  |................|
00000050  00 00 00 00 fa 33 c0 8e  d0 bc 00 7c fb 68 c0 07  |.....3.....|.h..|
00000060  1f 1e 68 66 00 cb 88 16  0e 00 66 81 3e 03 00 4e  |..hf......f.>..N|
00000070  54 46 53 75 15 b4 41 bb  aa 55 cd 13 72 0c 81 fb  |TFSu..A..U..r...|
00000080  55 aa 75 06 f7 c1 01 00  75 03 e9 dd 00 1e 83 ec  |U.u.....u.......|
00000090  18 68 1a 00 b4 48 8a 16  0e 00 8b f4 16 1f cd 13  |.h...H..........|
000000a0  9f 83 c4 18 9e 58 1f 72  e1 3b 06 0b 00 75 db a3  |.....X.r.;...u..|
000000b0  0f 00 c1 2e 0f 00 04 1e  5a 33 db b9 00 20 2b c8  |........Z3... +.|
000000c0  66 ff 06 11 00 03 16 0f  00 8e c2 ff 06 16 00 e8  |f...............|
000000d0  4b 00 2b c8 77 ef b8 00  bb cd 1a 66 23 c0 75 2d  |K.+.w......f#.u-|
000000e0  66 81 fb 54 43 50 41 75  24 81 f9 02 01 72 1e 16  |f..TCPAu$....r..|
000000f0  68 07 bb 16 68 52 11 16  68 09 00 66 53 66 53 66  |h...hR..h..fSfSf|
00000100  55 16 16 16 68 b8 01 66  61 0e 07 cd 1a 33 c0 bf  |U...h..fa....3..|
00000110  0a 13 b9 f6 0c fc f3 aa  e9 fe 01 90 90 66 60 1e  |.............f`.|
00000120  06 66 a1 11 00 66 03 06  1c 00 1e 66 68 00 00 00  |.f...f.....fh...|
00000130  00 66 50 06 53 68 01 00  68 10 00 b4 42 8a 16 0e  |.fP.Sh..h...B...|
00000140  00 16 1f 8b f4 cd 13 66  59 5b 5a 66 59 66 59 1f  |.......fY[ZfYfY.|
00000150  0f 82 16 00 66 ff 06 11  00 03 16 0f 00 8e c2 ff  |....f...........|
00000160  0e 16 00 75 bc 07 1f 66  61 c3 a1 f6 01 e8 09 00  |...u...fa.......|
00000170  a1 fa 01 e8 03 00 f4 eb  fd 8b f0 ac 3c 00 74 09  |............<.t.|
00000180  b4 0e bb 07 00 cd 10 eb  f2 c3 0d 0a 45 72 72 65  |............Erre|
00000190  75 72 20 6c 65 63 74 75  72 65 20 64 69 73 71 75  |ur lecture disqu|
000001a0  65 00 0d 0a 42 4f 4f 54  4d 47 52 20 63 6f 6d 70  |e...BOOTMGR comp|
000001b0  72 65 73 73 82 00 0d 0a  43 74 72 6c 2b 41 6c 74  |ress....Ctrl+Alt|
000001c0  2b 53 75 70 70 72 20 70  6f 75 72 20 72 65 64 82  |+Suppr pour red.|
000001d0  6d 61 72 72 65 72 0d 0a  00 73 74 61 72 74 0d 0a  |marrer...start..|
000001e0  00 00 00 00 00 00 00 00  00 00 00 00 00 00 00 00  |................|
000001f0  00 00 00 00 00 00 8a 01  a2 01 b6 01 00 00 55 aa  |..............U.|
00000200

=================== hexdump -n512 -C /dev/sda5
00000000  eb 52 90 4e 54 46 53 20  20 20 20 00 02 08 00 00  |.R.NTFS    .....|
00000010  00 00 00 00 00 f8 00 00  3f 00 ff 00 00 78 74 38  |........?....xt8|
00000020  00 00 00 00 80 00 80 00  ff e7 c3 01 00 00 00 00  |................|
00000030  00 00 0c 00 00 00 00 00  02 00 00 00 00 00 00 00  |................|
00000040  f6 00 00 00 01 00 00 00  dd 6f c7 80 9f c7 80 6a  |.........o.....j|
00000050  00 00 00 00 fa 33 c0 8e  d0 bc 00 7c fb 68 c0 07  |.....3.....|.h..|
00000060  1f 1e 68 66 00 cb 88 16  0e 00 66 81 3e 03 00 4e  |..hf......f.>..N|
00000070  54 46 53 75 15 b4 41 bb  aa 55 cd 13 72 0c 81 fb  |TFSu..A..U..r...|
00000080  55 aa 75 06 f7 c1 01 00  75 03 e9 dd 00 1e 83 ec  |U.u.....u.......|
00000090  18 68 1a 00 b4 48 8a 16  0e 00 8b f4 16 1f cd 13  |.h...H..........|
000000a0  9f 83 c4 18 9e 58 1f 72  e1 3b 06 0b 00 75 db a3  |.....X.r.;...u..|
000000b0  0f 00 c1 2e 0f 00 04 1e  5a 33 db b9 00 20 2b c8  |........Z3... +.|
000000c0  66 ff 06 11 00 03 16 0f  00 8e c2 ff 06 16 00 e8  |f...............|
000000d0  4b 00 2b c8 77 ef b8 00  bb cd 1a 66 23 c0 75 2d  |K.+.w......f#.u-|
000000e0  66 81 fb 54 43 50 41 75  24 81 f9 02 01 72 1e 16  |f..TCPAu$....r..|
000000f0  68 07 bb 16 68 52 11 16  68 09 00 66 53 66 53 66  |h...hR..h..fSfSf|
00000100  55 16 16 16 68 b8 01 66  61 0e 07 cd 1a 33 c0 bf  |U...h..fa....3..|
00000110  0a 13 b9 f6 0c fc f3 aa  e9 fe 01 90 90 66 60 1e  |.............f`.|
00000120  06 66 a1 11 00 66 03 06  1c 00 1e 66 68 00 00 00  |.f...f.....fh...|
00000130  00 66 50 06 53 68 01 00  68 10 00 b4 42 8a 16 0e  |.fP.Sh..h...B...|
00000140  00 16 1f 8b f4 cd 13 66  59 5b 5a 66 59 66 59 1f  |.......fY[ZfYfY.|
00000150  0f 82 16 00 66 ff 06 11  00 03 16 0f 00 8e c2 ff  |....f...........|
00000160  0e 16 00 75 bc 07 1f 66  61 c3 a1 f6 01 e8 09 00  |...u...fa.......|
00000170  a1 fa 01 e8 03 00 f4 eb  fd 8b f0 ac 3c 00 74 09  |............<.t.|
00000180  b4 0e bb 07 00 cd 10 eb  f2 c3 0d 0a 41 20 64 69  |............A di|
00000190  73 6b 20 72 65 61 64 20  65 72 72 6f 72 20 6f 63  |sk read error oc|
000001a0  63 75 72 72 65 64 00 0d  0a 42 4f 4f 54 4d 47 52  |curred...BOOTMGR|
000001b0  20 69 73 20 63 6f 6d 70  72 65 73 73 65 64 00 0d  | is compressed..|
000001c0  0a 50 72 65 73 73 20 43  74 72 6c 2b 41 6c 74 2b  |.Press Ctrl+Alt+|
000001d0  44 65 6c 20 74 6f 20 72  65 73 74 61 72 74 0d 0a  |Del to restart..|
000001e0  00 00 00 00 00 00 00 00  00 00 00 00 00 00 00 00  |................|
000001f0  00 00 00 00 00 00 8a 01  a7 01 bf 01 00 00 55 aa  |..............U.|
00000200

WARNING: GPT (GUID Partition Table) detected on '/dev/sda'! The util fdisk doesn't support GPT. Use GNU Parted.


=================== df -Th:

Filesystem     Type      Size  Used Avail Use% Mounted on
/cow           overlay   2.7G  106M  2.6G   4% /
udev           devtmpfs  2.6G   12K  2.6G   1% /dev
tmpfs          tmpfs     535M  1.3M  533M   1% /run
/dev/sdb1      vfat      1.9G  1.1G  875M  55% /cdrom
/dev/loop0     squashfs  963M  963M     0 100% /rofs
none           tmpfs     4.0K     0  4.0K   0% /sys/fs/cgroup
tmpfs          tmpfs     2.7G  1.2M  2.7G   1% /tmp
none           tmpfs     5.0M     0  5.0M   0% /run/lock
none           tmpfs     2.7G   80K  2.7G   1% /run/shm
none           tmpfs     100M   60K  100M   1% /run/user
/dev/sda1      fuseblk   400M  271M  130M  68% /mnt/boot-sav/sda1
/dev/sda2      vfat      296M   53M  244M  18% /mnt/boot-sav/sda2
/dev/sda4      fuseblk   187G   29G  158G  16% /mnt/boot-sav/sda4
/dev/sda5      fuseblk    15G   13G  1.4G  91% /mnt/boot-sav/sda5
/dev/sda6      ext4      255G  4.3G  238G   2% /mnt/boot-sav/sda6

=================== fdisk -l:

Disk /dev/sda: 500.1 GB, 500107862016 bytes
255 heads, 63 sectors/track, 60801 cylinders, total 976773168 sectors
Units = sectors of 1 * 512 = 512 bytes
Sector size (logical/physical): 512 bytes / 4096 bytes
I/O size (minimum/optimal): 4096 bytes / 4096 bytes
Disk identifier: 0x45cb590b

Device Boot      Start         End      Blocks   Id  System
/dev/sda1               1   976773167   488386583+  ee  GPT
Partition 1 does not start on physical sector boundary.

Disk /dev/sdb: 2004 MB, 2004877312 bytes
62 heads, 62 sectors/track, 1018 cylinders, total 3915776 sectors
Units = sectors of 1 * 512 = 512 bytes
Sector size (logical/physical): 512 bytes / 512 bytes
I/O size (minimum/optimal): 512 bytes / 512 bytes
Disk identifier: 0x00000000

Device Boot      Start         End      Blocks   Id  System
/dev/sdb1   *          62     3913191     1956565    c  W95 FAT32 (LBA)



=================== Recommended repair
The default repair of the Boot-Repair utility will reinstall the grub-efi-amd64-signed of sda6, using the following options:        sda2/boot/efi,
Additional repair will be performed: unhide-bootmenu-10s   fix-windows-boot use-standard-efi-file


Quantity of real Windows: 1
Mount sda2 on /mnt/boot-sav/sda6/boot/efi
ls sda2/efi: /OEM/Boot/zh-TW /OEM/Boot/zh-HK /OEM/Boot/zh-CN /OEM/Boot/uk-UA /OEM/Boot/tr-TR /OEM/Boot/sv-SE /OEM/Boot/sr-Latn-CS /OEM/Boot/sl-SI /OEM/Boot/sk-SK /OEM/Boot/ru-RU /OEM/Boot/ro-RO /OEM/Boot/Resources /OEM/Boot/qps-ploc /OEM/Boot/pt-PT /OEM/Boot/pt-BR /OEM/Boot/pl-PL /OEM/Boot/nl-NL /OEM/Boot/nb-NO /OEM/Boot/memtest.efi /OEM/Boot/lv-LV /OEM/Boot/lt-LT /OEM/Boot/ko-KR /OEM/Boot/ja-JP /OEM/Boot/it-IT /OEM/Boot/hu-HU /OEM/Boot/hr-HR /OEM/Boot/fr-FR /OEM/Boot/Fonts /OEM/Boot/fi-FI /OEM/Boot/et-EE /OEM/Boot/es-ES /OEM/Boot/en-US /OEM/Boot/en-GB /OEM/Boot/el-GR /OEM/Boot/de-DE /OEM/Boot/da-DK /OEM/Boot/cs-CZ /OEM/Boot/boot.stl /OEM/Boot/BOOTSTAT.DAT /OEM/Boot/bootmgr.efi /OEM/Boot/bootmgfw.efi /OEM/Boot/bg-BG /OEM/Boot/BCD.LOG2 /OEM/Boot/BCD.LOG1 /OEM/Boot/BCD.LOG /OEM/Boot/BCD /Microsoft/Boot/zh-TW /Microsoft/Boot/zh-HK /Microsoft/Boot/zh-CN /Microsoft/Boot/uk-UA /Microsoft/Boot/tr-TR /Microsoft/Boot/sv-SE /Microsoft/Boot/sr-Latn-CS /Microsoft/Boot/sl-SI /Microsoft/Boot/sk-SK /Microsoft/Boot/ru-RU /Microsoft/Boot/ro-RO /Microsoft/Boot/Resources /Microsoft/Boot/qps-ploc /Microsoft/Boot/pt-PT /Microsoft/Boot/pt-BR /Microsoft/Boot/pl-PL /Microsoft/Boot/nl-NL /Microsoft/Boot/nb-NO /Microsoft/Boot/memtest.efi /Microsoft/Boot/lv-LV /Microsoft/Boot/lt-LT /Microsoft/Boot/ko-KR /Microsoft/Boot/ja-JP /Microsoft/Boot/it-IT /Microsoft/Boot/hu-HU /Microsoft/Boot/hr-HR /Microsoft/Boot/fr-FR /Microsoft/Boot/Fonts /Microsoft/Boot/fi-FI /Microsoft/Boot/et-EE /Microsoft/Boot/es-ES /Microsoft/Boot/en-US /Microsoft/Boot/en-GB /Microsoft/Boot/el-GR /Microsoft/Boot/de-DE /Microsoft/Boot/da-DK /Microsoft/Boot/cs-CZ /Microsoft/Boot/boot.stl /Microsoft/Boot/BOOTSTAT.DAT /Microsoft/Boot/bootmgr.efi /Microsoft/Boot/bootmgfw.efi /Microsoft/Boot/bg-BG /Microsoft/Boot/BCD.LOG2 /Microsoft/Boot/BCD.LOG1 /Microsoft/Boot/BCD.LOG /Microsoft/Boot/BCD /OEM/Boot /Microsoft/Boot /Boot/bootx64.efi
ls sda2: boot-sav
EFI
FSCK0000.REC
FSCK0001.REC
FSCK0002.REC
FSCK0003.REC  . Please report this message to boot.repair@gmail.com

*******lspci -nnk | grep -iA3 vga
00:01.0 VGA compatible controller [0300]: Advanced Micro Devices, Inc. [AMD/ATI] Temash [Radeon HD 8250/8280G] [1002:983d]
Subsystem: Acer Incorporated [ALI] Device [1025:080d]
Kernel driver in use: radeon
00:01.1 Audio device [0403]: Advanced Micro Devices, Inc. [AMD/ATI] Device [1002:9840]
*******

grub-install --version
grub-install (GRUB) 2.02~beta2-9ubuntu1.3,grub-install (GRUB) 2.

chroot /mnt/boot-sav/sda6 efibootmgr -v
BootCurrent: 0001
Timeout: 2 seconds
BootOrder: 2001,0000,2002,2003
Boot0000* Windows Boot Manager	HD(2,c8800,96000,7e77172a-71dd-41ec-a543-f688fe140041)File(EFIMicrosoftBootbootmgfw.efi)WINDOWS.........x...B.C.D.O.B.J.E.C.T.=.{.9.d.e.a.8.6.2.c.-.5.c.d.d.-.4.e.7.0.-.a.c.c.1.-.f.3.2.b.3.4.4.d.4.7.9.5.}....................
Boot0001* USB HDD: General USB Flash Disk	ACPI(a0341d0,0)PCI(12,2)USB(0,0)HD(1,3e,3bb5aa,00000000)RC
Boot0002* WDC WD5000LPVX-22V0TT0          	BIOS(2,500,00)................-.d.......d.A.d........................................
Boot0003* General USB Flash Disk  	BIOS(6,500,00).......................................................................
Boot2001* EFI USB Device	RC
Boot2002* EFI DVD/CDROM	RC
Boot2003* EFI Network	RC

chroot /mnt/boot-sav/sda6 uname -r
Kernel: 3.19.0-25-generic

Reinstall the grub-efi-amd64-signed of sda6
Installing for x86_64-efi platform.
grub-install: error: cannot open `/boot/efi/EFI/ubuntu/shimx64.efi': Input/output error.
grub-install --efi-directory=/boot/efi --target=x86_64-efi --uefi-secure-boot : exit code of grub-install :1
mv 25_custom

dosfsck -a /dev/sda2
fsck.fat 3.0.26 (2014-03-07)
0x41: Dirty bit is set. Fs was not properly unmounted and some data may be corrupt.
Automatically removing dirty bit.
/EFI/ubuntu
Start does point to root directory. Deleting dir.
Performing changes.
/dev/sda2: 361 files, 13660/75776 clusters
Installing for x86_64-efi platform.
grub-install: error: cannot open `/boot/efi/EFI/ubuntu/shimx64.efi': Input/output error.
grub-install --efi-directory=/boot/efi --target=x86_64-efi --uefi-secure-boot : exit code of grub-install :1

rm -Rf /dev/sda2/ubuntu .. fedora
Installing for x86_64-efi platform.
grub-install: error: cannot open `/boot/efi/EFI/ubuntu/shimx64.efi': Input/output error.
grub-install --efi-directory=/boot/efi --target=x86_64-efi --uefi-secure-boot : exit code of grub-install :1
=================== (Write-locked ESP) dmesg:
[    0.000000] Initializing cgroup subsys cpuset
[    0.000000] Initializing cgroup subsys cpu
[    0.000000] Initializing cgroup subsys cpuacct
[    0.000000] Linux version 3.19.0-25-generic (buildd@lgw01-20) (gcc version 4.8.2 (Ubuntu 4.8.2-19ubuntu1) ) #26~14.04.1-Ubuntu SMP Fri Jul 24 21:16:20 UTC 2015 (Ubuntu 3.19.0-25.26~14.04.1-generic 3.19.8-ckt2)
[    0.000000] Command line: BOOT_IMAGE=/casper/vmlinuz.efi file=/cdrom/preseed/ubuntu.seed boot=casper quiet splash --
[    0.000000] KERNEL supported cpus:
[    0.000000]   Intel GenuineIntel
[    0.000000]   AMD AuthenticAMD
[    0.000000]   Centaur CentaurHauls
[    0.000000] tseg: 00bfc00000
[    0.000000] e820: BIOS-provided physical RAM map:
[    0.000000] BIOS-e820: [mem 0x0000000000000000-0x000000000006dfff] usable
[    0.000000] BIOS-e820: [mem 0x000000000006e000-0x000000000006ffff] ACPI NVS
[    0.000000] BIOS-e820: [mem 0x0000000000070000-0x0000000000085fff] usable
[    0.000000] BIOS-e820: [mem 0x0000000000086000-0x00000000000bffff] reserved
[    0.000000] BIOS-e820: [mem 0x0000000000100000-0x00000000b4b1cfff] usable
[    0.000000] BIOS-e820: [mem 0x00000000b4b1d000-0x00000000b531dfff] reserved
[    0.000000] BIOS-e820: [mem 0x00000000b531e000-0x00000000b537bfff] usable
[    0.000000] BIOS-e820: [mem 0x00000000b537c000-0x00000000b677bfff] reserved
[    0.000000] BIOS-e820: [mem 0x00000000b677c000-0x00000000bf17efff] usable
[    0.000000] BIOS-e820: [mem 0x00000000bf17f000-0x00000000bf57efff] type 20
[    0.000000] BIOS-e820: [mem 0x00000000bf57f000-0x00000000bf97efff] reserved
[    0.000000] BIOS-e820: [mem 0x00000000bf97f000-0x00000000bfb7efff] ACPI NVS
[    0.000000] BIOS-e820: [mem 0x00000000bfb7f000-0x00000000bfbfefff] ACPI data
[    0.000000] BIOS-e820: [mem 0x00000000bfbff000-0x00000000bfbfffff] usable
[    0.000000] BIOS-e820: [mem 0x00000000bfc00000-0x00000000dfffffff] reserved
[    0.000000] BIOS-e820: [mem 0x00000000f8000000-0x00000000fbffffff] reserved
[    0.000000] BIOS-e820: [mem 0x00000000fec00000-0x00000000fec00fff] reserved
[    0.000000] BIOS-e820: [mem 0x00000000fec10000-0x00000000fec10fff] reserved
[    0.000000] BIOS-e820: [mem 0x00000000fed80000-0x00000000fed80fff] reserved
[    0.000000] BIOS-e820: [mem 0x00000000fee00000-0x00000000fee00fff] reserved
[    0.000000] BIOS-e820: [mem 0x00000000ffc00000-0x00000000ffffffff] reserved
[    0.000000] BIOS-e820: [mem 0x0000000100000000-0x000000019effffff] usable
[    0.000000] NX (Execute Disable) protection: active
[    0.000000] efi: EFI v2.31 by INSYDE Corp.
[    0.000000] efi:  ACPI=0xbfbfe000  ACPI 2.0=0xbfbfe014  SMBIOS=0xbf97df18
[    0.000000] efi: mem00: [Boot Code          |   |  |  |  |   |WB|WT|WC|UC] range=[0x0000000000000000-0x0000000000001000) (0MB)
[    0.000000] efi: mem01: [Conventional Memory|   |  |  |  |   |WB|WT|WC|UC] range=[0x0000000000001000-0x000000000006e000) (0MB)
[    0.000000] efi: mem02: [ACPI Memory NVS    |   |  |  |  |   |WB|WT|WC|UC] range=[0x000000000006e000-0x0000000000070000) (0MB)
[    0.000000] efi: mem03: [Conventional Memory|   |  |  |  |   |WB|WT|WC|UC] range=[0x0000000000070000-0x0000000000086000) (0MB)
[    0.000000] efi: mem04: [Reserved           |   |  |  |  |   |WB|WT|WC|UC] range=[0x0000000000086000-0x0000000000088000) (0MB)
[    0.000000] efi: mem05: [Runtime Data       |RUN|  |  |  |   |WB|WT|WC|UC] range=[0x0000000000088000-0x00000000000a0000) (0MB)
[    0.000000] efi: mem06: [Loader Data        |   |  |  |  |   |WB|WT|WC|UC] range=[0x0000000000100000-0x00000000014d6000) (19MB)
[    0.000000] efi: mem07: [Conventional Memory|   |  |  |  |   |WB|WT|WC|UC] range=[0x00000000014d6000-0x0000000002000000) (11MB)
[    0.000000] efi: mem08: [Loader Data        |   |  |  |  |   |WB|WT|WC|UC] range=[0x0000000002000000-0x00000000033d6000) (19MB)
[    0.000000] efi: mem09: [Conventional Memory|   |  |  |  |   |WB|WT|WC|UC] range=[0x00000000033d6000-0x000000003568c000) (802MB)
[    0.000000] efi: mem10: [Loader Data        |   |  |  |  |   |WB|WT|WC|UC] range=[0x000000003568c000-0x0000000036b3e000) (20MB)
[    0.000000] efi: mem11: [Conventional Memory|   |  |  |  |   |WB|WT|WC|UC] range=[0x0000000036b3e000-0x0000000084f30000) (1251MB)
[    0.000000] efi: mem12: [Loader Data        |   |  |  |  |   |WB|WT|WC|UC] range=[0x0000000084f30000-0x00000000b3780000) (744MB)
[    0.000000] efi: mem13: [Boot Data          |   |  |  |  |   |WB|WT|WC|UC] range=[0x00000000b3780000-0x00000000b37a0000) (0MB)
[    0.000000] efi: mem14: [Conventional Memory|   |  |  |  |   |WB|WT|WC|UC] range=[0x00000000b37a0000-0x00000000b471d000) (15MB)
[    0.000000] efi: mem15: [Loader Data        |   |  |  |  |   |WB|WT|WC|UC] range=[0x00000000b471d000-0x00000000b4903000) (1MB)
[    0.000000] efi: mem16: [Boot Data          |   |  |  |  |   |WB|WT|WC|UC] range=[0x00000000b4903000-0x00000000b4b1d000) (2MB)
[    0.000000] efi: mem17: [Reserved           |   |  |  |  |   |WB|WT|WC|UC] range=[0x00000000b4b1d000-0x00000000b531e000) (8MB)
[    0.000000] efi: mem18: [Boot Data          |   |  |  |  |   |WB|WT|WC|UC] range=[0x00000000b531e000-0x00000000b537c000) (0MB)
[    0.000000] efi: mem19: [Reserved           |   |  |  |  |   |WB|WT|WC|UC] range=[0x00000000b537c000-0x00000000b677c000) (20MB)
[    0.000000] efi: mem20: [Conventional Memory|   |  |  |  |   |WB|WT|WC|UC] range=[0x00000000b677c000-0x00000000b677d000) (0MB)
[    0.000000] efi: mem21: [Loader Data        |   |  |  |  |   |WB|WT|WC|UC] range=[0x00000000b677d000-0x00000000b677f000) (0MB)
[    0.000000] efi: mem22: [Conventional Memory|   |  |  |  |   |WB|WT|WC|UC] range=[0x00000000b677f000-0x00000000b6859000) (0MB)
[    0.000000] efi: mem23: [Loader Code        |   |  |  |  |   |WB|WT|WC|UC] range=[0x00000000b6859000-0x00000000b697f000) (1MB)
[    0.000000] efi: mem24: [Conventional Memory|   |  |  |  |   |WB|WT|WC|UC] range=[0x00000000b697f000-0x00000000bbf63000) (85MB)
[    0.000000] efi: mem25: [Boot Data          |   |  |  |  |   |WB|WT|WC|UC] range=[0x00000000bbf63000-0x00000000bcd08000) (13MB)
[    0.000000] efi: mem26: [Conventional Memory|   |  |  |  |   |WB|WT|WC|UC] range=[0x00000000bcd08000-0x00000000bcd0a000) (0MB)
[    0.000000] efi: mem27: [Boot Data          |   |  |  |  |   |WB|WT|WC|UC] range=[0x00000000bcd0a000-0x00000000bcdff000) (0MB)
[    0.000000] efi: mem28: [Conventional Memory|   |  |  |  |   |WB|WT|WC|UC] range=[0x00000000bcdff000-0x00000000bce04000) (0MB)
[    0.000000] efi: mem29: [Boot Data          |   |  |  |  |   |WB|WT|WC|UC] range=[0x00000000bce04000-0x00000000bce39000) (0MB)
[    0.000000] efi: mem30: [Conventional Memory|   |  |  |  |   |WB|WT|WC|UC] range=[0x00000000bce39000-0x00000000bce3c000) (0MB)
[    0.000000] efi: mem31: [Boot Data          |   |  |  |  |   |WB|WT|WC|UC] range=[0x00000000bce3c000-0x00000000bcfbe000) (1MB)
[    0.000000] efi: mem32: [Conventional Memory|   |  |  |  |   |WB|WT|WC|UC] range=[0x00000000bcfbe000-0x00000000bd022000) (0MB)
[    0.000000] efi: mem33: [Boot Data          |   |  |  |  |   |WB|WT|WC|UC] range=[0x00000000bd022000-0x00000000be97f000) (25MB)
[    0.000000] efi: mem34: [Conventional Memory|   |  |  |  |   |WB|WT|WC|UC] range=[0x00000000be97f000-0x00000000bed92000) (4MB)
[    0.000000] efi: mem35: [Loader Data        |   |  |  |  |   |WB|WT|WC|UC] range=[0x00000000bed92000-0x00000000bed9c000) (0MB)
[    0.000000] efi: mem36: [Boot Code          |   |  |  |  |   |WB|WT|WC|UC] range=[0x00000000bed9c000-0x00000000bf17f000) (3MB)
[    0.000000] efi: mem37: [Runtime Code       |RUN|  |  |  |   |WB|WT|WC|UC] range=[0x00000000bf17f000-0x00000000bf57f000) (4MB)
[    0.000000] efi: mem38: [Runtime Data       |RUN|  |  |  |   |WB|WT|WC|UC] range=[0x00000000bf57f000-0x00000000bf77f000) (2MB)
[    0.000000] efi: mem39: [Reserved           |   |  |  |  |   |WB|WT|WC|UC] range=[0x00000000bf77f000-0x00000000bf97f000) (2MB)
[    0.000000] efi: mem40: [ACPI Memory NVS    |   |  |  |  |   |WB|WT|WC|UC] range=[0x00000000bf97f000-0x00000000bfb7f000) (2MB)
[    0.000000] efi: mem41: [ACPI Reclaim Memory|   |  |  |  |   |WB|WT|WC|UC] range=[0x00000000bfb7f000-0x00000000bfbff000) (0MB)
[    0.000000] efi: mem42: [Boot Data          |   |  |  |  |   |WB|WT|WC|UC] range=[0x00000000bfbff000-0x00000000bfc00000) (0MB)
[    0.000000] efi: mem43: [Conventional Memory|   |  |  |  |   |WB|WT|WC|UC] range=[0x0000000100000000-0x000000019f000000) (2544MB)
[    0.000000] efi: mem44: [Reserved           |   |  |  |  |   |  |  |  |  ] range=[0x00000000000a0000-0x00000000000c0000) (0MB)
[    0.000000] efi: mem45: [Reserved           |   |  |  |  |   |  |  |  |  ] range=[0x00000000bfc00000-0x00000000e0000000) (516MB)
[    0.000000] efi: mem46: [Memory Mapped I/O  |RUN|  |  |  |   |  |  |  |UC] range=[0x00000000f8000000-0x00000000fc000000) (64MB)
[    0.000000] efi: mem47: [Memory Mapped I/O  |RUN|  |  |  |   |  |  |  |UC] range=[0x00000000fec00000-0x00000000fec01000) (0MB)
[    0.000000] efi: mem48: [Memory Mapped I/O  |RUN|  |  |  |   |  |  |  |UC] range=[0x00000000fec10000-0x00000000fec11000) (0MB)
[    0.000000] efi: mem49: [Memory Mapped I/O  |RUN|  |  |  |   |  |  |  |UC] range=[0x00000000fed80000-0x00000000fed81000) (0MB)
[    0.000000] efi: mem50: [Memory Mapped I/O  |RUN|  |  |  |   |  |  |  |UC] range=[0x00000000fee00000-0x00000000fee01000) (0MB)
[    0.000000] efi: mem51: [Memory Mapped I/O  |RUN|  |  |  |   |  |  |  |  ] range=[0x00000000ffc00000-0x0000000100000000) (4MB)
[    0.000000] SMBIOS 2.7 present.
[    0.000000] DMI: Acer Aspire V5-122P/Aspire V5-122P, BIOS V2.04 05/10/2013
[    0.000000] e820: update [mem 0x00000000-0x00000fff] usable ==> reserved
[    0.000000] e820: remove [mem 0x000a0000-0x000fffff] usable
[    0.000000] AGP: No AGP bridge found
[    0.000000] e820: last_pfn = 0x19f000 max_arch_pfn = 0x400000000
[    0.000000] MTRR default type: uncachable
[    0.000000] MTRR fixed ranges enabled:
[    0.000000]   00000-9FFFF write-back
[    0.000000]   A0000-BFFFF uncachable
[    0.000000]   C0000-FFFFF write-through
[    0.000000] MTRR variable ranges enabled:
[    0.000000]   0 base 0000000000 mask FF80000000 write-back
[    0.000000]   1 base 0080000000 mask FFC0000000 write-back
[    0.000000]   2 base 00BFB67000 mask FFFFFFF000 uncachable
[    0.000000]   3 base 00FFC00000 mask FFFFC00000 write-protect
[    0.000000]   4 disabled
[    0.000000]   5 disabled
[    0.000000]   6 disabled
[    0.000000]   7 disabled
[    0.000000] TOM2: 000000019f000000 aka 6640M
[    0.000000] PAT configuration [0-7]: WB  WC  UC- UC  WB  WC  UC- UC
[    0.000000] e820: last_pfn = 0xbfc00 max_arch_pfn = 0x400000000
[    0.000000] Scanning 1 areas for low memory corruption
[    0.000000] Base memory trampoline at [ffff88000007c000] 7c000 size 24576
[    0.000000] Using GB pages for direct mapping
[    0.000000] init_memory_mapping: [mem 0x00000000-0x000fffff]
[    0.000000]  [mem 0x00000000-0x000fffff] page 4k
[    0.000000] BRK [0x02fd4000, 0x02fd4fff] PGTABLE
[    0.000000] BRK [0x02fd5000, 0x02fd5fff] PGTABLE
[    0.000000] BRK [0x02fd6000, 0x02fd6fff] PGTABLE
[    0.000000] init_memory_mapping: [mem 0x19ee00000-0x19effffff]
[    0.000000]  [mem 0x19ee00000-0x19effffff] page 2M
[    0.000000] BRK [0x02fd7000, 0x02fd7fff] PGTABLE
[    0.000000] init_memory_mapping: [mem 0x180000000-0x19edfffff]
[    0.000000]  [mem 0x180000000-0x19edfffff] page 2M
[    0.000000] init_memory_mapping: [mem 0x160000000-0x17fffffff]
[    0.000000]  [mem 0x160000000-0x17fffffff] page 1G
[    0.000000] init_memory_mapping: [mem 0x00100000-0xb4b1cfff]
[    0.000000]  [mem 0x00100000-0x001fffff] page 4k
[    0.000000]  [mem 0x00200000-0x3fffffff] page 2M
[    0.000000]  [mem 0x40000000-0x7fffffff] page 1G
[    0.000000]  [mem 0x80000000-0xb49fffff] page 2M
[    0.000000]  [mem 0xb4a00000-0xb4b1cfff] page 4k
[    0.000000] init_memory_mapping: [mem 0xb531e000-0xb537bfff]
[    0.000000]  [mem 0xb531e000-0xb537bfff] page 4k
[    0.000000] BRK [0x02fd8000, 0x02fd8fff] PGTABLE
[    0.000000] init_memory_mapping: [mem 0xb677c000-0xbf17efff]
[    0.000000]  [mem 0xb677c000-0xb67fffff] page 4k
[    0.000000]  [mem 0xb6800000-0xbeffffff] page 2M
[    0.000000]  [mem 0xbf000000-0xbf17efff] page 4k
[    0.000000] BRK [0x02fd9000, 0x02fd9fff] PGTABLE
[    0.000000] init_memory_mapping: [mem 0xbfbff000-0xbfbfffff]
[    0.000000]  [mem 0xbfbff000-0xbfbfffff] page 4k
[    0.000000] init_memory_mapping: [mem 0x100000000-0x15fffffff]
[    0.000000]  [mem 0x100000000-0x15fffffff] page 1G
[    0.000000] RAMDISK: [mem 0x3568c000-0x36b3dfff]
[    0.000000] ACPI: Early table checksum verification disabled
[    0.000000] ACPI: RSDP 0x00000000BFBFE014 000024 (v02 ACRSYS)
[    0.000000] ACPI: XSDT 0x00000000BFBFE120 0000B4 (v01 ACRSYS ACRPRDCT 00000001      01000013)
[    0.000000] ACPI: FACP 0x00000000BFBFC000 00010C (v05 ACRSYS ACRPRDCT 00000001 1025 00040000)
[    0.000000] ACPI: DSDT 0x00000000BFBEE000 009FDC (v01 ACRSYS ACRPRDCT F0000000 1025 00040000)
[    0.000000] ACPI: FACS 0x00000000BFB26000 000040
[    0.000000] ACPI: UEFI 0x00000000BFBFD000 000236 (v01 ACRSYS ACRPRDCT 00000001 1025 00040000)
[    0.000000] ACPI: HPET 0x00000000BFBFB000 000038 (v01 ACRSYS ACRPRDCT 00000001 1025 00040000)
[    0.000000] ACPI: APIC 0x00000000BFBFA000 000090 (v03 ACRSYS ACRPRDCT 00000001 1025 00040000)
[    0.000000] ACPI: MCFG 0x00000000BFBF9000 00003C (v01 ACRSYS ACRPRDCT 00000001 1025 00040000)
[    0.000000] ACPI: ASF! 0x00000000BFBF8000 0000A5 (v32 ACRSYS ACRPRDCT 00000001 1025 00040000)
[    0.000000] ACPI: BOOT 0x00000000BFBED000 000028 (v01 ACRSYS ACRPRDCT 00000001 1025 00040000)
[    0.000000] ACPI: WDRT 0x00000000BFBEC000 000047 (v01 ACRSYS ACRPRDCT 00000000 1025 00040000)
[    0.000000] ACPI: WDAT 0x00000000BFBEB000 0001AC (v01 ACRSYS ACRPRDCT 00000001 1025 00040000)
[    0.000000] ACPI: FPDT 0x00000000BFBEA000 000044 (v01 ACRSYS ACRPRDCT 00000001 1025 00040000)
[    0.000000] ACPI: MSDM 0x00000000BFBE8000 000055 (v03 ACRSYS ACRPRDCT 00000001 1025 00040000)
[    0.000000] ACPI: SSDT 0x00000000BFBE7000 000B10 (v01 ACRSYS ACRPRDCT 00000001 1025 00040000)
[    0.000000] ACPI: SSDT 0x00000000BFBE0000 006352 (v02 ACRSYS ACRPRDCT 00000002 1025 00040000)
[    0.000000] ACPI: VFCT 0x00000000BFBDB000 004684 (v01 ACRSYS ACRPRDCT 00000001 1025 00040000)
[    0.000000] ACPI: SSDT 0x00000000BFBD9000 0013CA (v01 ACRSYS ACRPRDCT 00000001 1025 00040000)
[    0.000000] ACPI: SSDT 0x00000000BFBD8000 000D61 (v01 ACRSYS ACRPRDCT 00000001 1025 00040000)
[    0.000000] ACPI: SSDT 0x00000000BFBD6000 0010C2 (v01 ACRSYS ACRPRDCT 00000001 1025 00040000)
[    0.000000] ACPI: BGRT 0x00000000BFBD5000 000038 (v01 ACRSYS ACRPRDCT 00000001 1025 00040000)
[    0.000000] ACPI: Local APIC address 0xfee00000
[    0.000000] No NUMA configuration found
[    0.000000] Faking a node at [mem 0x0000000000000000-0x000000019effffff]
[    0.000000] NODE_DATA(0) allocated [mem 0x19eff7000-0x19effbfff]
[    0.000000]  [ffffea0000000000-ffffea00067fffff] PMD -> [ffff880198e00000-ffff88019e5fffff] on node 0
[    0.000000] Zone ranges:
[    0.000000]   DMA      [mem 0x00001000-0x00ffffff]
[    0.000000]   DMA32    [mem 0x01000000-0xffffffff]
[    0.000000]   Normal   [mem 0x100000000-0x19effffff]
[    0.000000] Movable zone start for each node
[    0.000000] Early memory node ranges
[    0.000000]   node   0: [mem 0x00001000-0x0006dfff]
[    0.000000]   node   0: [mem 0x00070000-0x00085fff]
[    0.000000]   node   0: [mem 0x00100000-0xb4b1cfff]
[    0.000000]   node   0: [mem 0xb531e000-0xb537bfff]
[    0.000000]   node   0: [mem 0xb677c000-0xbf17efff]
[    0.000000]   node   0: [mem 0xbfbff000-0xbfbfffff]
[    0.000000]   node   0: [mem 0x100000000-0x19effffff]
[    0.000000] Initmem setup node 0 [mem 0x00001000-0x19effffff]
[    0.000000] On node 0 totalpages: 1426690
[    0.000000]   DMA zone: 64 pages used for memmap
[    0.000000]   DMA zone: 22 pages reserved
[    0.000000]   DMA zone: 3971 pages, LIFO batch:0
[    0.000000]   DMA32 zone: 12054 pages used for memmap
[    0.000000]   DMA32 zone: 771455 pages, LIFO batch:31
[    0.000000]   Normal zone: 10176 pages used for memmap
[    0.000000]   Normal zone: 651264 pages, LIFO batch:31
[    0.000000] ACPI: PM-Timer IO Port: 0x408
[    0.000000] ACPI: Local APIC address 0xfee00000
[    0.000000] ACPI: LAPIC (acpi_id[0x00] lapic_id[0x00] enabled)
[    0.000000] ACPI: LAPIC (acpi_id[0x01] lapic_id[0x01] enabled)
[    0.000000] ACPI: LAPIC (acpi_id[0x02] lapic_id[0x02] enabled)
[    0.000000] ACPI: LAPIC (acpi_id[0x03] lapic_id[0x03] enabled)
[    0.000000] ACPI: LAPIC_NMI (acpi_id[0x00] high edge lint[0x1])
[    0.000000] ACPI: LAPIC_NMI (acpi_id[0x01] high edge lint[0x1])
[    0.000000] ACPI: LAPIC_NMI (acpi_id[0x02] high edge lint[0x1])
[    0.000000] ACPI: LAPIC_NMI (acpi_id[0x03] high edge lint[0x1])
[    0.000000] ACPI: IOAPIC (id[0x04] address[0xfec00000] gsi_base[0])
[    0.000000] IOAPIC[0]: apic_id 4, version 33, address 0xfec00000, GSI 0-23
[    0.000000] ACPI: IOAPIC (id[0x05] address[0xfec01000] gsi_base[24])
[    0.000000] IOAPIC[1]: apic_id 5, version 33, address 0xfec01000, GSI 24-55
[    0.000000] ACPI: INT_SRC_OVR (bus 0 bus_irq 0 global_irq 2 dfl dfl)
[    0.000000] ACPI: INT_SRC_OVR (bus 0 bus_irq 9 global_irq 9 low level)
[    0.000000] ACPI: IRQ0 used by override.
[    0.000000] ACPI: IRQ9 used by override.
[    0.000000] Using ACPI (MADT) for SMP configuration information
[    0.000000] ACPI: HPET id: 0x10228210 base: 0xfed00000
[    0.000000] smpboot: Allowing 4 CPUs, 0 hotplug CPUs
[    0.000000] PM: Registered nosave memory: [mem 0x00000000-0x00000fff]
[    0.000000] PM: Registered nosave memory: [mem 0x0006e000-0x0006ffff]
[    0.000000] PM: Registered nosave memory: [mem 0x00086000-0x000bffff]
[    0.000000] PM: Registered nosave memory: [mem 0x000c0000-0x000fffff]
[    0.000000] PM: Registered nosave memory: [mem 0xb4b1d000-0xb531dfff]
[    0.000000] PM: Registered nosave memory: [mem 0xb537c000-0xb677bfff]
[    0.000000] PM: Registered nosave memory: [mem 0xbf17f000-0xbf57efff]
[    0.000000] PM: Registered nosave memory: [mem 0xbf57f000-0xbf97efff]
[    0.000000] PM: Registered nosave memory: [mem 0xbf97f000-0xbfb7efff]
[    0.000000] PM: Registered nosave memory: [mem 0xbfb7f000-0xbfbfefff]
[    0.000000] PM: Registered nosave memory: [mem 0xbfc00000-0xdfffffff]
[    0.000000] PM: Registered nosave memory: [mem 0xe0000000-0xf7ffffff]
[    0.000000] PM: Registered nosave memory: [mem 0xf8000000-0xfbffffff]
[    0.000000] PM: Registered nosave memory: [mem 0xfc000000-0xfebfffff]
[    0.000000] PM: Registered nosave memory: [mem 0xfec00000-0xfec00fff]
[    0.000000] PM: Registered nosave memory: [mem 0xfec01000-0xfec0ffff]
[    0.000000] PM: Registered nosave memory: [mem 0xfec10000-0xfec10fff]
[    0.000000] PM: Registered nosave memory: [mem 0xfec11000-0xfed7ffff]
[    0.000000] PM: Registered nosave memory: [mem 0xfed80000-0xfed80fff]
[    0.000000] PM: Registered nosave memory: [mem 0xfed81000-0xfedfffff]
[    0.000000] PM: Registered nosave memory: [mem 0xfee00000-0xfee00fff]
[    0.000000] PM: Registered nosave memory: [mem 0xfee01000-0xffbfffff]
[    0.000000] PM: Registered nosave memory: [mem 0xffc00000-0xffffffff]
[    0.000000] e820: [mem 0xe0000000-0xf7ffffff] available for PCI devices
[    0.000000] Booting paravirtualized kernel on bare hardware
[    0.000000] setup_percpu: NR_CPUS:256 nr_cpumask_bits:256 nr_cpu_ids:4 nr_node_ids:1
[    0.000000] PERCPU: Embedded 31 pages/cpu @ffff88019ec00000 s86144 r8192 d32640 u524288
[    0.000000] pcpu-alloc: s86144 r8192 d32640 u524288 alloc=1*2097152
[    0.000000] pcpu-alloc: [0] 0 1 2 3
[    0.000000] Built 1 zonelists in Node order, mobility grouping on.  Total pages: 1404374
[    0.000000] Policy zone: Normal
[    0.000000] Kernel command line: BOOT_IMAGE=/casper/vmlinuz.efi file=/cdrom/preseed/ubuntu.seed boot=casper quiet splash --
[    0.000000] PID hash table entries: 4096 (order: 3, 32768 bytes)
[    0.000000] xsave: enabled xstate_bv 0x7, cntxt size 0x340 using standard form
[    0.000000] AGP: Checking aperture...
[    0.000000] AGP: No AGP bridge found
[    0.000000] AGP: Node 0: aperture [bus addr 0x00000000-0x01ffffff] (32MB)
[    0.000000] AGP: Your BIOS doesn't leave a aperture memory hole
[    0.000000] AGP: Please enable the IOMMU option in the BIOS setup
[    0.000000] AGP: This costs you 64MB of RAM
[    0.000000] AGP: Mapping aperture over RAM [mem 0xac000000-0xafffffff] (65536KB)
[    0.000000] PM: Registered nosave memory: [mem 0xac000000-0xafffffff]
[    0.000000] Memory: 5397444K/5706760K available (7918K kernel code, 1172K rwdata, 3752K rodata, 1408K init, 1292K bss, 309316K reserved, 0K cma-reserved)
[    0.000000] SLUB: HWalign=64, Order=0-3, MinObjects=0, CPUs=4, Nodes=1
[    0.000000] Hierarchical RCU implementation.
[    0.000000] 	RCU dyntick-idle grace-period acceleration is enabled.
[    0.000000] 	RCU restricting CPUs from NR_CPUS=256 to nr_cpu_ids=4.
[    0.000000] RCU: Adjusting geometry for rcu_fanout_leaf=16, nr_cpu_ids=4
[    0.000000] NR_IRQS:16640 nr_irqs:1000 16
[    0.000000] 	Offload RCU callbacks from all CPUs
[    0.000000] 	Offload RCU callbacks from CPUs: 0-3.
[    0.000000] Console: colour dummy device 80x25
[    0.000000] console [tty0] enabled
[    0.000000] hpet clockevent registered
[    0.000000] tsc: Fast TSC calibration using PIT
[    0.000000] tsc: Detected 998.184 MHz processor
[    0.000077] Calibrating delay loop (skipped), value calculated using timer frequency.. 1996.36 BogoMIPS (lpj=3992736)
[    0.000084] pid_max: default: 32768 minimum: 301
[    0.000101] ACPI: Core revision 20141107
[    0.033737] ACPI: All ACPI Tables successfully acquired
[    0.061431] Security Framework initialized
[    0.061492] AppArmor: AppArmor initialized
[    0.061498] Yama: becoming mindful.
[    0.063354] Dentry cache hash table entries: 1048576 (order: 11, 8388608 bytes)
[    0.070325] Inode-cache hash table entries: 524288 (order: 10, 4194304 bytes)
[    0.073551] Mount-cache hash table entries: 16384 (order: 5, 131072 bytes)
[    0.073604] Mountpoint-cache hash table entries: 16384 (order: 5, 131072 bytes)
[    0.074541] Initializing cgroup subsys memory
[    0.074564] Initializing cgroup subsys devices
[    0.074582] Initializing cgroup subsys freezer
[    0.074594] Initializing cgroup subsys net_cls
[    0.074605] Initializing cgroup subsys blkio
[    0.074615] Initializing cgroup subsys perf_event
[    0.074626] Initializing cgroup subsys net_prio
[    0.074636] Initializing cgroup subsys hugetlb
[    0.074715] CPU: Physical Processor ID: 0
[    0.074721] CPU: Processor Core ID: 0
[    0.074727] mce: CPU supports 6 MCE banks
[    0.074754] Last level iTLB entries: 4KB 512, 2MB 8, 4MB 4
[    0.074754] Last level dTLB entries: 4KB 512, 2MB 256, 4MB 128, 1GB 0
[    0.075066] Freeing SMP alternatives memory: 32K (ffffffff81e87000 - ffffffff81e8f000)
[    0.079406] Ignoring BGRT: invalid status 0 (expected 1)
[    0.084438] ftrace: allocating 29988 entries in 118 pages
[    0.115140] ..TIMER: vector=0x30 apic1=0 pin1=2 apic2=-1 pin2=-1
[    0.154845] smpboot: CPU0: AMD A6-1450 APU with Radeon(TM) HD Graphics (fam: 16, model: 00, stepping: 01)
[    0.261794] Performance Events: AMD PMU driver.
[    0.261805] ... version:                0
[    0.261809] ... bit width:              48
[    0.261811] ... generic registers:      4
[    0.261815] ... value mask:             0000ffffffffffff
[    0.261818] ... max period:             00007fffffffffff
[    0.261821] ... fixed-purpose events:   0
[    0.261824] ... event mask:             000000000000000f
[    0.263840] NMI watchdog: enabled on all CPUs, permanently consumes one hw-PMU counter.
[    0.264143] x86: Booting SMP configuration:
[    0.264148] .... node  #0, CPUs:      #1 #2 #3
[    0.304396] x86: Booted up 1 node, 4 CPUs
[    0.304409] smpboot: Total of 4 processors activated (7985.47 BogoMIPS)
[    0.306035] devtmpfs: initialized
[    0.319968] evm: security.selinux
[    0.319972] evm: security.SMACK64
[    0.319976] evm: security.SMACK64EXEC
[    0.319979] evm: security.SMACK64TRANSMUTE
[    0.319982] evm: security.SMACK64MMAP
[    0.319984] evm: security.ima
[    0.319987] evm: security.capability
[    0.320195] PM: Registering ACPI NVS region [mem 0x0006e000-0x0006ffff] (8192 bytes)
[    0.320202] PM: Registering ACPI NVS region [mem 0xbf97f000-0xbfb7efff] (2097152 bytes)
[    0.320697] pinctrl core: initialized pinctrl subsystem
[    0.320954] RTC time: 21:50:24, date: 09/06/15
[    0.321248] NET: Registered protocol family 16
[    0.329529] cpuidle: using governor ladder
[    0.333520] cpuidle: using governor menu
[    0.333760] ACPI: bus type PCI registered
[    0.333767] acpiphp: ACPI Hot Plug PCI Controller Driver version: 0.5
[    0.334010] PCI: MMCONFIG for domain 0000 [bus 00-3f] at [mem 0xf8000000-0xfbffffff] (base 0xf8000000)
[    0.334018] PCI: MMCONFIG at [mem 0xf8000000-0xfbffffff] reserved in E820
[    0.334258] PCI: Using configuration type 1 for base access
[    0.342852] ACPI: Added _OSI(Module Device)
[    0.342863] ACPI: Added _OSI(Processor Device)
[    0.342867] ACPI: Added _OSI(3.0 _SCP Extensions)
[    0.342871] ACPI: Added _OSI(Processor Aggregator Device)
[    0.350735] ACPI: Executed 1 blocks of module-level executable AML code
[    0.362562] [Firmware Bug]: ACPI: BIOS _OSI(Linux) query ignored
[    2.399625] ACPI: Interpreter enabled
[    2.399656] ACPI Exception: AE_NOT_FOUND, While evaluating Sleep State [_S1_] (20141107/hwxface-580)
[    2.399670] ACPI Exception: AE_NOT_FOUND, While evaluating Sleep State [_S2_] (20141107/hwxface-580)
[    2.399713] ACPI: (supports S0 S3 S4 S5)
[    2.399719] ACPI: Using IOAPIC for interrupt routing
[    2.400233] PCI: Using host bridge windows from ACPI; if necessary, use "pci=nocrs" and report a bug
[    2.420502] ACPI: PCI Root Bridge [PCI0] (domain 0000 [bus 00-ff])
[    2.420520] acpi PNP0A08:00: _OSC: OS supports [ExtendedConfig ASPM ClockPM Segments MSI]
[    2.420971] acpi PNP0A08:00: _OSC: platform does not support [PME]
[    2.421391] acpi PNP0A08:00: _OSC: OS now controls [PCIeHotplug AER PCIeCapability]
[    2.421803] acpi PNP0A08:00: [Firmware Info]: MMCONFIG for domain 0000 [bus 00-3f] only partially covers this bridge
[    2.422145] PCI host bridge to bus 0000:00
[    2.422155] pci_bus 0000:00: root bus resource [bus 00-ff]
[    2.422162] pci_bus 0000:00: root bus resource [io  0x0000-0x0cf7]
[    2.422168] pci_bus 0000:00: root bus resource [io  0x0d00-0xffff]
[    2.422174] pci_bus 0000:00: root bus resource [mem 0x000a0000-0x000bffff]
[    2.422180] pci_bus 0000:00: root bus resource [mem 0x000c0000-0x000c3fff]
[    2.422185] pci_bus 0000:00: root bus resource [mem 0x000c4000-0x000c7fff]
[    2.422191] pci_bus 0000:00: root bus resource [mem 0x000c8000-0x000cbfff]
[    2.422196] pci_bus 0000:00: root bus resource [mem 0x000cc000-0x000cffff]
[    2.422201] pci_bus 0000:00: root bus resource [mem 0x000d0000-0x000d3fff]
[    2.422206] pci_bus 0000:00: root bus resource [mem 0x000d4000-0x000d7fff]
[    2.422211] pci_bus 0000:00: root bus resource [mem 0x000d8000-0x000dbfff]
[    2.422217] pci_bus 0000:00: root bus resource [mem 0x000dc000-0x000dffff]
[    2.422222] pci_bus 0000:00: root bus resource [mem 0x000e0000-0x000e3fff]
[    2.422227] pci_bus 0000:00: root bus resource [mem 0x000e4000-0x000e7fff]
[    2.422232] pci_bus 0000:00: root bus resource [mem 0x000e8000-0x000ebfff]
[    2.422237] pci_bus 0000:00: root bus resource [mem 0x000ec000-0x000effff]
[    2.422242] pci_bus 0000:00: root bus resource [mem 0xe0000000-0xf7ffffff]
[    2.422247] pci_bus 0000:00: root bus resource [mem 0xfc000000-0xfed3ffff]
[    2.422252] pci_bus 0000:00: root bus resource [mem 0xfed45000-0xffffffff]
[    2.422270] pci 0000:00:00.0: [1022:1536] type 00 class 0x060000
[    2.422490] pci 0000:00:01.0: [1002:983d] type 00 class 0x030000
[    2.422517] pci 0000:00:01.0: reg 0x10: [mem 0xe0000000-0xefffffff 64bit pref]
[    2.422535] pci 0000:00:01.0: reg 0x18: [mem 0xf0000000-0xf07fffff 64bit pref]
[    2.422547] pci 0000:00:01.0: reg 0x20: [io  0x2000-0x20ff]
[    2.422561] pci 0000:00:01.0: reg 0x24: [mem 0xf0900000-0xf093ffff]
[    2.422573] pci 0000:00:01.0: reg 0x30: [mem 0xfffe0000-0xffffffff pref]
[    2.422641] pci 0000:00:01.0: supports D1 D2
[    2.422647] pci 0000:00:01.0: PME# supported from D1 D2 D3hot
[    2.422849] pci 0000:00:01.1: [1002:9840] type 00 class 0x040300
[    2.422873] pci 0000:00:01.1: reg 0x10: [mem 0xf0940000-0xf0943fff 64bit]
[    2.422971] pci 0000:00:01.1: supports D1 D2
[    2.423131] pci 0000:00:02.0: [1022:1538] type 00 class 0x060000
[    2.423333] pci 0000:00:02.3: [1022:1439] type 01 class 0x060400
[    2.423445] pci 0000:00:02.3: PME# supported from D0 D3hot D3cold
[    2.423538] pci 0000:00:02.3: System wakeup disabled by ACPI
[    2.423675] pci 0000:00:10.0: [1022:7814] type 00 class 0x0c0330
[    2.423705] pci 0000:00:10.0: reg 0x10: [mem 0xf0948000-0xf0949fff 64bit]
[    2.423832] pci 0000:00:10.0: PME# supported from D0 D3hot D3cold
[    2.423925] pci 0000:00:10.0: System wakeup disabled by ACPI
[    2.424038] pci 0000:00:11.0: [1022:7801] type 00 class 0x010601
[    2.424064] pci 0000:00:11.0: reg 0x10: [io  0x2118-0x211f]
[    2.424078] pci 0000:00:11.0: reg 0x14: [io  0x2124-0x2127]
[    2.424091] pci 0000:00:11.0: reg 0x18: [io  0x2110-0x2117]
[    2.424106] pci 0000:00:11.0: reg 0x1c: [io  0x2120-0x2123]
[    2.424125] pci 0000:00:11.0: reg 0x20: [io  0x2100-0x210f]
[    2.424138] pci 0000:00:11.0: reg 0x24: [mem 0xf094f000-0xf094f3ff]
[    2.424193] pci 0000:00:11.0: PME# supported from D3hot
[    2.424343] pci 0000:00:12.0: [1022:7807] type 00 class 0x0c0310
[    2.424368] pci 0000:00:12.0: reg 0x10: [mem 0xf094e000-0xf094efff]
[    2.424496] pci 0000:00:12.0: System wakeup disabled by ACPI
[    2.424593] pci 0000:00:12.2: [1022:7808] type 00 class 0x0c0320
[    2.424617] pci 0000:00:12.2: reg 0x10: [mem 0xf094d000-0xf094d0ff]
[    2.424709] pci 0000:00:12.2: supports D1 D2
[    2.424714] pci 0000:00:12.2: PME# supported from D0 D1 D2 D3hot D3cold
[    2.424786] pci 0000:00:12.2: System wakeup disabled by ACPI
[    2.424878] pci 0000:00:13.0: [1022:7807] type 00 class 0x0c0310
[    2.424896] pci 0000:00:13.0: reg 0x10: [mem 0xf094c000-0xf094cfff]
[    2.425026] pci 0000:00:13.0: System wakeup disabled by ACPI
[    2.425118] pci 0000:00:13.2: [1022:7808] type 00 class 0x0c0320
[    2.425141] pci 0000:00:13.2: reg 0x10: [mem 0xf094b000-0xf094b0ff]
[    2.425234] pci 0000:00:13.2: supports D1 D2
[    2.425238] pci 0000:00:13.2: PME# supported from D0 D1 D2 D3hot D3cold
[    2.425310] pci 0000:00:13.2: System wakeup disabled by ACPI
[    2.425402] pci 0000:00:14.0: [1022:780b] type 00 class 0x0c0500
[    2.425606] pci 0000:00:14.2: [1022:780d] type 00 class 0x040300
[    2.425634] pci 0000:00:14.2: reg 0x10: [mem 0xf0944000-0xf0947fff 64bit]
[    2.425721] pci 0000:00:14.2: PME# supported from D0 D3hot D3cold
[    2.425868] pci 0000:00:14.3: [1022:780e] type 00 class 0x060100
[    2.426078] pci 0000:00:14.7: [1022:7813] type 00 class 0x080501
[    2.426101] pci 0000:00:14.7: reg 0x10: [mem 0xf094a000-0xf094a0ff 64bit]
[    2.426280] pci 0000:00:18.0: [1022:1530] type 00 class 0x060000
[    2.426434] pci 0000:00:18.1: [1022:1531] type 00 class 0x060000
[    2.426582] pci 0000:00:18.2: [1022:1532] type 00 class 0x060000
[    2.426732] pci 0000:00:18.3: [1022:1533] type 00 class 0x060000
[    2.426889] pci 0000:00:18.4: [1022:1534] type 00 class 0x060000
[    2.427037] pci 0000:00:18.5: [1022:1535] type 00 class 0x060000
[    2.427324] pci 0000:01:00.0: [168c:0036] type 00 class 0x028000
[    2.427355] pci 0000:01:00.0: reg 0x10: [mem 0xf0800000-0xf087ffff 64bit]
[    2.427413] pci 0000:01:00.0: reg 0x30: [mem 0xffff0000-0xffffffff pref]
[    2.427499] pci 0000:01:00.0: supports D1 D2
[    2.427504] pci 0000:01:00.0: PME# supported from D0 D1 D2 D3hot D3cold
[    2.436551] pci 0000:00:02.3: PCI bridge to [bus 01]
[    2.436575] pci 0000:00:02.3:   bridge window [mem 0xf0800000-0xf08fffff]
[    2.438876] ACPI: PCI Interrupt Link [LNKA] (IRQs 3 4 5 7 10 11 12 14 15) *0
[    2.439064] ACPI: PCI Interrupt Link [LNKB] (IRQs 3 4 5 7 10 11 12 14 15) *0
[    2.439253] ACPI: PCI Interrupt Link [LNKC] (IRQs 3 4 5 7 10 11 12 14 15) *0
[    2.439440] ACPI: PCI Interrupt Link [LNKD] (IRQs 3 4 5 7 10 11 12 14 15) *0
[    2.439601] ACPI: PCI Interrupt Link [LNKE] (IRQs 3 4 5 7 10 11 12 14 15) *0
[    2.439722] ACPI: PCI Interrupt Link [LNKF] (IRQs 3 4 5 7 10 11 12 14 15) *0
[    2.439842] ACPI: PCI Interrupt Link [LNKG] (IRQs 3 4 5 7 10 11 12 14 15) *0
[    2.439962] ACPI: PCI Interrupt Link [LNKH] (IRQs 3 4 5 7 10 11 12 14 15) *0
[    2.440553] ACPI : EC: GPE = 0x3, I/O: command/status = 0x66, data = 0x62
[    2.440954] vgaarb: setting as boot device: PCI:0000:00:01.0
[    2.440961] vgaarb: device added: PCI:0000:00:01.0,decodes=io+mem,owns=io+mem,locks=none
[    2.440971] vgaarb: loaded
[    2.440975] vgaarb: bridge control possible 0000:00:01.0
[    2.441661] SCSI subsystem initialized
[    2.441870] libata version 3.00 loaded.
[    2.441949] ACPI: bus type USB registered
[    2.442006] usbcore: registered new interface driver usbfs
[    2.442033] usbcore: registered new interface driver hub
[    2.442081] usbcore: registered new device driver usb
[    2.442603] PCI: Using ACPI for IRQ routing
[    2.444780] PCI: pci_cache_line_size set to 64 bytes
[    2.444876] e820: reserve RAM buffer [mem 0x0006e000-0x0006ffff]
[    2.444883] e820: reserve RAM buffer [mem 0x00086000-0x0008ffff]
[    2.444887] e820: reserve RAM buffer [mem 0xb4b1d000-0xb7ffffff]
[    2.444892] e820: reserve RAM buffer [mem 0xb537c000-0xb7ffffff]
[    2.444897] e820: reserve RAM buffer [mem 0xbf17f000-0xbfffffff]
[    2.444902] e820: reserve RAM buffer [mem 0xbfc00000-0xbfffffff]
[    2.444906] e820: reserve RAM buffer [mem 0x19f000000-0x19fffffff]
[    2.445275] NetLabel: Initializing
[    2.445281] NetLabel:  domain hash size = 128
[    2.445284] NetLabel:  protocols = UNLABELED CIPSOv4
[    2.445322] NetLabel:  unlabeled traffic allowed by default
[    2.445557] hpet0: at MMIO 0xfed00000, IRQs 2, 8, 0
[    2.445568] hpet0: 3 comparators, 32-bit 14.318180 MHz counter
[    2.447745] Switched to clocksource hpet
[    2.465272] AppArmor: AppArmor Filesystem Enabled
[    2.465478] pnp: PnP ACPI init
[    2.465834] system 00:00: [mem 0xfec00000-0xfec01fff] could not be reserved
[    2.465841] system 00:00: [mem 0xfee00000-0xfee00fff] has been reserved
[    2.465851] system 00:00: Plug and Play ACPI device, IDs PNP0c02 (active)
[    2.466140] pnp 00:01: Plug and Play ACPI device, IDs PNP0b00 (active)
[    2.466256] system 00:02: [io  0x0400-0x04cf] could not be reserved
[    2.466263] system 00:02: [io  0x04d0-0x04d1] has been reserved
[    2.466269] system 00:02: [io  0x04d6] has been reserved
[    2.466274] system 00:02: [io  0x0680-0x06ff] has been reserved
[    2.466279] system 00:02: [io  0x077a] has been reserved
[    2.466284] system 00:02: [io  0x0c00-0x0c01] has been reserved
[    2.466289] system 00:02: [io  0x0c14] has been reserved
[    2.466294] system 00:02: [io  0x0c50-0x0c52] has been reserved
[    2.466299] system 00:02: [io  0x0c6c] has been reserved
[    2.466304] system 00:02: [io  0x0c6f] has been reserved
[    2.466309] system 00:02: [io  0x0cd0-0x0cdb] has been reserved
[    2.466314] system 00:02: [io  0x0840-0x0847] has been reserved
[    2.466321] system 00:02: Plug and Play ACPI device, IDs PNP0c02 (active)
[    2.466486] system 00:03: [mem 0x000e0000-0x000fffff] could not be reserved
[    2.466492] system 00:03: [mem 0xffc00000-0xffffffff] has been reserved
[    2.466498] system 00:03: Plug and Play ACPI device, IDs PNP0c01 (active)
[    2.466725] pnp 00:04: Plug and Play ACPI device, IDs PNP0303 (active)
[    2.466832] pnp 00:05: Plug and Play ACPI device, IDs ETD050a PNP0f13 (active)
[    2.467793] pnp: PnP ACPI: found 6 devices
[    2.477526] pci 0000:00:01.0: can't claim BAR 6 [mem 0xfffe0000-0xffffffff pref]: address conflict with reserved [mem 0xffc00000-0xffffffff]
[    2.477535] pci 0000:01:00.0: can't claim BAR 6 [mem 0xffff0000-0xffffffff pref]: no compatible bridge window
[    2.477583] pci 0000:00:01.0: BAR 6: assigned [mem 0xf0960000-0xf097ffff pref]
[    2.477594] pci 0000:01:00.0: BAR 6: assigned [mem 0xf0880000-0xf088ffff pref]
[    2.477601] pci 0000:00:02.3: PCI bridge to [bus 01]
[    2.477611] pci 0000:00:02.3:   bridge window [mem 0xf0800000-0xf08fffff]
[    2.477626] pci_bus 0000:00: resource 4 [io  0x0000-0x0cf7]
[    2.477631] pci_bus 0000:00: resource 5 [io  0x0d00-0xffff]
[    2.477636] pci_bus 0000:00: resource 6 [mem 0x000a0000-0x000bffff]
[    2.477642] pci_bus 0000:00: resource 7 [mem 0x000c0000-0x000c3fff]
[    2.477647] pci_bus 0000:00: resource 8 [mem 0x000c4000-0x000c7fff]
[    2.477652] pci_bus 0000:00: resource 9 [mem 0x000c8000-0x000cbfff]
[    2.477657] pci_bus 0000:00: resource 10 [mem 0x000cc000-0x000cffff]
[    2.477662] pci_bus 0000:00: resource 11 [mem 0x000d0000-0x000d3fff]
[    2.477667] pci_bus 0000:00: resource 12 [mem 0x000d4000-0x000d7fff]
[    2.477672] pci_bus 0000:00: resource 13 [mem 0x000d8000-0x000dbfff]
[    2.477677] pci_bus 0000:00: resource 14 [mem 0x000dc000-0x000dffff]
[    2.477681] pci_bus 0000:00: resource 15 [mem 0x000e0000-0x000e3fff]
[    2.477686] pci_bus 0000:00: resource 16 [mem 0x000e4000-0x000e7fff]
[    2.477691] pci_bus 0000:00: resource 17 [mem 0x000e8000-0x000ebfff]
[    2.477696] pci_bus 0000:00: resource 18 [mem 0x000ec000-0x000effff]
[    2.477701] pci_bus 0000:00: resource 19 [mem 0xe0000000-0xf7ffffff]
[    2.477706] pci_bus 0000:00: resource 20 [mem 0xfc000000-0xfed3ffff]
[    2.477710] pci_bus 0000:00: resource 21 [mem 0xfed45000-0xffffffff]
[    2.477716] pci_bus 0000:01: resource 1 [mem 0xf0800000-0xf08fffff]
[    2.477787] NET: Registered protocol family 2
[    2.478310] TCP established hash table entries: 65536 (order: 7, 524288 bytes)
[    2.478758] TCP bind hash table entries: 65536 (order: 8, 1048576 bytes)
[    2.479173] TCP: Hash tables configured (established 65536 bind 65536)
[    2.479247] TCP: reno registered
[    2.479278] UDP hash table entries: 4096 (order: 5, 131072 bytes)
[    2.479367] UDP-Lite hash table entries: 4096 (order: 5, 131072 bytes)
[    2.479589] NET: Registered protocol family 1
[    2.479639] pci 0000:00:01.0: Video device with shadowed ROM
[    2.624489] PCI: CLS 64 bytes, default 64
[    2.624659] Trying to unpack rootfs image as initramfs...
[   13.263239] Freeing initrd memory: 21192K (ffff88003568c000 - ffff880036b3e000)
[   13.263349] PCI-DMA: Using software bounce buffering for IO (SWIOTLB)
[   13.263359] software IO TLB [mem 0xb7f63000-0xbbf63000] (64MB) mapped at [ffff8800b7f63000-ffff8800bbf62fff]
[   13.263473] Simple Boot Flag at 0x44 set to 0x1
[   13.263909] perf: AMD NB counters detected
[   13.263915] perf: AMD L2I counters detected
[   13.264138] microcode: CPU0: patch_level=0x07000106
[   13.264156] microcode: CPU1: patch_level=0x07000106
[   13.264174] microcode: CPU2: patch_level=0x07000106
[   13.264187] microcode: CPU3: patch_level=0x07000106
[   13.264382] microcode: Microcode Update Driver: v2.00 <tigran@aivazian.fsnet.co.uk>, Peter Oruba
[   13.264395] LVT offset 0 assigned for vector 0x400
[   13.264444] perf: AMD IBS detected (0x000000ff)
[   13.264507] Scanning for low memory corruption every 60 seconds
[   13.265465] futex hash table entries: 1024 (order: 4, 65536 bytes)
[   13.265511] Initialise system trusted keyring
[   13.265565] audit: initializing netlink subsys (disabled)
[   13.265606] audit: type=2000 audit(1441576235.128:1): initialized
[   13.266533] HugeTLB registered 2 MB page size, pre-allocated 0 pages
[   13.270946] zpool: loaded
[   13.270954] zbud: loaded
[   13.271380] VFS: Disk quotas dquot_6.5.2
[   13.271478] VFS: Dquot-cache hash table entries: 512 (order 0, 4096 bytes)
[   13.272836] fuse init (API version 7.23)
[   13.273220] Key type big_key registered
[   13.274347] Key type asymmetric registered
[   13.274358] Asymmetric key parser 'x509' registered
[   13.274483] Block layer SCSI generic (bsg) driver version 0.4 loaded (major 252)
[   13.274608] io scheduler noop registered
[   13.274620] io scheduler deadline registered (default)
[   13.274731] io scheduler cfq registered
[   13.275354] pci_hotplug: PCI Hot Plug PCI Core version: 0.5
[   13.275407] pciehp: PCI Express Hot Plug Controller Driver version: 0.4
[   13.275530] efifb: probing for efifb
[   13.275581] efifb: framebuffer at 0xe0000000, mapped to 0xffffc90004f00000, using 4224k, total 4224k
[   13.275587] efifb: mode is 1366x768x32, linelength=5632, pages=1
[   13.275590] efifb: scrolling: redraw
[   13.275596] efifb: Truecolor: size=8:8:8:8, shift=24:16:8:0
[   13.285610] Console: switching to colour frame buffer device 170x48
[   13.293962] fb0: EFI VGA frame buffer device
[   13.294707] ACPI: AC Adapter [ADP1] (on-line)
[   13.295355] input: Power Button as /devices/LNXSYSTM:00/LNXSYBUS:00/PNP0C0C:00/input/input0
[   13.295364] ACPI: Power Button [PWRB]
[   13.295482] input: Lid Switch as /devices/LNXSYSTM:00/LNXSYBUS:00/PNP0C0D:00/input/input1
[   13.295529] ACPI: Lid Switch [LID0]
[   13.295625] input: Sleep Button as /devices/LNXSYSTM:00/LNXSYBUS:00/PNP0C0E:00/input/input2
[   13.295631] ACPI: Sleep Button [SLPB]
[   13.295725] input: Power Button as /devices/LNXSYSTM:00/LNXPWRBN:00/input/input3
[   13.295731] ACPI: Power Button [PWRF]
[   13.295832] ACPI: acpi_idle registered with cpuidle
[   13.301084] thermal LNXTHERM:00: registered as thermal_zone0
[   13.301093] ACPI: Thermal Zone [TZS0] (67 C)
[   13.301583] thermal LNXTHERM:01: registered as thermal_zone1
[   13.301587] ACPI: Thermal Zone [TZS1] (44 C)
[   13.301693] GHES: HEST is not enabled!
[   13.301990] Serial: 8250/16550 driver, 32 ports, IRQ sharing enabled
[   13.302911] ACPI: Battery Slot [BAT0] (battery present)
[   13.306383] Linux agpgart interface v0.103
[   13.310338] brd: module loaded
[   13.312283] loop: module loaded
[   13.312904] libphy: Fixed MDIO Bus: probed
[   13.312914] tun: Universal TUN/TAP device driver, 1.6
[   13.312918] tun: (C) 1999-2004 Max Krasnyansky <maxk@qualcomm.com>
[   13.313049] PPP generic driver version 2.4.2
[   13.313505] xhci_hcd 0000:00:10.0: xHCI Host Controller
[   13.313523] xhci_hcd 0000:00:10.0: new USB bus registered, assigned bus number 1
[   13.314095] usb usb1: New USB device found, idVendor=1d6b, idProduct=0002
[   13.314104] usb usb1: New USB device strings: Mfr=3, Product=2, SerialNumber=1
[   13.314110] usb usb1: Product: xHCI Host Controller
[   13.314116] usb usb1: Manufacturer: Linux 3.19.0-25-generic xhci-hcd
[   13.314122] usb usb1: SerialNumber: 0000:00:10.0
[   13.314516] hub 1-0:1.0: USB hub found
[   13.314545] hub 1-0:1.0: 2 ports detected
[   13.314826] xhci_hcd 0000:00:10.0: xHCI Host Controller
[   13.314837] xhci_hcd 0000:00:10.0: new USB bus registered, assigned bus number 2
[   13.317208] usb usb2: New USB device found, idVendor=1d6b, idProduct=0003
[   13.317215] usb usb2: New USB device strings: Mfr=3, Product=2, SerialNumber=1
[   13.317221] usb usb2: Product: xHCI Host Controller
[   13.317227] usb usb2: Manufacturer: Linux 3.19.0-25-generic xhci-hcd
[   13.317233] usb usb2: SerialNumber: 0000:00:10.0
[   13.317553] hub 2-0:1.0: USB hub found
[   13.317580] hub 2-0:1.0: 2 ports detected
[   13.317873] ehci_hcd: USB 2.0 'Enhanced' Host Controller (EHCI) Driver
[   13.317892] ehci-pci: EHCI PCI platform driver
[   13.318118] ehci-pci 0000:00:12.2: EHCI Host Controller
[   13.318135] ehci-pci 0000:00:12.2: new USB bus registered, assigned bus number 3
[   13.318147] ehci-pci 0000:00:12.2: applying AMD SB700/SB800/Hudson-2/3 EHCI dummy qh workaround
[   13.318164] ehci-pci 0000:00:12.2: debug port 1
[   13.318240] ehci-pci 0000:00:12.2: irq 17, io mem 0xf094d000
[   13.328803] ehci-pci 0000:00:12.2: USB 2.0 started, EHCI 1.00
[   13.329015] usb usb3: New USB device found, idVendor=1d6b, idProduct=0002
[   13.329024] usb usb3: New USB device strings: Mfr=3, Product=2, SerialNumber=1
[   13.329032] usb usb3: Product: EHCI Host Controller
[   13.329039] usb usb3: Manufacturer: Linux 3.19.0-25-generic ehci_hcd
[   13.329046] usb usb3: SerialNumber: 0000:00:12.2
[   13.329516] hub 3-0:1.0: USB hub found
[   13.329546] hub 3-0:1.0: 4 ports detected
[   13.330280] ehci-pci 0000:00:13.2: EHCI Host Controller
[   13.330302] ehci-pci 0000:00:13.2: new USB bus registered, assigned bus number 4
[   13.330315] ehci-pci 0000:00:13.2: applying AMD SB700/SB800/Hudson-2/3 EHCI dummy qh workaround
[   13.330334] ehci-pci 0000:00:13.2: debug port 1
[   13.330405] ehci-pci 0000:00:13.2: irq 17, io mem 0xf094b000
[   13.340800] ehci-pci 0000:00:13.2: USB 2.0 started, EHCI 1.00
[   13.341040] usb usb4: New USB device found, idVendor=1d6b, idProduct=0002
[   13.341049] usb usb4: New USB device strings: Mfr=3, Product=2, SerialNumber=1
[   13.341056] usb usb4: Product: EHCI Host Controller
[   13.341062] usb usb4: Manufacturer: Linux 3.19.0-25-generic ehci_hcd
[   13.341068] usb usb4: SerialNumber: 0000:00:13.2
[   13.341596] hub 4-0:1.0: USB hub found
[   13.341623] hub 4-0:1.0: 4 ports detected
[   13.342130] ehci-platform: EHCI generic platform driver
[   13.342183] ohci_hcd: USB 1.1 'Open' Host Controller (OHCI) Driver
[   13.342209] ohci-pci: OHCI PCI platform driver
[   13.342488] ohci-pci 0000:00:12.0: OHCI PCI host controller
[   13.342513] ohci-pci 0000:00:12.0: new USB bus registered, assigned bus number 5
[   13.342584] ohci-pci 0000:00:12.0: irq 18, io mem 0xf094e000
[   13.400902] usb usb5: New USB device found, idVendor=1d6b, idProduct=0001
[   13.400913] usb usb5: New USB device strings: Mfr=3, Product=2, SerialNumber=1
[   13.400918] usb usb5: Product: OHCI PCI host controller
[   13.400922] usb usb5: Manufacturer: Linux 3.19.0-25-generic ohci_hcd
[   13.400927] usb usb5: SerialNumber: 0000:00:12.0
[   13.401356] hub 5-0:1.0: USB hub found
[   13.401378] hub 5-0:1.0: 4 ports detected
[   13.401947] ohci-pci 0000:00:13.0: OHCI PCI host controller
[   13.401969] ohci-pci 0000:00:13.0: new USB bus registered, assigned bus number 6
[   13.402022] ohci-pci 0000:00:13.0: irq 18, io mem 0xf094c000
[   13.460905] usb usb6: New USB device found, idVendor=1d6b, idProduct=0001
[   13.460915] usb usb6: New USB device strings: Mfr=3, Product=2, SerialNumber=1
[   13.460921] usb usb6: Product: OHCI PCI host controller
[   13.460925] usb usb6: Manufacturer: Linux 3.19.0-25-generic ohci_hcd
[   13.460930] usb usb6: SerialNumber: 0000:00:13.0
[   13.461388] hub 6-0:1.0: USB hub found
[   13.461410] hub 6-0:1.0: 4 ports detected
[   13.461855] ohci-platform: OHCI generic platform driver
[   13.461902] uhci_hcd: USB Universal Host Controller Interface driver
[   13.462081] i8042: PNP: PS/2 Controller [PNP0303:KBD0,PNP0f13:PS2M] at 0x60,0x64 irq 1,12
[   13.469407] serio: i8042 KBD port at 0x60,0x64 irq 1
[   13.469421] serio: i8042 AUX port at 0x60,0x64 irq 12
[   13.469807] mousedev: PS/2 mouse device common for all mice
[   13.470360] rtc_cmos 00:01: RTC can wake from S4
[   13.470560] rtc_cmos 00:01: rtc core: registered rtc_cmos as rtc0
[   13.470601] rtc_cmos 00:01: alarms up to one month, 114 bytes nvram, hpet irqs
[   13.470646] i2c /dev entries driver
[   13.470842] device-mapper: uevent: version 1.0.3
[   13.471033] device-mapper: ioctl: 4.29.0-ioctl (2014-10-28) initialised: dm-devel@redhat.com
[   13.471077] ledtrig-cpu: registered to indicate activity on CPUs
[   13.471099] EFI Variables Facility v0.08 2004-May-17
[   13.483200] input: AT Translated Set 2 keyboard as /devices/platform/i8042/serio0/input/input4
[   13.522880] PCCT header not found.
[   13.522889] ACPI PCC probe failed.
[   13.523193] TCP: cubic registered
[   13.523475] NET: Registered protocol family 10
[   13.524043] NET: Registered protocol family 17
[   13.524072] Key type dns_resolver registered
[   13.525058] Loading compiled-in X.509 certificates
[   13.527858] Loaded X.509 cert 'Magrathea: Glacier signing key: 6aaa11d18c2d3a40b1b4dbe5bf8ad656ddf51838'
[   13.527902] registered taskstats version 1
[   13.533091] Key type trusted registered
[   13.543360] Key type encrypted registered
[   13.543381] AppArmor: AppArmor sha1 policy hashing enabled
[   13.543395] ima: No TPM chip found, activating TPM-bypass!
[   13.543462] evm: HMAC attrs: 0x1
[   13.544342]   Magic number: 15:484:853
[   13.544640] rtc_cmos 00:01: setting system clock to 2015-09-06 21:50:37 UTC (1441576237)
[   13.545049] acpi-cpufreq: overriding BIOS provided _PSD data
[   13.545519] BIOS EDD facility v0.16 2004-Jun-25, 0 devices found
[   13.545524] EDD information not available.
[   13.545756] PM: Hibernation image not present or could not be loaded.
[   13.546669] Freeing unused kernel memory: 1408K (ffffffff81d27000 - ffffffff81e87000)
[   13.546676] Write protecting the kernel read-only data: 12288k
[   13.547293] Freeing unused kernel memory: 260K (ffff8800027bf000 - ffff880002800000)
[   13.547579] Freeing unused kernel memory: 344K (ffff880002baa000 - ffff880002c00000)
[   13.579517] systemd-udevd[122]: starting version 204
[   13.610202] ACPI: Video Device [VGA] (multi-head: yes  rom: no  post: no)
[   13.631900] acpi device:00: registered as cooling_device4
[   13.632112] input: Video Bus as /devices/LNXSYSTM:00/LNXSYBUS:00/PNP0A08:00/LNXVIDEO:00/input/input7
[   13.634421] wmi: Mapper loaded
[   13.654005] ahci 0000:00:11.0: version 3.0
[   13.654483] ahci 0000:00:11.0: AHCI 0001.0300 32 slots 1 ports 6 Gbps 0x1 impl SATA mode
[   13.654500] ahci 0000:00:11.0: flags: 64bit ncq sntf ilck pm led clo pmp fbs pio slum part
[   13.655366] usb 4-1: new high-speed USB device number 2 using ehci-pci
[   13.657136] sdhci: Secure Digital Host Controller Interface driver
[   13.657145] sdhci: Copyright(c) Pierre Ossman
[   13.658104] usb 3-1: new high-speed USB device number 2 using ehci-pci
[   13.660271] scsi host0: ahci
[   13.660698] ata1: SATA max UDMA/133 abar m1024@0xf094f000 port 0xf094f100 irq 19
[   13.666129] [drm] Initialized drm 1.1.0 20060810
[   13.666316] sdhci-pci 0000:00:14.7: SDHCI controller found [1022:7813] (rev 1)
[   13.666791] sdhci-pci 0000:00:14.7: No vmmc regulator found
[   13.666802] sdhci-pci 0000:00:14.7: No vqmmc regulator found
[   13.669378] mmc0: SDHCI controller on PCI [0000:00:14.7] using ADMA
[   13.773830] [drm] radeon kernel modesetting enabled.
[   13.781906] AMD IOMMUv2 driver by Joerg Roedel <joerg.roedel@amd.com>
[   13.781917] AMD IOMMUv2 functionality not available on this system
[   13.789598] CRAT table not found
[   13.789607] Finished initializing topology ret=0
[   13.789766] kfd kfd: Initialized module
[   13.790266] checking generic (e0000000 420000) vs hw (e0000000 10000000)
[   13.790272] fb: switching to radeondrmfb from EFI VGA
[   13.790327] Console: switching to colour dummy device 80x25
[   13.791283] [drm] initializing kernel modesetting (KABINI 0x1002:0x983D 0x1025:0x080D).
[   13.791312] [drm] register mmio base: 0xF0900000
[   13.791316] [drm] register mmio size: 262144
[   13.791329] [drm] doorbell mmio base: 0xF0000000
[   13.791332] [drm] doorbell mmio size: 8388608
[   13.791344] [drm] ACPI VFCT contains a BIOS for 00:01.0 1002:983d, size 17920
[   13.791367] ATOM BIOS: AMD
[   13.791463] radeon 0000:00:01.0: VRAM: 512M 0x0000000000000000 - 0x000000001FFFFFFF (512M used)
[   13.791470] radeon 0000:00:01.0: GTT: 1024M 0x0000000020000000 - 0x000000005FFFFFFF
[   13.791475] [drm] Detected VRAM RAM=512M, BAR=256M
[   13.791478] [drm] RAM width 128bits DDR
[   13.791612] [TTM] Zone  kernel: Available graphics memory: 2735000 kiB
[   13.791617] [TTM] Zone   dma32: Available graphics memory: 2097152 kiB
[   13.791620] [TTM] Initializing pool allocator
[   13.791634] [TTM] Initializing DMA pool allocator
[   13.791684] [drm] radeon: 512M of VRAM memory ready
[   13.791689] [drm] radeon: 1024M of GTT memory ready.
[   13.791733] [drm] Loading kabini Microcode
[   13.792008] [drm] Internal thermal controller without fan control
[   13.793248] [drm] radeon: dpm initialized
[   13.797611] [drm] Found VCE firmware/feedback version 40.2.2 / 15!
[   13.797636] [drm] GART: num cpu pages 262144, num gpu pages 262144
[   13.820590] [drm] PCIE GART of 1024M enabled (table at 0x000000000078C000).
[   13.820931] radeon 0000:00:01.0: WB enabled
[   13.820966] radeon 0000:00:01.0: fence driver on ring 0 use gpu addr 0x0000000020000c00 and cpu addr 0xffff880194bdfc00
[   13.820975] radeon 0000:00:01.0: fence driver on ring 1 use gpu addr 0x0000000020000c04 and cpu addr 0xffff880194bdfc04
[   13.820982] radeon 0000:00:01.0: fence driver on ring 2 use gpu addr 0x0000000020000c08 and cpu addr 0xffff880194bdfc08
[   13.820989] radeon 0000:00:01.0: fence driver on ring 3 use gpu addr 0x0000000020000c0c and cpu addr 0xffff880194bdfc0c
[   13.820996] radeon 0000:00:01.0: fence driver on ring 4 use gpu addr 0x0000000020000c10 and cpu addr 0xffff880194bdfc10
[   13.821931] radeon 0000:00:01.0: fence driver on ring 5 use gpu addr 0x0000000000076c98 and cpu addr 0xffffc90005436c98
[   13.822059] radeon 0000:00:01.0: fence driver on ring 6 use gpu addr 0x0000000020000c18 and cpu addr 0xffff880194bdfc18
[   13.822067] radeon 0000:00:01.0: fence driver on ring 7 use gpu addr 0x0000000020000c1c and cpu addr 0xffff880194bdfc1c
[   13.822074] [drm] Supports vblank timestamp caching Rev 2 (21.10.2013).
[   13.822078] [drm] Driver supports precise vblank timestamp query.
[   13.822154] radeon 0000:00:01.0: radeon: using MSI.
[   13.822216] [drm] radeon: irq initialized.
[   13.827833] [drm] ring test on 0 succeeded in 3 usecs
[   13.827952] [drm] ring test on 1 succeeded in 3 usecs
[   13.827986] [drm] ring test on 2 succeeded in 3 usecs
[   13.828250] [drm] ring test on 3 succeeded in 4 usecs
[   13.828263] [drm] ring test on 4 succeeded in 4 usecs
[   13.841628] usb 3-1: New USB device found, idVendor=8644, idProduct=800b
[   13.841637] usb 3-1: New USB device strings: Mfr=1, Product=2, SerialNumber=3
[   13.841644] usb 3-1: Product: USB Flash Disk
[   13.841651] usb 3-1: Manufacturer: General
[   13.841656] usb 3-1: SerialNumber: 0000000000001379
[   13.853723] usb-storage 3-1:1.0: USB Mass Storage device detected
[   13.854057] scsi host1: usb-storage 3-1:1.0
[   13.854300] usbcore: registered new interface driver usb-storage
[   13.859025] usbcore: registered new interface driver uas
[   13.874028] [drm] ring test on 5 succeeded in 2 usecs
[   13.893109] usb 4-1: New USB device found, idVendor=064e, idProduct=e330
[   13.893118] usb 4-1: New USB device strings: Mfr=3, Product=1, SerialNumber=0
[   13.893125] usb 4-1: Product: HD WebCam
[   13.893131] usb 4-1: Manufacturer: SuYin
[   13.893920] [drm] UVD initialized successfully.
[   14.003250] [drm] ring test on 6 succeeded in 17 usecs
[   14.003267] [drm] ring test on 7 succeeded in 3 usecs
[   14.003271] [drm] VCE initialized successfully.
[   14.008474] [drm] ib test on ring 0 succeeded in 0 usecs
[   14.152750] ata1: SATA link up 6.0 Gbps (SStatus 133 SControl 300)
[   14.153404] ata1.00: ATA-9: WDC WD5000LPVX-22V0TT0, 01.01A01, max UDMA/133
[   14.153413] ata1.00: 976773168 sectors, multi 0: LBA48 NCQ (depth 31/32), AA
[   14.154064] ata1.00: configured for UDMA/133
[   14.154484] scsi 0:0:0:0: Direct-Access     ATA      WDC WD5000LPVX-2 1A01 PQ: 0 ANSI: 5
[   14.155243] sd 0:0:0:0: [sda] 976773168 512-byte logical blocks: (500 GB/465 GiB)
[   14.155251] sd 0:0:0:0: [sda] 4096-byte physical blocks
[   14.155298] sd 0:0:0:0: Attached scsi generic sg0 type 0
[   14.155452] sd 0:0:0:0: [sda] Write Protect is off
[   14.155462] sd 0:0:0:0: [sda] Mode Sense: 00 3a 00 00
[   14.155559] sd 0:0:0:0: [sda] Write cache: enabled, read cache: enabled, doesn't support DPO or FUA
[   14.260727] tsc: Refined TSC clocksource calibration: 998.128 MHz
[   14.268811] usb 6-2: new full-speed USB device number 2 using ohci-pci
[   14.284618] psmouse serio1: elantech: assuming hardware version 4 (with firmware version 0x461f00)
[   14.301813] psmouse serio1: elantech: Synaptics capabilities query result 0x11, 0x16, 0x0c.
[   14.388909] input: ETPS/2 Elantech Touchpad as /devices/platform/i8042/serio1/input/input8
[   14.429820] usb 6-2: New USB device found, idVendor=0489, idProduct=e05f
[   14.429833] usb 6-2: New USB device strings: Mfr=0, Product=0, SerialNumber=0
[   14.504984] [drm] ib test on ring 1 succeeded in 0 usecs
[   14.696946] usb 6-3: new full-speed USB device number 3 using ohci-pci
[   14.853764] scsi 1:0:0:0: Direct-Access     General  USB Flash Disk   1.0  PQ: 0 ANSI: 2
[   14.854532] sd 1:0:0:0: Attached scsi generic sg1 type 0
[   14.855478] sd 1:0:0:0: [sdb] 3915776 512-byte logical blocks: (2.00 GB/1.86 GiB)
[   14.856480] sd 1:0:0:0: [sdb] Write Protect is off
[   14.856490] sd 1:0:0:0: [sdb] Mode Sense: 03 00 00 00
[   14.857611] sd 1:0:0:0: [sdb] No Caching mode page found
[   14.857625] sd 1:0:0:0: [sdb] Assuming drive cache: write through
[   14.863653]  sdb: sdb1
[   14.867847] sd 1:0:0:0: [sdb] Attached SCSI removable disk
[   14.869786] usb 6-3: New USB device found, idVendor=04f3, idProduct=009e
[   14.869812] usb 6-3: New USB device strings: Mfr=4, Product=14, SerialNumber=0
[   14.869819] usb 6-3: Product: Touchscreen
[   14.869825] usb 6-3: Manufacturer: ELAN
[   14.882450] hidraw: raw HID events driver (C) Jiri Kosina
[   14.900209] usbcore: registered new interface driver usbhid
[   14.900216] usbhid: USB HID core driver
[   15.004821] [drm] ib test on ring 2 succeeded in 0 usecs
[   15.004941] [drm] ib test on ring 3 succeeded in 0 usecs
[   15.005025] [drm] ib test on ring 4 succeeded in 0 usecs
[   15.261277] Switched to clocksource tsc
[   15.525046] [drm] ib test on ring 5 succeeded
[   15.545820] [drm] ib test on ring 6 succeeded
[   15.546611] [drm] ib test on ring 7 succeeded
[   15.554074] [drm] radeon atom DIG backlight initialized
[   15.554082] [drm] Radeon Display Connectors
[   15.554086] [drm] Connector 0:
[   15.554090] [drm]   eDP-1
[   15.554094] [drm]   HPD1
[   15.554100] [drm]   DDC: 0x6530 0x6530 0x6534 0x6534 0x6538 0x6538 0x653c 0x653c
[   15.554103] [drm]   Encoders:
[   15.554107] [drm]     LCD1: INTERNAL_UNIPHY
[   15.554110] [drm] Connector 1:
[   15.554113] [drm]   DP-1
[   15.554116] [drm]   HPD2
[   15.554121] [drm]   DDC: 0x6540 0x6540 0x6544 0x6544 0x6548 0x6548 0x654c 0x654c
[   15.554124] [drm]   Encoders:
[   15.554127] [drm]     DFP1: INTERNAL_UNIPHY
[   15.554130] [drm] Connector 2:
[   15.554133] [drm]   VGA-1
[   15.554138] [drm]   DDC: 0x65c0 0x65c0 0x65c4 0x65c4 0x65c8 0x65c8 0x65cc 0x65cc
[   15.554141] [drm]   Encoders:
[   15.554144] [drm]     CRT1: INTERNAL_KLDSCP_DAC1
[   15.934342]  sda: sda1 sda2 sda3 sda4 sda5 sda6 sda7
[   15.935749] sd 0:0:0:0: [sda] Attached SCSI disk
[   16.291133] [drm] fb mappable at 0xE0990000
[   16.291142] [drm] vram apper at 0xE0000000
[   16.291145] [drm] size 4325376
[   16.291148] [drm] fb depth is 24
[   16.291150] [drm]    pitch is 5632
[   16.291395] fbcon: radeondrmfb (fb0) is primary device
[   16.987612] random: nonblocking pool is initialized
[   17.465690] Console: switching to colour frame buffer device 170x48
[   17.474022] radeon 0000:00:01.0: fb0: radeondrmfb frame buffer device
[   17.474028] radeon 0000:00:01.0: registered panic notifier
[   17.489789] [drm] Initialized radeon 2.40.0 20080528 for 0000:00:01.0 on minor 0
[   19.694646] EXT4-fs (sda6): mounted filesystem with ordered data mode. Opts: (null)
[   21.042470] squashfs: version 4.0 (2009/01/31) Phillip Lougher
[   41.832563] Adding 5705724k swap on /dev/sda7.  Priority:-1 extents:1 across:5705724k FS
[   42.117945] systemd-udevd[1185]: starting version 204
[   42.477537] lp: driver loaded but no devices found
[   42.524123] ppdev: user-space parallel port driver
[   42.781627] Bluetooth: Core ver 2.20
[   42.781678] NET: Registered protocol family 31
[   42.781685] Bluetooth: HCI device and connection manager initialized
[   42.785083] Bluetooth: HCI socket layer initialized
[   42.785102] Bluetooth: L2CAP socket layer initialized
[   42.785127] Bluetooth: SCO socket layer initialized
[   42.860083] Bluetooth: RFCOMM TTY layer initialized
[   42.860110] Bluetooth: RFCOMM socket layer initialized
[   42.860133] Bluetooth: RFCOMM ver 1.11
[   42.878491] Bluetooth: BNEP (Ethernet Emulation) ver 1.3
[   42.878503] Bluetooth: BNEP filters: protocol multicast
[   42.878518] Bluetooth: BNEP socket layer initialized
[   42.934089] shpchp: Standard Hot Plug PCI Controller Driver version: 0.4
[   43.010061] ACPI Warning: SystemIO range 0x0000000000000b00-0x0000000000000b07 conflicts with OpRegion 0x0000000000000b00-0x0000000000000b0f (_SB_.PCI0.SMBS.SMB0) (20141107/utaddress-258)
[   43.010086] ACPI: If an ACPI driver is available for this device, you should use it instead of the native driver
[   43.175751] MCE: In-kernel MCE decoding enabled.
[   43.192578] usbcore: registered new interface driver btusb
[   43.208831] EDAC MC: Ver: 3.0.0
[   43.340443] init: cups main process (1282) killed by HUP signal
[   43.340489] init: cups main process ended, respawning
[   43.449483] AMD64 EDAC driver v3.4.0
[   43.449594] EDAC amd64: DRAM ECC disabled.
[   43.449610] EDAC amd64: ECC disabled in the BIOS or no ECC capability, module will not load.
[   43.449610]  Either enable ECC checking or force module loading by setting 'ecc_enable_override'.
[   43.449610]  (Note that use of the override may cause unknown side effects.)
[   43.471439] usb 6-2: USB disconnect, device number 2
[   43.476431] usbcore: registered new interface driver ath3k
[   43.515864] media: Linux media interface: v0.10
[   43.517356] input: ELAN Touchscreen as /devices/pci0000:00/0000:00:13.0/usb6/6-3/6-3:1.0/0003:04F3:009E.0001/input/input9
[   43.518905] hid-multitouch 0003:04F3:009E.0001: input,hiddev0,hidraw0: USB HID v1.10 Device [ELAN Touchscreen] on usb-0000:00:13.0-3/input0
[   43.570731] Linux video capture interface: v2.00
[   43.627066] cfg80211: Calling CRDA to update world regulatory domain
[   43.640944] AVX version of gcm_enc/dec engaged.
[   43.640958] AES CTR mode by8 optimization enabled
[   43.739001] device-mapper: multipath: version 1.7.0 loaded
[   43.779683] uvcvideo: Found UVC 1.00 device HD WebCam (064e:e330)
[   43.792539] input: HD WebCam as /devices/pci0000:00/0000:00:13.2/usb4/4-1/4-1:1.0/input/input11
[   43.795285] usbcore: registered new interface driver uvcvideo
[   43.795296] USB Video Class driver (1.1.1)
[   43.883237] kvm: disabled by bios
[   43.954367] input: HD-Audio Generic HDMI/DP,pcm=3 as /devices/pci0000:00/0000:00:01.1/sound/card0/input12
[   43.967080] usb 6-2: new full-speed USB device number 4 using ohci-pci
[   43.981582] sound hdaudioC1D0: autoconfig: line_outs=1 (0x14/0x0/0x0/0x0/0x0) type:speaker
[   43.981602] sound hdaudioC1D0:    speaker_outs=0 (0x0/0x0/0x0/0x0/0x0)
[   43.981612] sound hdaudioC1D0:    hp_outs=1 (0x21/0x0/0x0/0x0/0x0)
[   43.981620] sound hdaudioC1D0:    mono: mono_out=0x0
[   43.981626] sound hdaudioC1D0:    inputs:
[   43.981636] sound hdaudioC1D0:      Mic=0x1b
[   43.996436] input: HD-Audio Generic Headphone as /devices/pci0000:00/0000:00:14.2/sound/card1/input13
[   44.084600] ath: phy0: WB335 1-ANT card detected
[   44.093945] ath: phy0: Enable LNA combining
[   44.095121] ath: phy0: ASPM enabled: 0x42
[   44.095131] ath: EEPROM regdomain: 0x6c
[   44.095135] ath: EEPROM indicates we should expect a direct regpair map
[   44.095141] ath: Country alpha2 being used: 00
[   44.095144] ath: Regpair used: 0x6c
[   44.123722] ieee80211 phy0: Selected rate control algorithm 'minstrel_ht'
[   44.125397] ieee80211 phy0: Atheros AR9565 Rev:1 mem=0xffffc90006180000, irq=35
[   44.231489] acer_wmi: Acer Laptop ACPI-WMI Extras
[   44.231867] acer_wmi: Function bitmap for Communication Button: 0x801
[   44.247614] acer_wmi: Brightness must be controlled by acpi video driver
[   44.260869] acer_wmi: Enabling Launch Manager failed: 0xe2 - 0x0
[   44.261185] input: Acer WMI hotkeys as /devices/virtual/input/input14
[   44.263901] input: Acer BMA150 accelerometer as /devices/virtual/input/input15
[   44.430239] cfg80211: World regulatory domain updated:
[   44.430256] cfg80211:  DFS Master region: unset
[   44.430262] cfg80211:   (start_freq - end_freq @ bandwidth), (max_antenna_gain, max_eirp), (dfs_cac_time)
[   44.430272] cfg80211:   (2402000 KHz - 2472000 KHz @ 40000 KHz), (300 mBi, 2000 mBm), (N/A)
[   44.430281] cfg80211:   (2457000 KHz - 2482000 KHz @ 40000 KHz), (300 mBi, 2000 mBm), (N/A)
[   44.430290] cfg80211:   (2474000 KHz - 2494000 KHz @ 20000 KHz), (300 mBi, 2000 mBm), (N/A)
[   44.430297] cfg80211:   (5170000 KHz - 5250000 KHz @ 40000 KHz), (300 mBi, 2000 mBm), (N/A)
[   44.430305] cfg80211:   (5735000 KHz - 5835000 KHz @ 40000 KHz), (300 mBi, 2000 mBm), (N/A)
[   44.492634] init: failsafe main process (1471) killed by TERM signal
[   45.737582] init: alsa-restore main process (1826) terminated with status 99
[   46.399123] IPv6: ADDRCONF(NETDEV_UP): wlan0: link is not ready
[   49.128460] usb 6-2: New USB device found, idVendor=0489, idProduct=e05f
[   49.128475] usb 6-2: New USB device strings: Mfr=0, Product=0, SerialNumber=0
[   50.264433] init: plymouth-upstart-bridge main process ended, respawning
[   50.277451] init: plymouth-upstart-bridge main process (2238) terminated with status 1
[   50.277492] init: plymouth-upstart-bridge main process ended, respawning
[   91.735685] systemd-hostnamed[3143]: Warning: nss-myhostname is not installed. Changing the local hostname might make it unresolveable. Please install nss-myhostname!
[  100.188514] wlan0: authenticate with 14:0c:76:72:18:d7
[  100.205667] wlan0: send auth to 14:0c:76:72:18:d7 (try 1/3)
[  100.207644] wlan0: authenticated
[  100.211348] wlan0: associate with 14:0c:76:72:18:d7 (try 1/3)
[  100.214901] wlan0: RX AssocResp from 14:0c:76:72:18:d7 (capab=0x411 status=0 aid=2)
[  100.215043] wlan0: associated
[  100.215073] IPv6: ADDRCONF(NETDEV_CHANGE): wlan0: link becomes ready
[  642.530020] systemd-hostnamed[4247]: Warning: nss-myhostname is not installed. Changing the local hostname might make it unresolveable. Please install nss-myhostname!
[  648.094957] SGI XFS with ACLs, security attributes, realtime, no debug enabled
[  648.137917] JFS: nTxBlock = 8192, nTxLock = 65536
[  648.194255] ntfs: driver 2.1.31 [Flags: R/O MODULE].
[  648.287772] QNX4 filesystem 0.2.3 registered.
[  648.533832] raid6: using ssse3x2 recovery algorithm
[  648.547329] xor: automatically using best checksumming function:
[  648.671429] Btrfs loaded
[  658.515694] EXT4-fs (sda6): mounted filesystem with ordered data mode. Opts: (null)
[  692.744306] FAT-fs (sda2): Corrupted directory (i_pos 131203)
[  706.258795] FAT-fs (sda2): Corrupted directory (i_pos 131203)
[  719.153220] FAT-fs (sda2): Corrupted directory (i_pos 131203)


=================== cat /var/log/syslog:
Sep  6 21:51:07 ubuntu rsyslogd: [origin software="rsyslogd" swVersion="7.4.4" x-pid="1316" x-info="http://www.rsyslog.com"] start
Sep  6 21:51:07 ubuntu rsyslogd: rsyslogd's groupid changed to 104
Sep  6 21:51:07 ubuntu rsyslogd: rsyslogd's userid changed to 101
Sep  6 21:51:07 ubuntu kernel: [    0.000000] Initializing cgroup subsys cpuset
Sep  6 21:51:07 ubuntu kernel: [    0.000000] Initializing cgroup subsys cpu
Sep  6 21:51:07 ubuntu kernel: [    0.000000] Initializing cgroup subsys cpuacct
Sep  6 21:51:07 ubuntu kernel: [    0.000000] Linux version 3.19.0-25-generic (buildd@lgw01-20) (gcc version 4.8.2 (Ubuntu 4.8.2-19ubuntu1) ) #26~14.04.1-Ubuntu SMP Fri Jul 24 21:16:20 UTC 2015 (Ubuntu 3.19.0-25.26~14.04.1-generic 3.19.8-ckt2)
Sep  6 21:51:07 ubuntu kernel: [    0.000000] Command line: BOOT_IMAGE=/casper/vmlinuz.efi file=/cdrom/preseed/ubuntu.seed boot=casper quiet splash --
Sep  6 21:51:07 ubuntu kernel: [    0.000000] KERNEL supported cpus:
Sep  6 21:51:07 ubuntu kernel: [    0.000000]   Intel GenuineIntel
Sep  6 21:51:07 ubuntu kernel: [    0.000000]   AMD AuthenticAMD
Sep  6 21:51:07 ubuntu kernel: [    0.000000]   Centaur CentaurHauls
Sep  6 21:51:07 ubuntu kernel: [    0.000000] tseg: 00bfc00000
Sep  6 21:51:07 ubuntu kernel: [    0.000000] e820: BIOS-provided physical RAM map:
Sep  6 21:51:07 ubuntu kernel: [    0.000000] BIOS-e820: [mem 0x0000000000000000-0x000000000006dfff] usable
Sep  6 21:51:07 ubuntu kernel: [    0.000000] BIOS-e820: [mem 0x000000000006e000-0x000000000006ffff] ACPI NVS
Sep  6 21:51:07 ubuntu kernel: [    0.000000] BIOS-e820: [mem 0x0000000000070000-0x0000000000085fff] usable
Sep  6 21:51:07 ubuntu kernel: [    0.000000] BIOS-e820: [mem 0x0000000000086000-0x00000000000bffff] reserved
Sep  6 21:51:07 ubuntu kernel: [    0.000000] BIOS-e820: [mem 0x0000000000100000-0x00000000b4b1cfff] usable
Sep  6 21:51:07 ubuntu kernel: [    0.000000] BIOS-e820: [mem 0x00000000b4b1d000-0x00000000b531dfff] reserved
Sep  6 21:51:07 ubuntu kernel: [    0.000000] BIOS-e820: [mem 0x00000000b531e000-0x00000000b537bfff] usable
Sep  6 21:51:07 ubuntu kernel: [    0.000000] BIOS-e820: [mem 0x00000000b537c000-0x00000000b677bfff] reserved
Sep  6 21:51:07 ubuntu kernel: [    0.000000] BIOS-e820: [mem 0x00000000b677c000-0x00000000bf17efff] usable
Sep  6 21:51:07 ubuntu kernel: [    0.000000] BIOS-e820: [mem 0x00000000bf17f000-0x00000000bf57efff] type 20
Sep  6 21:51:07 ubuntu kernel: [    0.000000] BIOS-e820: [mem 0x00000000bf57f000-0x00000000bf97efff] reserved
Sep  6 21:51:07 ubuntu kernel: [    0.000000] BIOS-e820: [mem 0x00000000bf97f000-0x00000000bfb7efff] ACPI NVS
Sep  6 21:51:07 ubuntu kernel: [    0.000000] BIOS-e820: [mem 0x00000000bfb7f000-0x00000000bfbfefff] ACPI data
Sep  6 21:51:07 ubuntu kernel: [    0.000000] BIOS-e820: [mem 0x00000000bfbff000-0x00000000bfbfffff] usable
Sep  6 21:51:07 ubuntu kernel: [    0.000000] BIOS-e820: [mem 0x00000000bfc00000-0x00000000dfffffff] reserved
Sep  6 21:51:07 ubuntu kernel: [    0.000000] BIOS-e820: [mem 0x00000000f8000000-0x00000000fbffffff] reserved
Sep  6 21:51:07 ubuntu kernel: [    0.000000] BIOS-e820: [mem 0x00000000fec00000-0x00000000fec00fff] reserved
Sep  6 21:51:07 ubuntu kernel: [    0.000000] BIOS-e820: [mem 0x00000000fec10000-0x00000000fec10fff] reserved
Sep  6 21:51:07 ubuntu kernel: [    0.000000] BIOS-e820: [mem 0x00000000fed80000-0x00000000fed80fff] reserved
Sep  6 21:51:07 ubuntu kernel: [    0.000000] BIOS-e820: [mem 0x00000000fee00000-0x00000000fee00fff] reserved
Sep  6 21:51:07 ubuntu kernel: [    0.000000] BIOS-e820: [mem 0x00000000ffc00000-0x00000000ffffffff] reserved
Sep  6 21:51:07 ubuntu kernel: [    0.000000] BIOS-e820: [mem 0x0000000100000000-0x000000019effffff] usable
Sep  6 21:51:07 ubuntu kernel: [    0.000000] NX (Execute Disable) protection: active
Sep  6 21:51:07 ubuntu kernel: [    0.000000] efi: EFI v2.31 by INSYDE Corp.
Sep  6 21:51:07 ubuntu kernel: [    0.000000] efi:  ACPI=0xbfbfe000  ACPI 2.0=0xbfbfe014  SMBIOS=0xbf97df18
Sep  6 21:51:07 ubuntu kernel: [    0.000000] efi: mem00: [Boot Code          |   |  |  |  |   |WB|WT|WC|UC] range=[0x0000000000000000-0x0000000000001000) (0MB)
Sep  6 21:51:07 ubuntu kernel: [    0.000000] efi: mem01: [Conventional Memory|   |  |  |  |   |WB|WT|WC|UC] range=[0x0000000000001000-0x000000000006e000) (0MB)
Sep  6 21:51:07 ubuntu kernel: [    0.000000] efi: mem02: [ACPI Memory NVS    |   |  |  |  |   |WB|WT|WC|UC] range=[0x000000000006e000-0x0000000000070000) (0MB)
Sep  6 21:51:07 ubuntu kernel: [    0.000000] efi: mem03: [Conventional Memory|   |  |  |  |   |WB|WT|WC|UC] range=[0x0000000000070000-0x0000000000086000) (0MB)
Sep  6 21:51:07 ubuntu kernel: [    0.000000] efi: mem04: [Reserved           |   |  |  |  |   |WB|WT|WC|UC] range=[0x0000000000086000-0x0000000000088000) (0MB)
Sep  6 21:51:07 ubuntu kernel: [    0.000000] efi: mem05: [Runtime Data       |RUN|  |  |  |   |WB|WT|WC|UC] range=[0x0000000000088000-0x00000000000a0000) (0MB)
Sep  6 21:51:07 ubuntu kernel: [    0.000000] efi: mem06: [Loader Data        |   |  |  |  |   |WB|WT|WC|UC] range=[0x0000000000100000-0x00000000014d6000) (19MB)
Sep  6 21:51:07 ubuntu kernel: [    0.000000] efi: mem07: [Conventional Memory|   |  |  |  |   |WB|WT|WC|UC] range=[0x00000000014d6000-0x0000000002000000) (11MB)
Sep  6 21:51:07 ubuntu kernel: [    0.000000] efi: mem08: [Loader Data        |   |  |  |  |   |WB|WT|WC|UC] range=[0x0000000002000000-0x00000000033d6000) (19MB)
Sep  6 21:51:07 ubuntu kernel: [    0.000000] efi: mem09: [Conventional Memory|   |  |  |  |   |WB|WT|WC|UC] range=[0x00000000033d6000-0x000000003568c000) (802MB)
Sep  6 21:51:07 ubuntu kernel: [    0.000000] efi: mem10: [Loader Data        |   |  |  |  |   |WB|WT|WC|UC] range=[0x000000003568c000-0x0000000036b3e000) (20MB)
Sep  6 21:51:07 ubuntu kernel: [    0.000000] efi: mem11: [Conventional Memory|   |  |  |  |   |WB|WT|WC|UC] range=[0x0000000036b3e000-0x0000000084f30000) (1251MB)
Sep  6 21:51:07 ubuntu kernel: [    0.000000] efi: mem12: [Loader Data        |   |  |  |  |   |WB|WT|WC|UC] range=[0x0000000084f30000-0x00000000b3780000) (744MB)
Sep  6 21:51:07 ubuntu kernel: [    0.000000] efi: mem13: [Boot Data          |   |  |  |  |   |WB|WT|WC|UC] range=[0x00000000b3780000-0x00000000b37a0000) (0MB)
Sep  6 21:51:07 ubuntu kernel: [    0.000000] efi: mem14: [Conventional Memory|   |  |  |  |   |WB|WT|WC|UC] range=[0x00000000b37a0000-0x00000000b471d000) (15MB)
Sep  6 21:51:07 ubuntu kernel: [    0.000000] efi: mem15: [Loader Data        |   |  |  |  |   |WB|WT|WC|UC] range=[0x00000000b471d000-0x00000000b4903000) (1MB)
Sep  6 21:51:07 ubuntu kernel: [    0.000000] efi: mem16: [Boot Data          |   |  |  |  |   |WB|WT|WC|UC] range=[0x00000000b4903000-0x00000000b4b1d000) (2MB)
Sep  6 21:51:07 ubuntu kernel: [    0.000000] efi: mem17: [Reserved           |   |  |  |  |   |WB|WT|WC|UC] range=[0x00000000b4b1d000-0x00000000b531e000) (8MB)
Sep  6 21:51:07 ubuntu kernel: [    0.000000] efi: mem18: [Boot Data          |   |  |  |  |   |WB|WT|WC|UC] range=[0x00000000b531e000-0x00000000b537c000) (0MB)
Sep  6 21:51:07 ubuntu kernel: [    0.000000] efi: mem19: [Reserved           |   |  |  |  |   |WB|WT|WC|UC] range=[0x00000000b537c000-0x00000000b677c000) (20MB)
Sep  6 21:51:07 ubuntu kernel: [    0.000000] efi: mem20: [Conventional Memory|   |  |  |  |   |WB|WT|WC|UC] range=[0x00000000b677c000-0x00000000b677d000) (0MB)
Sep  6 21:51:07 ubuntu kernel: [    0.000000] efi: mem21: [Loader Data        |   |  |  |  |   |WB|WT|WC|UC] range=[0x00000000b677d000-0x00000000b677f000) (0MB)
Sep  6 21:51:07 ubuntu kernel: [    0.000000] efi: mem22: [Conventional Memory|   |  |  |  |   |WB|WT|WC|UC] range=[0x00000000b677f000-0x00000000b6859000) (0MB)
Sep  6 21:51:07 ubuntu kernel: [    0.000000] efi: mem23: [Loader Code        |   |  |  |  |   |WB|WT|WC|UC] range=[0x00000000b6859000-0x00000000b697f000) (1MB)
Sep  6 21:51:07 ubuntu kernel: [    0.000000] efi: mem24: [Conventional Memory|   |  |  |  |   |WB|WT|WC|UC] range=[0x00000000b697f000-0x00000000bbf63000) (85MB)
Sep  6 21:51:07 ubuntu kernel: [    0.000000] efi: mem25: [Boot Data          |   |  |  |  |   |WB|WT|WC|UC] range=[0x00000000bbf63000-0x00000000bcd08000) (13MB)
Sep  6 21:51:07 ubuntu kernel: [    0.000000] efi: mem26: [Conventional Memory|   |  |  |  |   |WB|WT|WC|UC] range=[0x00000000bcd08000-0x00000000bcd0a000) (0MB)
Sep  6 21:51:07 ubuntu kernel: [    0.000000] efi: mem27: [Boot Data          |   |  |  |  |   |WB|WT|WC|UC] range=[0x00000000bcd0a000-0x00000000bcdff000) (0MB)
Sep  6 21:51:07 ubuntu kernel: [    0.000000] efi: mem28: [Conventional Memory|   |  |  |  |   |WB|WT|WC|UC] range=[0x00000000bcdff000-0x00000000bce04000) (0MB)
Sep  6 21:51:07 ubuntu kernel: [    0.000000] efi: mem29: [Boot Data          |   |  |  |  |   |WB|WT|WC|UC] range=[0x00000000bce04000-0x00000000bce39000) (0MB)
Sep  6 21:51:07 ubuntu kernel: [    0.000000] efi: mem30: [Conventional Memory|   |  |  |  |   |WB|WT|WC|UC] range=[0x00000000bce39000-0x00000000bce3c000) (0MB)
Sep  6 21:51:07 ubuntu kernel: [    0.000000] efi: mem31: [Boot Data          |   |  |  |  |   |WB|WT|WC|UC] range=[0x00000000bce3c000-0x00000000bcfbe000) (1MB)
Sep  6 21:51:07 ubuntu kernel: [    0.000000] efi: mem32: [Conventional Memory|   |  |  |  |   |WB|WT|WC|UC] range=[0x00000000bcfbe000-0x00000000bd022000) (0MB)
Sep  6 21:51:07 ubuntu kernel: [    0.000000] efi: mem33: [Boot Data          |   |  |  |  |   |WB|WT|WC|UC] range=[0x00000000bd022000-0x00000000be97f000) (25MB)
Sep  6 21:51:07 ubuntu kernel: [    0.000000] efi: mem34: [Conventional Memory|   |  |  |  |   |WB|WT|WC|UC] range=[0x00000000be97f000-0x00000000bed92000) (4MB)
Sep  6 21:51:07 ubuntu kernel: [    0.000000] efi: mem35: [Loader Data        |   |  |  |  |   |WB|WT|WC|UC] range=[0x00000000bed92000-0x00000000bed9c000) (0MB)
Sep  6 21:51:07 ubuntu kernel: [    0.000000] efi: mem36: [Boot Code          |   |  |  |  |   |WB|WT|WC|UC] range=[0x00000000bed9c000-0x00000000bf17f000) (3MB)
Sep  6 21:51:07 ubuntu kernel: [    0.000000] efi: mem37: [Runtime Code       |RUN|  |  |  |   |WB|WT|WC|UC] range=[0x00000000bf17f000-0x00000000bf57f000) (4MB)
Sep  6 21:51:07 ubuntu kernel: [    0.000000] efi: mem38: [Runtime Data       |RUN|  |  |  |   |WB|WT|WC|UC] range=[0x00000000bf57f000-0x00000000bf77f000) (2MB)
Sep  6 21:51:07 ubuntu kernel: [    0.000000] efi: mem39: [Reserved           |   |  |  |  |   |WB|WT|WC|UC] range=[0x00000000bf77f000-0x00000000bf97f000) (2MB)
Sep  6 21:51:07 ubuntu kernel: [    0.000000] efi: mem40: [ACPI Memory NVS    |   |  |  |  |   |WB|WT|WC|UC] range=[0x00000000bf97f000-0x00000000bfb7f000) (2MB)
Sep  6 21:51:07 ubuntu kernel: [    0.000000] efi: mem41: [ACPI Reclaim Memory|   |  |  |  |   |WB|WT|WC|UC] range=[0x00000000bfb7f000-0x00000000bfbff000) (0MB)
Sep  6 21:51:07 ubuntu rsyslogd-2039: Could no open output pipe '/dev/xconsole': No such file or directory [try http://www.rsyslog.com/e/2039 ]
Sep  6 21:51:07 ubuntu kernel: [    0.000000] efi: mem42: [Boot Data          |   |  |  |  |   |WB|WT|WC|UC] range=[0x00000000bfbff000-0x00000000bfc00000) (0MB)
Sep  6 21:51:07 ubuntu kernel: [    0.000000] efi: mem43: [Conventional Memory|   |  |  |  |   |WB|WT|WC|UC] range=[0x0000000100000000-0x000000019f000000) (2544MB)
Sep  6 21:51:07 ubuntu kernel: [    0.000000] efi: mem44: [Reserved           |   |  |  |  |   |  |  |  |  ] range=[0x00000000000a0000-0x00000000000c0000) (0MB)
Sep  6 21:51:07 ubuntu kernel: [    0.000000] efi: mem45: [Reserved           |   |  |  |  |   |  |  |  |  ] range=[0x00000000bfc00000-0x00000000e0000000) (516MB)
Sep  6 21:51:07 ubuntu kernel: [    0.000000] efi: mem46: [Memory Mapped I/O  |RUN|  |  |  |   |  |  |  |UC] range=[0x00000000f8000000-0x00000000fc000000) (64MB)
Sep  6 21:51:07 ubuntu kernel: [    0.000000] efi: mem47: [Memory Mapped I/O  |RUN|  |  |  |   |  |  |  |UC] range=[0x00000000fec00000-0x00000000fec01000) (0MB)
Sep  6 21:51:07 ubuntu kernel: [    0.000000] efi: mem48: [Memory Mapped I/O  |RUN|  |  |  |   |  |  |  |UC] range=[0x00000000fec10000-0x00000000fec11000) (0MB)
Sep  6 21:51:07 ubuntu kernel: [    0.000000] efi: mem49: [Memory Mapped I/O  |RUN|  |  |  |   |  |  |  |UC] range=[0x00000000fed80000-0x00000000fed81000) (0MB)
Sep  6 21:51:07 ubuntu kernel: [    0.000000] efi: mem50: [Memory Mapped I/O  |RUN|  |  |  |   |  |  |  |UC] range=[0x00000000fee00000-0x00000000fee01000) (0MB)
Sep  6 21:51:07 ubuntu kernel: [    0.000000] efi: mem51: [Memory Mapped I/O  |RUN|  |  |  |   |  |  |  |  ] range=[0x00000000ffc00000-0x0000000100000000) (4MB)
Sep  6 21:51:07 ubuntu kernel: [    0.000000] SMBIOS 2.7 present.
Sep  6 21:51:07 ubuntu kernel: [    0.000000] DMI: Acer Aspire V5-122P/Aspire V5-122P, BIOS V2.04 05/10/2013
Sep  6 21:51:07 ubuntu kernel: [    0.000000] e820: update [mem 0x00000000-0x00000fff] usable ==> reserved
Sep  6 21:51:07 ubuntu kernel: [    0.000000] e820: remove [mem 0x000a0000-0x000fffff] usable
Sep  6 21:51:07 ubuntu kernel: [    0.000000] AGP: No AGP bridge found
Sep  6 21:51:07 ubuntu kernel: [    0.000000] e820: last_pfn = 0x19f000 max_arch_pfn = 0x400000000
Sep  6 21:51:07 ubuntu kernel: [    0.000000] MTRR default type: uncachable
Sep  6 21:51:07 ubuntu kernel: [    0.000000] MTRR fixed ranges enabled:
Sep  6 21:51:07 ubuntu kernel: [    0.000000]   00000-9FFFF write-back
Sep  6 21:51:07 ubuntu kernel: [    0.000000]   A0000-BFFFF uncachable
Sep  6 21:51:07 ubuntu kernel: [    0.000000]   C0000-FFFFF write-through
Sep  6 21:51:07 ubuntu kernel: [    0.000000] MTRR variable ranges enabled:
Sep  6 21:51:07 ubuntu kernel: [    0.000000]   0 base 0000000000 mask FF80000000 write-back
Sep  6 21:51:07 ubuntu kernel: [    0.000000]   1 base 0080000000 mask FFC0000000 write-back
Sep  6 21:51:07 ubuntu kernel: [    0.000000]   2 base 00BFB67000 mask FFFFFFF000 uncachable
Sep  6 21:51:07 ubuntu kernel: [    0.000000]   3 base 00FFC00000 mask FFFFC00000 write-protect
Sep  6 21:51:07 ubuntu kernel: [    0.000000]   4 disabled
Sep  6 21:51:07 ubuntu kernel: [    0.000000]   5 disabled
Sep  6 21:51:07 ubuntu kernel: [    0.000000]   6 disabled
Sep  6 21:51:07 ubuntu kernel: [    0.000000]   7 disabled
Sep  6 21:51:07 ubuntu kernel: [    0.000000] TOM2: 000000019f000000 aka 6640M
Sep  6 21:51:07 ubuntu kernel: [    0.000000] PAT configuration [0-7]: WB  WC  UC- UC  WB  WC  UC- UC
Sep  6 21:51:07 ubuntu kernel: [    0.000000] e820: last_pfn = 0xbfc00 max_arch_pfn = 0x400000000
Sep  6 21:51:07 ubuntu kernel: [    0.000000] Scanning 1 areas for low memory corruption
Sep  6 21:51:07 ubuntu kernel: [    0.000000] Base memory trampoline at [ffff88000007c000] 7c000 size 24576
Sep  6 21:51:07 ubuntu kernel: [    0.000000] Using GB pages for direct mapping
Sep  6 21:51:07 ubuntu kernel: [    0.000000] init_memory_mapping: [mem 0x00000000-0x000fffff]
Sep  6 21:51:07 ubuntu kernel: [    0.000000]  [mem 0x00000000-0x000fffff] page 4k
Sep  6 21:51:07 ubuntu kernel: [    0.000000] BRK [0x02fd4000, 0x02fd4fff] PGTABLE
Sep  6 21:51:07 ubuntu kernel: [    0.000000] BRK [0x02fd5000, 0x02fd5fff] PGTABLE
Sep  6 21:51:07 ubuntu kernel: [    0.000000] BRK [0x02fd6000, 0x02fd6fff] PGTABLE
Sep  6 21:51:07 ubuntu kernel: [    0.000000] init_memory_mapping: [mem 0x19ee00000-0x19effffff]
Sep  6 21:51:07 ubuntu kernel: [    0.000000]  [mem 0x19ee00000-0x19effffff] page 2M
Sep  6 21:51:07 ubuntu kernel: [    0.000000] BRK [0x02fd7000, 0x02fd7fff] PGTABLE
Sep  6 21:51:07 ubuntu kernel: [    0.000000] init_memory_mapping: [mem 0x180000000-0x19edfffff]
Sep  6 21:51:07 ubuntu kernel: [    0.000000]  [mem 0x180000000-0x19edfffff] page 2M
Sep  6 21:51:07 ubuntu kernel: [    0.000000] init_memory_mapping: [mem 0x160000000-0x17fffffff]
Sep  6 21:51:07 ubuntu kernel: [    0.000000]  [mem 0x160000000-0x17fffffff] page 1G
Sep  6 21:51:07 ubuntu kernel: [    0.000000] init_memory_mapping: [mem 0x00100000-0xb4b1cfff]
Sep  6 21:51:07 ubuntu kernel: [    0.000000]  [mem 0x00100000-0x001fffff] page 4k
Sep  6 21:51:07 ubuntu kernel: [    0.000000]  [mem 0x00200000-0x3fffffff] page 2M
Sep  6 21:51:07 ubuntu kernel: [    0.000000]  [mem 0x40000000-0x7fffffff] page 1G
Sep  6 21:51:07 ubuntu kernel: [    0.000000]  [mem 0x80000000-0xb49fffff] page 2M
Sep  6 21:51:07 ubuntu kernel: [    0.000000]  [mem 0xb4a00000-0xb4b1cfff] page 4k
Sep  6 21:51:07 ubuntu kernel: [    0.000000] init_memory_mapping: [mem 0xb531e000-0xb537bfff]
Sep  6 21:51:07 ubuntu kernel: [    0.000000]  [mem 0xb531e000-0xb537bfff] page 4k
Sep  6 21:51:07 ubuntu kernel: [    0.000000] BRK [0x02fd8000, 0x02fd8fff] PGTABLE
Sep  6 21:51:07 ubuntu kernel: [    0.000000] init_memory_mapping: [mem 0xb677c000-0xbf17efff]
Sep  6 21:51:07 ubuntu kernel: [    0.000000]  [mem 0xb677c000-0xb67fffff] page 4k
Sep  6 21:51:07 ubuntu kernel: [    0.000000]  [mem 0xb6800000-0xbeffffff] page 2M
Sep  6 21:51:07 ubuntu kernel: [    0.000000]  [mem 0xbf000000-0xbf17efff] page 4k
Sep  6 21:51:07 ubuntu kernel: [    0.000000] BRK [0x02fd9000, 0x02fd9fff] PGTABLE
Sep  6 21:51:07 ubuntu kernel: [    0.000000] init_memory_mapping: [mem 0xbfbff000-0xbfbfffff]
Sep  6 21:51:07 ubuntu kernel: [    0.000000]  [mem 0xbfbff000-0xbfbfffff] page 4k
Sep  6 21:51:07 ubuntu kernel: [    0.000000] init_memory_mapping: [mem 0x100000000-0x15fffffff]
Sep  6 21:51:07 ubuntu kernel: [    0.000000]  [mem 0x100000000-0x15fffffff] page 1G
Sep  6 21:51:07 ubuntu kernel: [    0.000000] RAMDISK: [mem 0x3568c000-0x36b3dfff]
Sep  6 21:51:07 ubuntu kernel: [    0.000000] ACPI: Early table checksum verification disabled
Sep  6 21:51:07 ubuntu kernel: [    0.000000] ACPI: RSDP 0x00000000BFBFE014 000024 (v02 ACRSYS)
Sep  6 21:51:07 ubuntu kernel: [    0.000000] ACPI: XSDT 0x00000000BFBFE120 0000B4 (v01 ACRSYS ACRPRDCT 00000001      01000013)
Sep  6 21:51:07 ubuntu kernel: [    0.000000] ACPI: FACP 0x00000000BFBFC000 00010C (v05 ACRSYS ACRPRDCT 00000001 1025 00040000)
Sep  6 21:51:07 ubuntu kernel: [    0.000000] ACPI: DSDT 0x00000000BFBEE000 009FDC (v01 ACRSYS ACRPRDCT F0000000 1025 00040000)
Sep  6 21:51:07 ubuntu kernel: [    0.000000] ACPI: FACS 0x00000000BFB26000 000040
Sep  6 21:51:07 ubuntu kernel: [    0.000000] ACPI: UEFI 0x00000000BFBFD000 000236 (v01 ACRSYS ACRPRDCT 00000001 1025 00040000)
Sep  6 21:51:07 ubuntu kernel: [    0.000000] ACPI: HPET 0x00000000BFBFB000 000038 (v01 ACRSYS ACRPRDCT 00000001 1025 00040000)
Sep  6 21:51:07 ubuntu kernel: [    0.000000] ACPI: APIC 0x00000000BFBFA000 000090 (v03 ACRSYS ACRPRDCT 00000001 1025 00040000)
Sep  6 21:51:07 ubuntu kernel: [    0.000000] ACPI: MCFG 0x00000000BFBF9000 00003C (v01 ACRSYS ACRPRDCT 00000001 1025 00040000)
Sep  6 21:51:07 ubuntu kernel: [    0.000000] ACPI: ASF! 0x00000000BFBF8000 0000A5 (v32 ACRSYS ACRPRDCT 00000001 1025 00040000)
Sep  6 21:51:07 ubuntu kernel: [    0.000000] ACPI: BOOT 0x00000000BFBED000 000028 (v01 ACRSYS ACRPRDCT 00000001 1025 00040000)
Sep  6 21:51:07 ubuntu kernel: [    0.000000] ACPI: WDRT 0x00000000BFBEC000 000047 (v01 ACRSYS ACRPRDCT 00000000 1025 00040000)
Sep  6 21:51:07 ubuntu kernel: [    0.000000] ACPI: WDAT 0x00000000BFBEB000 0001AC (v01 ACRSYS ACRPRDCT 00000001 1025 00040000)
Sep  6 21:51:07 ubuntu kernel: [    0.000000] ACPI: FPDT 0x00000000BFBEA000 000044 (v01 ACRSYS ACRPRDCT 00000001 1025 00040000)
Sep  6 21:51:07 ubuntu kernel: [    0.000000] ACPI: MSDM 0x00000000BFBE8000 000055 (v03 ACRSYS ACRPRDCT 00000001 1025 00040000)
Sep  6 21:51:07 ubuntu kernel: [    0.000000] ACPI: SSDT 0x00000000BFBE7000 000B10 (v01 ACRSYS ACRPRDCT 00000001 1025 00040000)
Sep  6 21:51:07 ubuntu kernel: [    0.000000] ACPI: SSDT 0x00000000BFBE0000 006352 (v02 ACRSYS ACRPRDCT 00000002 1025 00040000)
Sep  6 21:51:07 ubuntu kernel: [    0.000000] ACPI: VFCT 0x00000000BFBDB000 004684 (v01 ACRSYS ACRPRDCT 00000001 1025 00040000)
Sep  6 21:51:07 ubuntu kernel: [    0.000000] ACPI: SSDT 0x00000000BFBD9000 0013CA (v01 ACRSYS ACRPRDCT 00000001 1025 00040000)
Sep  6 21:51:07 ubuntu kernel: [    0.000000] ACPI: SSDT 0x00000000BFBD8000 000D61 (v01 ACRSYS ACRPRDCT 00000001 1025 00040000)
Sep  6 21:51:07 ubuntu kernel: [    0.000000] ACPI: SSDT 0x00000000BFBD6000 0010C2 (v01 ACRSYS ACRPRDCT 00000001 1025 00040000)
Sep  6 21:51:07 ubuntu kernel: [    0.000000] ACPI: BGRT 0x00000000BFBD5000 000038 (v01 ACRSYS ACRPRDCT 00000001 1025 00040000)
Sep  6 21:51:07 ubuntu kernel: [    0.000000] ACPI: Local APIC address 0xfee00000
Sep  6 21:51:07 ubuntu kernel: [    0.000000] No NUMA configuration found
Sep  6 21:51:07 ubuntu kernel: [    0.000000] Faking a node at [mem 0x0000000000000000-0x000000019effffff]
Sep  6 21:51:07 ubuntu kernel: [    0.000000] NODE_DATA(0) allocated [mem 0x19eff7000-0x19effbfff]
Sep  6 21:51:07 ubuntu kernel: [    0.000000]  [ffffea0000000000-ffffea00067fffff] PMD -> [ffff880198e00000-ffff88019e5fffff] on node 0
Sep  6 21:51:07 ubuntu kernel: [    0.000000] Zone ranges:
Sep  6 21:51:07 ubuntu kernel: [    0.000000]   DMA      [mem 0x00001000-0x00ffffff]
Sep  6 21:51:07 ubuntu kernel: [    0.000000]   DMA32    [mem 0x01000000-0xffffffff]
Sep  6 21:51:07 ubuntu kernel: [    0.000000]   Normal   [mem 0x100000000-0x19effffff]
Sep  6 21:51:07 ubuntu kernel: [    0.000000] Movable zone start for each node
Sep  6 21:51:07 ubuntu kernel: [    0.000000] Early memory node ranges
Sep  6 21:51:07 ubuntu kernel: [    0.000000]   node   0: [mem 0x00001000-0x0006dfff]
Sep  6 21:51:07 ubuntu kernel: [    0.000000]   node   0: [mem 0x00070000-0x00085fff]
Sep  6 21:51:07 ubuntu kernel: [    0.000000]   node   0: [mem 0x00100000-0xb4b1cfff]
Sep  6 21:51:07 ubuntu kernel: [    0.000000]   node   0: [mem 0xb531e000-0xb537bfff]
Sep  6 21:51:07 ubuntu kernel: [    0.000000]   node   0: [mem 0xb677c000-0xbf17efff]
Sep  6 21:51:07 ubuntu kernel: [    0.000000]   node   0: [mem 0xbfbff000-0xbfbfffff]
Sep  6 21:51:07 ubuntu kernel: [    0.000000]   node   0: [mem 0x100000000-0x19effffff]
Sep  6 21:51:07 ubuntu kernel: [    0.000000] Initmem setup node 0 [mem 0x00001000-0x19effffff]
Sep  6 21:51:07 ubuntu kernel: [    0.000000] On node 0 totalpages: 1426690
Sep  6 21:51:07 ubuntu kernel: [    0.000000]   DMA zone: 64 pages used for memmap
Sep  6 21:51:07 ubuntu kernel: [    0.000000]   DMA zone: 22 pages reserved
Sep  6 21:51:07 ubuntu kernel: [    0.000000]   DMA zone: 3971 pages, LIFO batch:0
Sep  6 21:51:07 ubuntu kernel: [    0.000000]   DMA32 zone: 12054 pages used for memmap
Sep  6 21:51:07 ubuntu kernel: [    0.000000]   DMA32 zone: 771455 pages, LIFO batch:31
Sep  6 21:51:07 ubuntu kernel: [    0.000000]   Normal zone: 10176 pages used for memmap
Sep  6 21:51:07 ubuntu kernel: [    0.000000]   Normal zone: 651264 pages, LIFO batch:31
Sep  6 21:51:07 ubuntu kernel: [    0.000000] ACPI: PM-Timer IO Port: 0x408
Sep  6 21:51:07 ubuntu kernel: [    0.000000] ACPI: Local APIC address 0xfee00000
Sep  6 21:51:07 ubuntu kernel: [    0.000000] ACPI: LAPIC (acpi_id[0x00] lapic_id[0x00] enabled)
Sep  6 21:51:07 ubuntu kernel: [    0.000000] ACPI: LAPIC (acpi_id[0x01] lapic_id[0x01] enabled)
Sep  6 21:51:07 ubuntu kernel: [    0.000000] ACPI: LAPIC (acpi_id[0x02] lapic_id[0x02] enabled)
Sep  6 21:51:07 ubuntu kernel: [    0.000000] ACPI: LAPIC (acpi_id[0x03] lapic_id[0x03] enabled)
Sep  6 21:51:07 ubuntu kernel: [    0.000000] ACPI: LAPIC_NMI (acpi_id[0x00] high edge lint[0x1])
Sep  6 21:51:07 ubuntu kernel: [    0.000000] ACPI: LAPIC_NMI (acpi_id[0x01] high edge lint[0x1])
Sep  6 21:51:07 ubuntu kernel: [    0.000000] ACPI: LAPIC_NMI (acpi_id[0x02] high edge lint[0x1])
Sep  6 21:51:07 ubuntu kernel: [    0.000000] ACPI: LAPIC_NMI (acpi_id[0x03] high edge lint[0x1])
Sep  6 21:51:07 ubuntu kernel: [    0.000000] ACPI: IOAPIC (id[0x04] address[0xfec00000] gsi_base[0])
Sep  6 21:51:07 ubuntu kernel: [    0.000000] IOAPIC[0]: apic_id 4, version 33, address 0xfec00000, GSI 0-23
Sep  6 21:51:07 ubuntu kernel: [    0.000000] ACPI: IOAPIC (id[0x05] address[0xfec01000] gsi_base[24])
Sep  6 21:51:07 ubuntu kernel: [    0.000000] IOAPIC[1]: apic_id 5, version 33, address 0xfec01000, GSI 24-55
Sep  6 21:51:07 ubuntu kernel: [    0.000000] ACPI: INT_SRC_OVR (bus 0 bus_irq 0 global_irq 2 dfl dfl)
Sep  6 21:51:07 ubuntu kernel: [    0.000000] ACPI: INT_SRC_OVR (bus 0 bus_irq 9 global_irq 9 low level)
Sep  6 21:51:07 ubuntu kernel: [    0.000000] ACPI: IRQ0 used by override.
Sep  6 21:51:07 ubuntu kernel: [    0.000000] ACPI: IRQ9 used by override.
Sep  6 21:51:07 ubuntu kernel: [    0.000000] Using ACPI (MADT) for SMP configuration information
Sep  6 21:51:07 ubuntu kernel: [    0.000000] ACPI: HPET id: 0x10228210 base: 0xfed00000
Sep  6 21:51:07 ubuntu kernel: [    0.000000] smpboot: Allowing 4 CPUs, 0 hotplug CPUs
Sep  6 21:51:07 ubuntu kernel: [    0.000000] PM: Registered nosave memory: [mem 0x00000000-0x00000fff]
Sep  6 21:51:07 ubuntu kernel: [    0.000000] PM: Registered nosave memory: [mem 0x0006e000-0x0006ffff]
Sep  6 21:51:07 ubuntu kernel: [    0.000000] PM: Registered nosave memory: [mem 0x00086000-0x000bffff]
Sep  6 21:51:07 ubuntu kernel: [    0.000000] PM: Registered nosave memory: [mem 0x000c0000-0x000fffff]
Sep  6 21:51:07 ubuntu kernel: [    0.000000] PM: Registered nosave memory: [mem 0xb4b1d000-0xb531dfff]
Sep  6 21:51:07 ubuntu kernel: [    0.000000] PM: Registered nosave memory: [mem 0xb537c000-0xb677bfff]
Sep  6 21:51:07 ubuntu kernel: [    0.000000] PM: Registered nosave memory: [mem 0xbf17f000-0xbf57efff]
Sep  6 21:51:07 ubuntu kernel: [    0.000000] PM: Registered nosave memory: [mem 0xbf57f000-0xbf97efff]
Sep  6 21:51:07 ubuntu kernel: [    0.000000] PM: Registered nosave memory: [mem 0xbf97f000-0xbfb7efff]
Sep  6 21:51:07 ubuntu kernel: [    0.000000] PM: Registered nosave memory: [mem 0xbfb7f000-0xbfbfefff]
Sep  6 21:51:07 ubuntu kernel: [    0.000000] PM: Registered nosave memory: [mem 0xbfc00000-0xdfffffff]
Sep  6 21:51:07 ubuntu kernel: [    0.000000] PM: Registered nosave memory: [mem 0xe0000000-0xf7ffffff]
Sep  6 21:51:07 ubuntu kernel: [    0.000000] PM: Registered nosave memory: [mem 0xf8000000-0xfbffffff]
Sep  6 21:51:07 ubuntu kernel: [    0.000000] PM: Registered nosave memory: [mem 0xfc000000-0xfebfffff]
Sep  6 21:51:07 ubuntu kernel: [    0.000000] PM: Registered nosave memory: [mem 0xfec00000-0xfec00fff]
Sep  6 21:51:07 ubuntu kernel: [    0.000000] PM: Registered nosave memory: [mem 0xfec01000-0xfec0ffff]
Sep  6 21:51:07 ubuntu kernel: [    0.000000] PM: Registered nosave memory: [mem 0xfec10000-0xfec10fff]
Sep  6 21:51:07 ubuntu kernel: [    0.000000] PM: Registered nosave memory: [mem 0xfec11000-0xfed7ffff]
Sep  6 21:51:07 ubuntu kernel: [    0.000000] PM: Registered nosave memory: [mem 0xfed80000-0xfed80fff]
Sep  6 21:51:07 ubuntu kernel: [    0.000000] PM: Registered nosave memory: [mem 0xfed81000-0xfedfffff]
Sep  6 21:51:07 ubuntu kernel: [    0.000000] PM: Registered nosave memory: [mem 0xfee00000-0xfee00fff]
Sep  6 21:51:07 ubuntu kernel: [    0.000000] PM: Registered nosave memory: [mem 0xfee01000-0xffbfffff]
Sep  6 21:51:07 ubuntu kernel: [    0.000000] PM: Registered nosave memory: [mem 0xffc00000-0xffffffff]
Sep  6 21:51:07 ubuntu kernel: [    0.000000] e820: [mem 0xe0000000-0xf7ffffff] available for PCI devices
Sep  6 21:51:07 ubuntu kernel: [    0.000000] Booting paravirtualized kernel on bare hardware
Sep  6 21:51:07 ubuntu kernel: [    0.000000] setup_percpu: NR_CPUS:256 nr_cpumask_bits:256 nr_cpu_ids:4 nr_node_ids:1
Sep  6 21:51:07 ubuntu kernel: [    0.000000] PERCPU: Embedded 31 pages/cpu @ffff88019ec00000 s86144 r8192 d32640 u524288
Sep  6 21:51:07 ubuntu kernel: [    0.000000] pcpu-alloc: s86144 r8192 d32640 u524288 alloc=1*2097152
Sep  6 21:51:07 ubuntu kernel: [    0.000000] pcpu-alloc: [0] 0 1 2 3
Sep  6 21:51:07 ubuntu kernel: [    0.000000] Built 1 zonelists in Node order, mobility grouping on.  Total pages: 1404374
Sep  6 21:51:07 ubuntu kernel: [    0.000000] Policy zone: Normal
Sep  6 21:51:07 ubuntu kernel: [    0.000000] Kernel command line: BOOT_IMAGE=/casper/vmlinuz.efi file=/cdrom/preseed/ubuntu.seed boot=casper quiet splash --
Sep  6 21:51:07 ubuntu kernel: [    0.000000] PID hash table entries: 4096 (order: 3, 32768 bytes)
Sep  6 21:51:07 ubuntu kernel: [    0.000000] xsave: enabled xstate_bv 0x7, cntxt size 0x340 using standard form
Sep  6 21:51:07 ubuntu kernel: [    0.000000] AGP: Checking aperture...
Sep  6 21:51:07 ubuntu kernel: [    0.000000] AGP: No AGP bridge found
Sep  6 21:51:07 ubuntu kernel: [    0.000000] AGP: Node 0: aperture [bus addr 0x00000000-0x01ffffff] (32MB)
Sep  6 21:51:07 ubuntu kernel: [    0.000000] AGP: Your BIOS doesn't leave a aperture memory hole
Sep  6 21:51:07 ubuntu kernel: [    0.000000] AGP: Please enable the IOMMU option in the BIOS setup
Sep  6 21:51:07 ubuntu kernel: [    0.000000] AGP: This costs you 64MB of RAM
Sep  6 21:51:07 ubuntu kernel: [    0.000000] AGP: Mapping aperture over RAM [mem 0xac000000-0xafffffff] (65536KB)
Sep  6 21:51:07 ubuntu kernel: [    0.000000] PM: Registered nosave memory: [mem 0xac000000-0xafffffff]
Sep  6 21:51:07 ubuntu kernel: [    0.000000] Memory: 5397444K/5706760K available (7918K kernel code, 1172K rwdata, 3752K rodata, 1408K init, 1292K bss, 309316K reserved, 0K cma-reserved)
Sep  6 21:51:07 ubuntu kernel: [    0.000000] SLUB: HWalign=64, Order=0-3, MinObjects=0, CPUs=4, Nodes=1
Sep  6 21:51:07 ubuntu kernel: [    0.000000] Hierarchical RCU implementation.
Sep  6 21:51:07 ubuntu kernel: [    0.000000] 	RCU dyntick-idle grace-period acceleration is enabled.
Sep  6 21:51:07 ubuntu kernel: [    0.000000] 	RCU restricting CPUs from NR_CPUS=256 to nr_cpu_ids=4.
Sep  6 21:51:07 ubuntu kernel: [    0.000000] RCU: Adjusting geometry for rcu_fanout_leaf=16, nr_cpu_ids=4
Sep  6 21:51:07 ubuntu kernel: [    0.000000] NR_IRQS:16640 nr_irqs:1000 16
Sep  6 21:51:07 ubuntu kernel: [    0.000000] 	Offload RCU callbacks from all CPUs
Sep  6 21:51:07 ubuntu kernel: [    0.000000] 	Offload RCU callbacks from CPUs: 0-3.
Sep  6 21:51:07 ubuntu kernel: [    0.000000] Console: colour dummy device 80x25
Sep  6 21:51:07 ubuntu kernel: [    0.000000] console [tty0] enabled
Sep  6 21:51:07 ubuntu kernel: [    0.000000] hpet clockevent registered
Sep  6 21:51:07 ubuntu kernel: [    0.000000] tsc: Fast TSC calibration using PIT
Sep  6 21:51:07 ubuntu kernel: [    0.000000] tsc: Detected 998.184 MHz processor
Sep  6 21:51:07 ubuntu kernel: [    0.000077] Calibrating delay loop (skipped), value calculated using timer frequency.. 1996.36 BogoMIPS (lpj=3992736)
Sep  6 21:51:07 ubuntu kernel: [    0.000084] pid_max: default: 32768 minimum: 301
Sep  6 21:51:07 ubuntu kernel: [    0.000101] ACPI: Core revision 20141107
Sep  6 21:51:07 ubuntu kernel: [    0.033737] ACPI: All ACPI Tables successfully acquired
Sep  6 21:51:07 ubuntu kernel: [    0.061431] Security Framework initialized
Sep  6 21:51:07 ubuntu kernel: [    0.061492] AppArmor: AppArmor initialized
Sep  6 21:51:07 ubuntu kernel: [    0.061498] Yama: becoming mindful.
Sep  6 21:51:07 ubuntu kernel: [    0.063354] Dentry cache hash table entries: 1048576 (order: 11, 8388608 bytes)
Sep  6 21:51:07 ubuntu kernel: [    0.070325] Inode-cache hash table entries: 524288 (order: 10, 4194304 bytes)
Sep  6 21:51:07 ubuntu kernel: [    0.073551] Mount-cache hash table entries: 16384 (order: 5, 131072 bytes)
Sep  6 21:51:07 ubuntu kernel: [    0.073604] Mountpoint-cache hash table entries: 16384 (order: 5, 131072 bytes)
Sep  6 21:51:07 ubuntu kernel: [    0.074541] Initializing cgroup subsys memory
Sep  6 21:51:07 ubuntu kernel: [    0.074564] Initializing cgroup subsys devices
Sep  6 21:51:07 ubuntu kernel: [    0.074582] Initializing cgroup subsys freezer
Sep  6 21:51:07 ubuntu kernel: [    0.074594] Initializing cgroup subsys net_cls
Sep  6 21:51:07 ubuntu kernel: [    0.074605] Initializing cgroup subsys blkio
Sep  6 21:51:07 ubuntu kernel: [    0.074615] Initializing cgroup subsys perf_event
Sep  6 21:51:07 ubuntu kernel: [    0.074626] Initializing cgroup subsys net_prio
Sep  6 21:51:07 ubuntu kernel: [    0.074636] Initializing cgroup subsys hugetlb
Sep  6 21:51:07 ubuntu kernel: [    0.074715] CPU: Physical Processor ID: 0
Sep  6 21:51:07 ubuntu kernel: [    0.074721] CPU: Processor Core ID: 0
Sep  6 21:51:07 ubuntu kernel: [    0.074727] mce: CPU supports 6 MCE banks
Sep  6 21:51:07 ubuntu kernel: [    0.074754] Last level iTLB entries: 4KB 512, 2MB 8, 4MB 4
Sep  6 21:51:07 ubuntu kernel: [    0.074754] Last level dTLB entries: 4KB 512, 2MB 256, 4MB 128, 1GB 0
Sep  6 21:51:07 ubuntu kernel: [    0.075066] Freeing SMP alternatives memory: 32K (ffffffff81e87000 - ffffffff81e8f000)
Sep  6 21:51:07 ubuntu kernel: [    0.079406] Ignoring BGRT: invalid status 0 (expected 1)
Sep  6 21:51:07 ubuntu kernel: [    0.084438] ftrace: allocating 29988 entries in 118 pages
Sep  6 21:51:07 ubuntu kernel: [    0.115140] ..TIMER: vector=0x30 apic1=0 pin1=2 apic2=-1 pin2=-1
Sep  6 21:51:07 ubuntu kernel: [    0.154845] smpboot: CPU0: AMD A6-1450 APU with Radeon(TM) HD Graphics (fam: 16, model: 00, stepping: 01)
Sep  6 21:51:07 ubuntu kernel: [    0.261794] Performance Events: AMD PMU driver.
Sep  6 21:51:07 ubuntu kernel: [    0.261805] ... version:                0
Sep  6 21:51:07 ubuntu kernel: [    0.261809] ... bit width:              48
Sep  6 21:51:07 ubuntu kernel: [    0.261811] ... generic registers:      4
Sep  6 21:51:07 ubuntu kernel: [    0.261815] ... value mask:             0000ffffffffffff
Sep  6 21:51:07 ubuntu kernel: [    0.261818] ... max period:             00007fffffffffff
Sep  6 21:51:07 ubuntu kernel: [    0.261821] ... fixed-purpose events:   0
Sep  6 21:51:07 ubuntu kernel: [    0.261824] ... event mask:             000000000000000f
Sep  6 21:51:07 ubuntu kernel: [    0.263840] NMI watchdog: enabled on all CPUs, permanently consumes one hw-PMU counter.
Sep  6 21:51:07 ubuntu kernel: [    0.264143] x86: Booting SMP configuration:
Sep  6 21:51:07 ubuntu kernel: [    0.264148] .... node  #0, CPUs:      #1 #2 #3
Sep  6 21:51:07 ubuntu kernel: [    0.304396] x86: Booted up 1 node, 4 CPUs
Sep  6 21:51:07 ubuntu kernel: [    0.304409] smpboot: Total of 4 processors activated (7985.47 BogoMIPS)
Sep  6 21:51:07 ubuntu kernel: [    0.306035] devtmpfs: initialized
Sep  6 21:51:07 ubuntu kernel: [    0.319968] evm: security.selinux
Sep  6 21:51:07 ubuntu kernel: [    0.319972] evm: security.SMACK64
Sep  6 21:51:07 ubuntu kernel: [    0.319976] evm: security.SMACK64EXEC
Sep  6 21:51:07 ubuntu kernel: [    0.319979] evm: security.SMACK64TRANSMUTE
Sep  6 21:51:07 ubuntu kernel: [    0.319982] evm: security.SMACK64MMAP
Sep  6 21:51:07 ubuntu kernel: [    0.319984] evm: security.ima
Sep  6 21:51:07 ubuntu kernel: [    0.319987] evm: security.capability
Sep  6 21:51:07 ubuntu kernel: [    0.320195] PM: Registering ACPI NVS region [mem 0x0006e000-0x0006ffff] (8192 bytes)
Sep  6 21:51:07 ubuntu kernel: [    0.320202] PM: Registering ACPI NVS region [mem 0xbf97f000-0xbfb7efff] (2097152 bytes)
Sep  6 21:51:07 ubuntu kernel: [    0.320697] pinctrl core: initialized pinctrl subsystem
Sep  6 21:51:07 ubuntu kernel: [    0.320954] RTC time: 21:50:24, date: 09/06/15
Sep  6 21:51:07 ubuntu kernel: [    0.321248] NET: Registered protocol family 16
Sep  6 21:51:07 ubuntu kernel: [    0.329529] cpuidle: using governor ladder
Sep  6 21:51:07 ubuntu kernel: [    0.333520] cpuidle: using governor menu
Sep  6 21:51:07 ubuntu kernel: [    0.333760] ACPI: bus type PCI registered
Sep  6 21:51:07 ubuntu kernel: [    0.333767] acpiphp: ACPI Hot Plug PCI Controller Driver version: 0.5
Sep  6 21:51:07 ubuntu kernel: [    0.334010] PCI: MMCONFIG for domain 0000 [bus 00-3f] at [mem 0xf8000000-0xfbffffff] (base 0xf8000000)
Sep  6 21:51:07 ubuntu kernel: [    0.334018] PCI: MMCONFIG at [mem 0xf8000000-0xfbffffff] reserved in E820
Sep  6 21:51:07 ubuntu kernel: [    0.334258] PCI: Using configuration type 1 for base access
Sep  6 21:51:07 ubuntu kernel: [    0.342852] ACPI: Added _OSI(Module Device)
Sep  6 21:51:07 ubuntu kernel: [    0.342863] ACPI: Added _OSI(Processor Device)
Sep  6 21:51:07 ubuntu kernel: [    0.342867] ACPI: Added _OSI(3.0 _SCP Extensions)
Sep  6 21:51:07 ubuntu kernel: [    0.342871] ACPI: Added _OSI(Processor Aggregator Device)
Sep  6 21:51:07 ubuntu kernel: [    0.350735] ACPI: Executed 1 blocks of module-level executable AML code
Sep  6 21:51:07 ubuntu kernel: [    0.362562] [Firmware Bug]: ACPI: BIOS _OSI(Linux) query ignored
Sep  6 21:51:07 ubuntu kernel: [    2.399625] ACPI: Interpreter enabled
Sep  6 21:51:07 ubuntu kernel: [    2.399656] ACPI Exception: AE_NOT_FOUND, While evaluating Sleep State [_S1_] (20141107/hwxface-580)
Sep  6 21:51:07 ubuntu kernel: [    2.399670] ACPI Exception: AE_NOT_FOUND, While evaluating Sleep State [_S2_] (20141107/hwxface-580)
Sep  6 21:51:07 ubuntu kernel: [    2.399713] ACPI: (supports S0 S3 S4 S5)
Sep  6 21:51:07 ubuntu kernel: [    2.399719] ACPI: Using IOAPIC for interrupt routing
Sep  6 21:51:07 ubuntu kernel: [    2.400233] PCI: Using host bridge windows from ACPI; if necessary, use "pci=nocrs" and report a bug
Sep  6 21:51:07 ubuntu kernel: [    2.420502] ACPI: PCI Root Bridge [PCI0] (domain 0000 [bus 00-ff])
Sep  6 21:51:07 ubuntu kernel: [    2.420520] acpi PNP0A08:00: _OSC: OS supports [ExtendedConfig ASPM ClockPM Segments MSI]
Sep  6 21:51:07 ubuntu kernel: [    2.420971] acpi PNP0A08:00: _OSC: platform does not support [PME]
Sep  6 21:51:07 ubuntu kernel: [    2.421391] acpi PNP0A08:00: _OSC: OS now controls [PCIeHotplug AER PCIeCapability]
Sep  6 21:51:07 ubuntu kernel: [    2.421803] acpi PNP0A08:00: [Firmware Info]: MMCONFIG for domain 0000 [bus 00-3f] only partially covers this bridge
Sep  6 21:51:07 ubuntu kernel: [    2.422145] PCI host bridge to bus 0000:00
Sep  6 21:51:07 ubuntu kernel: [    2.422155] pci_bus 0000:00: root bus resource [bus 00-ff]
Sep  6 21:51:07 ubuntu kernel: [    2.422162] pci_bus 0000:00: root bus resource [io  0x0000-0x0cf7]
Sep  6 21:51:07 ubuntu kernel: [    2.422168] pci_bus 0000:00: root bus resource [io  0x0d00-0xffff]
Sep  6 21:51:07 ubuntu kernel: [    2.422174] pci_bus 0000:00: root bus resource [mem 0x000a0000-0x000bffff]
Sep  6 21:51:07 ubuntu kernel: [    2.422180] pci_bus 0000:00: root bus resource [mem 0x000c0000-0x000c3fff]
Sep  6 21:51:07 ubuntu kernel: [    2.422185] pci_bus 0000:00: root bus resource [mem 0x000c4000-0x000c7fff]
Sep  6 21:51:07 ubuntu kernel: [    2.422191] pci_bus 0000:00: root bus resource [mem 0x000c8000-0x000cbfff]
Sep  6 21:51:07 ubuntu kernel: [    2.422196] pci_bus 0000:00: root bus resource [mem 0x000cc000-0x000cffff]
Sep  6 21:51:07 ubuntu kernel: [    2.422201] pci_bus 0000:00: root bus resource [mem 0x000d0000-0x000d3fff]
Sep  6 21:51:07 ubuntu kernel: [    2.422206] pci_bus 0000:00: root bus resource [mem 0x000d4000-0x000d7fff]
Sep  6 21:51:07 ubuntu kernel: [    2.422211] pci_bus 0000:00: root bus resource [mem 0x000d8000-0x000dbfff]
Sep  6 21:51:07 ubuntu kernel: [    2.422217] pci_bus 0000:00: root bus resource [mem 0x000dc000-0x000dffff]
Sep  6 21:51:07 ubuntu kernel: [    2.422222] pci_bus 0000:00: root bus resource [mem 0x000e0000-0x000e3fff]
Sep  6 21:51:07 ubuntu kernel: [    2.422227] pci_bus 0000:00: root bus resource [mem 0x000e4000-0x000e7fff]
Sep  6 21:51:07 ubuntu kernel: [    2.422232] pci_bus 0000:00: root bus resource [mem 0x000e8000-0x000ebfff]
Sep  6 21:51:07 ubuntu kernel: [    2.422237] pci_bus 0000:00: root bus resource [mem 0x000ec000-0x000effff]
Sep  6 21:51:07 ubuntu kernel: [    2.422242] pci_bus 0000:00: root bus resource [mem 0xe0000000-0xf7ffffff]
Sep  6 21:51:07 ubuntu kernel: [    2.422247] pci_bus 0000:00: root bus resource [mem 0xfc000000-0xfed3ffff]
Sep  6 21:51:07 ubuntu kernel: [    2.422252] pci_bus 0000:00: root bus resource [mem 0xfed45000-0xffffffff]
Sep  6 21:51:07 ubuntu kernel: [    2.422270] pci 0000:00:00.0: [1022:1536] type 00 class 0x060000
Sep  6 21:51:07 ubuntu kernel: [    2.422490] pci 0000:00:01.0: [1002:983d] type 00 class 0x030000
Sep  6 21:51:07 ubuntu kernel: [    2.422517] pci 0000:00:01.0: reg 0x10: [mem 0xe0000000-0xefffffff 64bit pref]
Sep  6 21:51:07 ubuntu kernel: [    2.422535] pci 0000:00:01.0: reg 0x18: [mem 0xf0000000-0xf07fffff 64bit pref]
Sep  6 21:51:07 ubuntu kernel: [    2.422547] pci 0000:00:01.0: reg 0x20: [io  0x2000-0x20ff]
Sep  6 21:51:07 ubuntu kernel: [    2.422561] pci 0000:00:01.0: reg 0x24: [mem 0xf0900000-0xf093ffff]
Sep  6 21:51:07 ubuntu kernel: [    2.422573] pci 0000:00:01.0: reg 0x30: [mem 0xfffe0000-0xffffffff pref]
Sep  6 21:51:07 ubuntu kernel: [    2.422641] pci 0000:00:01.0: supports D1 D2
Sep  6 21:51:07 ubuntu kernel: [    2.422647] pci 0000:00:01.0: PME# supported from D1 D2 D3hot
Sep  6 21:51:07 ubuntu kernel: [    2.422849] pci 0000:00:01.1: [1002:9840] type 00 class 0x040300
Sep  6 21:51:07 ubuntu kernel: [    2.422873] pci 0000:00:01.1: reg 0x10: [mem 0xf0940000-0xf0943fff 64bit]
Sep  6 21:51:07 ubuntu kernel: [    2.422971] pci 0000:00:01.1: supports D1 D2
Sep  6 21:51:07 ubuntu kernel: [    2.423131] pci 0000:00:02.0: [1022:1538] type 00 class 0x060000
Sep  6 21:51:07 ubuntu kernel: [    2.423333] pci 0000:00:02.3: [1022:1439] type 01 class 0x060400
Sep  6 21:51:07 ubuntu kernel: [    2.423445] pci 0000:00:02.3: PME# supported from D0 D3hot D3cold
Sep  6 21:51:07 ubuntu kernel: [    2.423538] pci 0000:00:02.3: System wakeup disabled by ACPI
Sep  6 21:51:07 ubuntu kernel: [    2.423675] pci 0000:00:10.0: [1022:7814] type 00 class 0x0c0330
Sep  6 21:51:07 ubuntu kernel: [    2.423705] pci 0000:00:10.0: reg 0x10: [mem 0xf0948000-0xf0949fff 64bit]
Sep  6 21:51:07 ubuntu kernel: [    2.423832] pci 0000:00:10.0: PME# supported from D0 D3hot D3cold
Sep  6 21:51:07 ubuntu kernel: [    2.423925] pci 0000:00:10.0: System wakeup disabled by ACPI
Sep  6 21:51:07 ubuntu kernel: [    2.424038] pci 0000:00:11.0: [1022:7801] type 00 class 0x010601
Sep  6 21:51:07 ubuntu kernel: [    2.424064] pci 0000:00:11.0: reg 0x10: [io  0x2118-0x211f]
Sep  6 21:51:07 ubuntu kernel: [    2.424078] pci 0000:00:11.0: reg 0x14: [io  0x2124-0x2127]
Sep  6 21:51:07 ubuntu kernel: [    2.424091] pci 0000:00:11.0: reg 0x18: [io  0x2110-0x2117]
Sep  6 21:51:07 ubuntu kernel: [    2.424106] pci 0000:00:11.0: reg 0x1c: [io  0x2120-0x2123]
Sep  6 21:51:07 ubuntu kernel: [    2.424125] pci 0000:00:11.0: reg 0x20: [io  0x2100-0x210f]
Sep  6 21:51:07 ubuntu kernel: [    2.424138] pci 0000:00:11.0: reg 0x24: [mem 0xf094f000-0xf094f3ff]
Sep  6 21:51:07 ubuntu kernel: [    2.424193] pci 0000:00:11.0: PME# supported from D3hot
Sep  6 21:51:07 ubuntu kernel: [    2.424343] pci 0000:00:12.0: [1022:7807] type 00 class 0x0c0310
Sep  6 21:51:07 ubuntu kernel: [    2.424368] pci 0000:00:12.0: reg 0x10: [mem 0xf094e000-0xf094efff]
Sep  6 21:51:07 ubuntu kernel: [    2.424496] pci 0000:00:12.0: System wakeup disabled by ACPI
Sep  6 21:51:07 ubuntu kernel: [    2.424593] pci 0000:00:12.2: [1022:7808] type 00 class 0x0c0320
Sep  6 21:51:07 ubuntu kernel: [    2.424617] pci 0000:00:12.2: reg 0x10: [mem 0xf094d000-0xf094d0ff]
Sep  6 21:51:07 ubuntu kernel: [    2.424709] pci 0000:00:12.2: supports D1 D2
Sep  6 21:51:07 ubuntu kernel: [    2.424714] pci 0000:00:12.2: PME# supported from D0 D1 D2 D3hot D3cold
Sep  6 21:51:07 ubuntu kernel: [    2.424786] pci 0000:00:12.2: System wakeup disabled by ACPI
Sep  6 21:51:07 ubuntu kernel: [    2.424878] pci 0000:00:13.0: [1022:7807] type 00 class 0x0c0310
Sep  6 21:51:07 ubuntu kernel: [    2.424896] pci 0000:00:13.0: reg 0x10: [mem 0xf094c000-0xf094cfff]
Sep  6 21:51:07 ubuntu kernel: [    2.425026] pci 0000:00:13.0: System wakeup disabled by ACPI
Sep  6 21:51:07 ubuntu kernel: [    2.425118] pci 0000:00:13.2: [1022:7808] type 00 class 0x0c0320
Sep  6 21:51:07 ubuntu kernel: [    2.425141] pci 0000:00:13.2: reg 0x10: [mem 0xf094b000-0xf094b0ff]
Sep  6 21:51:07 ubuntu kernel: [    2.425234] pci 0000:00:13.2: supports D1 D2
Sep  6 21:51:07 ubuntu kernel: [    2.425238] pci 0000:00:13.2: PME# supported from D0 D1 D2 D3hot D3cold
Sep  6 21:51:07 ubuntu kernel: [    2.425310] pci 0000:00:13.2: System wakeup disabled by ACPI
Sep  6 21:51:07 ubuntu kernel: [    2.425402] pci 0000:00:14.0: [1022:780b] type 00 class 0x0c0500
Sep  6 21:51:07 ubuntu kernel: [    2.425606] pci 0000:00:14.2: [1022:780d] type 00 class 0x040300
Sep  6 21:51:07 ubuntu kernel: [    2.425634] pci 0000:00:14.2: reg 0x10: [mem 0xf0944000-0xf0947fff 64bit]
Sep  6 21:51:07 ubuntu kernel: [    2.425721] pci 0000:00:14.2: PME# supported from D0 D3hot D3cold
Sep  6 21:51:07 ubuntu kernel: [    2.425868] pci 0000:00:14.3: [1022:780e] type 00 class 0x060100
Sep  6 21:51:07 ubuntu kernel: [    2.426078] pci 0000:00:14.7: [1022:7813] type 00 class 0x080501
Sep  6 21:51:07 ubuntu kernel: [    2.426101] pci 0000:00:14.7: reg 0x10: [mem 0xf094a000-0xf094a0ff 64bit]
Sep  6 21:51:07 ubuntu kernel: [    2.426280] pci 0000:00:18.0: [1022:1530] type 00 class 0x060000
Sep  6 21:51:07 ubuntu kernel: [    2.426434] pci 0000:00:18.1: [1022:1531] type 00 class 0x060000
Sep  6 21:51:07 ubuntu kernel: [    2.426582] pci 0000:00:18.2: [1022:1532] type 00 class 0x060000
Sep  6 21:51:07 ubuntu kernel: [    2.426732] pci 0000:00:18.3: [1022:1533] type 00 class 0x060000
Sep  6 21:51:07 ubuntu kernel: [    2.426889] pci 0000:00:18.4: [1022:1534] type 00 class 0x060000
Sep  6 21:51:07 ubuntu kernel: [    2.427037] pci 0000:00:18.5: [1022:1535] type 00 class 0x060000
Sep  6 21:51:07 ubuntu kernel: [    2.427324] pci 0000:01:00.0: [168c:0036] type 00 class 0x028000
Sep  6 21:51:07 ubuntu kernel: [    2.427355] pci 0000:01:00.0: reg 0x10: [mem 0xf0800000-0xf087ffff 64bit]
Sep  6 21:51:07 ubuntu kernel: [    2.427413] pci 0000:01:00.0: reg 0x30: [mem 0xffff0000-0xffffffff pref]
Sep  6 21:51:07 ubuntu kernel: [    2.427499] pci 0000:01:00.0: supports D1 D2
Sep  6 21:51:07 ubuntu kernel: [    2.427504] pci 0000:01:00.0: PME# supported from D0 D1 D2 D3hot D3cold
Sep  6 21:51:07 ubuntu kernel: [    2.436551] pci 0000:00:02.3: PCI bridge to [bus 01]
Sep  6 21:51:07 ubuntu kernel: [    2.436575] pci 0000:00:02.3:   bridge window [mem 0xf0800000-0xf08fffff]
Sep  6 21:51:07 ubuntu kernel: [    2.438876] ACPI: PCI Interrupt Link [LNKA] (IRQs 3 4 5 7 10 11 12 14 15) *0
Sep  6 21:51:07 ubuntu kernel: [    2.439064] ACPI: PCI Interrupt Link [LNKB] (IRQs 3 4 5 7 10 11 12 14 15) *0
Sep  6 21:51:07 ubuntu kernel: [    2.439253] ACPI: PCI Interrupt Link [LNKC] (IRQs 3 4 5 7 10 11 12 14 15) *0
Sep  6 21:51:07 ubuntu kernel: [    2.439440] ACPI: PCI Interrupt Link [LNKD] (IRQs 3 4 5 7 10 11 12 14 15) *0
Sep  6 21:51:07 ubuntu kernel: [    2.439601] ACPI: PCI Interrupt Link [LNKE] (IRQs 3 4 5 7 10 11 12 14 15) *0
Sep  6 21:51:07 ubuntu kernel: [    2.439722] ACPI: PCI Interrupt Link [LNKF] (IRQs 3 4 5 7 10 11 12 14 15) *0
Sep  6 21:51:07 ubuntu kernel: [    2.439842] ACPI: PCI Interrupt Link [LNKG] (IRQs 3 4 5 7 10 11 12 14 15) *0
Sep  6 21:51:07 ubuntu kernel: [    2.439962] ACPI: PCI Interrupt Link [LNKH] (IRQs 3 4 5 7 10 11 12 14 15) *0
Sep  6 21:51:07 ubuntu kernel: [    2.440553] ACPI : EC: GPE = 0x3, I/O: command/status = 0x66, data = 0x62
Sep  6 21:51:07 ubuntu kernel: [    2.440954] vgaarb: setting as boot device: PCI:0000:00:01.0
Sep  6 21:51:07 ubuntu kernel: [    2.440961] vgaarb: device added: PCI:0000:00:01.0,decodes=io+mem,owns=io+mem,locks=none
Sep  6 21:51:07 ubuntu kernel: [    2.440971] vgaarb: loaded
Sep  6 21:51:07 ubuntu kernel: [    2.440975] vgaarb: bridge control possible 0000:00:01.0
Sep  6 21:51:07 ubuntu kernel: [    2.441661] SCSI subsystem initialized
Sep  6 21:51:07 ubuntu kernel: [    2.441870] libata version 3.00 loaded.
Sep  6 21:51:07 ubuntu kernel: [    2.441949] ACPI: bus type USB registered
Sep  6 21:51:07 ubuntu kernel: [    2.442006] usbcore: registered new interface driver usbfs
Sep  6 21:51:07 ubuntu kernel: [    2.442033] usbcore: registered new interface driver hub
Sep  6 21:51:07 ubuntu kernel: [    2.442081] usbcore: registered new device driver usb
Sep  6 21:51:07 ubuntu kernel: [    2.442603] PCI: Using ACPI for IRQ routing
Sep  6 21:51:07 ubuntu kernel: [    2.444780] PCI: pci_cache_line_size set to 64 bytes
Sep  6 21:51:07 ubuntu kernel: [    2.444876] e820: reserve RAM buffer [mem 0x0006e000-0x0006ffff]
Sep  6 21:51:07 ubuntu kernel: [    2.444883] e820: reserve RAM buffer [mem 0x00086000-0x0008ffff]
Sep  6 21:51:07 ubuntu kernel: [    2.444887] e820: reserve RAM buffer [mem 0xb4b1d000-0xb7ffffff]
Sep  6 21:51:07 ubuntu kernel: [    2.444892] e820: reserve RAM buffer [mem 0xb537c000-0xb7ffffff]
Sep  6 21:51:07 ubuntu kernel: [    2.444897] e820: reserve RAM buffer [mem 0xbf17f000-0xbfffffff]
Sep  6 21:51:07 ubuntu kernel: [    2.444902] e820: reserve RAM buffer [mem 0xbfc00000-0xbfffffff]
Sep  6 21:51:07 ubuntu kernel: [    2.444906] e820: reserve RAM buffer [mem 0x19f000000-0x19fffffff]
Sep  6 21:51:07 ubuntu kernel: [    2.445275] NetLabel: Initializing
Sep  6 21:51:07 ubuntu kernel: [    2.445281] NetLabel:  domain hash size = 128
Sep  6 21:51:07 ubuntu kernel: [    2.445284] NetLabel:  protocols = UNLABELED CIPSOv4
Sep  6 21:51:07 ubuntu kernel: [    2.445322] NetLabel:  unlabeled traffic allowed by default
Sep  6 21:51:07 ubuntu kernel: [    2.445557] hpet0: at MMIO 0xfed00000, IRQs 2, 8, 0
Sep  6 21:51:07 ubuntu kernel: [    2.445568] hpet0: 3 comparators, 32-bit 14.318180 MHz counter
Sep  6 21:51:07 ubuntu kernel: [    2.447745] Switched to clocksource hpet
Sep  6 21:51:07 ubuntu kernel: [    2.465272] AppArmor: AppArmor Filesystem Enabled
Sep  6 21:51:07 ubuntu kernel: [    2.465478] pnp: PnP ACPI init
Sep  6 21:51:07 ubuntu kernel: [    2.465834] system 00:00: [mem 0xfec00000-0xfec01fff] could not be reserved
Sep  6 21:51:07 ubuntu kernel: [    2.465841] system 00:00: [mem 0xfee00000-0xfee00fff] has been reserved
Sep  6 21:51:07 ubuntu kernel: [    2.465851] system 00:00: Plug and Play ACPI device, IDs PNP0c02 (active)
Sep  6 21:51:07 ubuntu kernel: [    2.466140] pnp 00:01: Plug and Play ACPI device, IDs PNP0b00 (active)
Sep  6 21:51:07 ubuntu kernel: [    2.466256] system 00:02: [io  0x0400-0x04cf] could not be reserved
Sep  6 21:51:07 ubuntu kernel: [    2.466263] system 00:02: [io  0x04d0-0x04d1] has been reserved
Sep  6 21:51:07 ubuntu kernel: [    2.466269] system 00:02: [io  0x04d6] has been reserved
Sep  6 21:51:07 ubuntu kernel: [    2.466274] system 00:02: [io  0x0680-0x06ff] has been reserved
Sep  6 21:51:07 ubuntu kernel: [    2.466279] system 00:02: [io  0x077a] has been reserved
Sep  6 21:51:07 ubuntu kernel: [    2.466284] system 00:02: [io  0x0c00-0x0c01] has been reserved
Sep  6 21:51:07 ubuntu kernel: [    2.466289] system 00:02: [io  0x0c14] has been reserved
Sep  6 21:51:07 ubuntu kernel: [    2.466294] system 00:02: [io  0x0c50-0x0c52] has been reserved
Sep  6 21:51:07 ubuntu kernel: [    2.466299] system 00:02: [io  0x0c6c] has been reserved
Sep  6 21:51:07 ubuntu kernel: [    2.466304] system 00:02: [io  0x0c6f] has been reserved
Sep  6 21:51:07 ubuntu kernel: [    2.466309] system 00:02: [io  0x0cd0-0x0cdb] has been reserved
Sep  6 21:51:07 ubuntu kernel: [    2.466314] system 00:02: [io  0x0840-0x0847] has been reserved
Sep  6 21:51:07 ubuntu kernel: [    2.466321] system 00:02: Plug and Play ACPI device, IDs PNP0c02 (active)
Sep  6 21:51:07 ubuntu kernel: [    2.466486] system 00:03: [mem 0x000e0000-0x000fffff] could not be reserved
Sep  6 21:51:07 ubuntu kernel: [    2.466492] system 00:03: [mem 0xffc00000-0xffffffff] has been reserved
Sep  6 21:51:07 ubuntu kernel: [    2.466498] system 00:03: Plug and Play ACPI device, IDs PNP0c01 (active)
Sep  6 21:51:07 ubuntu kernel: [    2.466725] pnp 00:04: Plug and Play ACPI device, IDs PNP0303 (active)
Sep  6 21:51:07 ubuntu kernel: [    2.466832] pnp 00:05: Plug and Play ACPI device, IDs ETD050a PNP0f13 (active)
Sep  6 21:51:07 ubuntu kernel: [    2.467793] pnp: PnP ACPI: found 6 devices
Sep  6 21:51:07 ubuntu kernel: [    2.477526] pci 0000:00:01.0: can't claim BAR 6 [mem 0xfffe0000-0xffffffff pref]: address conflict with reserved [mem 0xffc00000-0xffffffff]
Sep  6 21:51:07 ubuntu kernel: [    2.477535] pci 0000:01:00.0: can't claim BAR 6 [mem 0xffff0000-0xffffffff pref]: no compatible bridge window
Sep  6 21:51:07 ubuntu kernel: [    2.477583] pci 0000:00:01.0: BAR 6: assigned [mem 0xf0960000-0xf097ffff pref]
Sep  6 21:51:07 ubuntu kernel: [    2.477594] pci 0000:01:00.0: BAR 6: assigned [mem 0xf0880000-0xf088ffff pref]
Sep  6 21:51:07 ubuntu kernel: [    2.477601] pci 0000:00:02.3: PCI bridge to [bus 01]
Sep  6 21:51:07 ubuntu kernel: [    2.477611] pci 0000:00:02.3:   bridge window [mem 0xf0800000-0xf08fffff]
Sep  6 21:51:07 ubuntu kernel: [    2.477626] pci_bus 0000:00: resource 4 [io  0x0000-0x0cf7]
Sep  6 21:51:07 ubuntu kernel: [    2.477631] pci_bus 0000:00: resource 5 [io  0x0d00-0xffff]
Sep  6 21:51:07 ubuntu kernel: [    2.477636] pci_bus 0000:00: resource 6 [mem 0x000a0000-0x000bffff]
Sep  6 21:51:07 ubuntu kernel: [    2.477642] pci_bus 0000:00: resource 7 [mem 0x000c0000-0x000c3fff]
Sep  6 21:51:07 ubuntu kernel: [    2.477647] pci_bus 0000:00: resource 8 [mem 0x000c4000-0x000c7fff]
Sep  6 21:51:07 ubuntu kernel: [    2.477652] pci_bus 0000:00: resource 9 [mem 0x000c8000-0x000cbfff]
Sep  6 21:51:07 ubuntu kernel: [    2.477657] pci_bus 0000:00: resource 10 [mem 0x000cc000-0x000cffff]
Sep  6 21:51:07 ubuntu kernel: [    2.477662] pci_bus 0000:00: resource 11 [mem 0x000d0000-0x000d3fff]
Sep  6 21:51:07 ubuntu kernel: [    2.477667] pci_bus 0000:00: resource 12 [mem 0x000d4000-0x000d7fff]
Sep  6 21:51:07 ubuntu kernel: [    2.477672] pci_bus 0000:00: resource 13 [mem 0x000d8000-0x000dbfff]
Sep  6 21:51:07 ubuntu kernel: [    2.477677] pci_bus 0000:00: resource 14 [mem 0x000dc000-0x000dffff]
Sep  6 21:51:07 ubuntu kernel: [    2.477681] pci_bus 0000:00: resource 15 [mem 0x000e0000-0x000e3fff]
Sep  6 21:51:07 ubuntu kernel: [    2.477686] pci_bus 0000:00: resource 16 [mem 0x000e4000-0x000e7fff]
Sep  6 21:51:07 ubuntu kernel: [    2.477691] pci_bus 0000:00: resource 17 [mem 0x000e8000-0x000ebfff]
Sep  6 21:51:07 ubuntu kernel: [    2.477696] pci_bus 0000:00: resource 18 [mem 0x000ec000-0x000effff]
Sep  6 21:51:07 ubuntu kernel: [    2.477701] pci_bus 0000:00: resource 19 [mem 0xe0000000-0xf7ffffff]
Sep  6 21:51:07 ubuntu kernel: [    2.477706] pci_bus 0000:00: resource 20 [mem 0xfc000000-0xfed3ffff]
Sep  6 21:51:07 ubuntu kernel: [    2.477710] pci_bus 0000:00: resource 21 [mem 0xfed45000-0xffffffff]
Sep  6 21:51:07 ubuntu kernel: [    2.477716] pci_bus 0000:01: resource 1 [mem 0xf0800000-0xf08fffff]
Sep  6 21:51:07 ubuntu kernel: [    2.477787] NET: Registered protocol family 2
Sep  6 21:51:07 ubuntu kernel: [    2.478310] TCP established hash table entries: 65536 (order: 7, 524288 bytes)
Sep  6 21:51:07 ubuntu kernel: [    2.478758] TCP bind hash table entries: 65536 (order: 8, 1048576 bytes)
Sep  6 21:51:07 ubuntu kernel: [    2.479173] TCP: Hash tables configured (established 65536 bind 65536)
Sep  6 21:51:07 ubuntu kernel: [    2.479247] TCP: reno registered
Sep  6 21:51:07 ubuntu kernel: [    2.479278] UDP hash table entries: 4096 (order: 5, 131072 bytes)
Sep  6 21:51:07 ubuntu kernel: [    2.479367] UDP-Lite hash table entries: 4096 (order: 5, 131072 bytes)
Sep  6 21:51:07 ubuntu kernel: [    2.479589] NET: Registered protocol family 1
Sep  6 21:51:07 ubuntu kernel: [    2.479639] pci 0000:00:01.0: Video device with shadowed ROM
Sep  6 21:51:07 ubuntu kernel: [    2.624489] PCI: CLS 64 bytes, default 64
Sep  6 21:51:07 ubuntu kernel: [    2.624659] Trying to unpack rootfs image as initramfs...
Sep  6 21:51:07 ubuntu kernel: [   13.263239] Freeing initrd memory: 21192K (ffff88003568c000 - ffff880036b3e000)
Sep  6 21:51:07 ubuntu kernel: [   13.263349] PCI-DMA: Using software bounce buffering for IO (SWIOTLB)
Sep  6 21:51:07 ubuntu kernel: [   13.263359] software IO TLB [mem 0xb7f63000-0xbbf63000] (64MB) mapped at [ffff8800b7f63000-ffff8800bbf62fff]
Sep  6 21:51:07 ubuntu kernel: [   13.263473] Simple Boot Flag at 0x44 set to 0x1
Sep  6 21:51:07 ubuntu kernel: [   13.263909] perf: AMD NB counters detected
Sep  6 21:51:07 ubuntu kernel: [   13.263915] perf: AMD L2I counters detected
Sep  6 21:51:07 ubuntu kernel: [   13.264138] microcode: CPU0: patch_level=0x07000106
Sep  6 21:51:07 ubuntu kernel: [   13.264156] microcode: CPU1: patch_level=0x07000106
Sep  6 21:51:07 ubuntu kernel: [   13.264174] microcode: CPU2: patch_level=0x07000106
Sep  6 21:51:07 ubuntu kernel: [   13.264187] microcode: CPU3: patch_level=0x07000106
Sep  6 21:51:07 ubuntu kernel: [   13.264382] microcode: Microcode Update Driver: v2.00 <tigran@aivazian.fsnet.co.uk>, Peter Oruba
Sep  6 21:51:07 ubuntu kernel: [   13.264395] LVT offset 0 assigned for vector 0x400
Sep  6 21:51:07 ubuntu kernel: [   13.264444] perf: AMD IBS detected (0x000000ff)
Sep  6 21:51:07 ubuntu kernel: [   13.264507] Scanning for low memory corruption every 60 seconds
Sep  6 21:51:07 ubuntu kernel: [   13.265465] futex hash table entries: 1024 (order: 4, 65536 bytes)
Sep  6 21:51:07 ubuntu kernel: [   13.265511] Initialise system trusted keyring
Sep  6 21:51:07 ubuntu kernel: [   13.265565] audit: initializing netlink subsys (disabled)
Sep  6 21:51:07 ubuntu kernel: [   13.265606] audit: type=2000 audit(1441576235.128:1): initialized
Sep  6 21:51:07 ubuntu kernel: [   13.266533] HugeTLB registered 2 MB page size, pre-allocated 0 pages
Sep  6 21:51:07 ubuntu kernel: [   13.270946] zpool: loaded
Sep  6 21:51:07 ubuntu kernel: [   13.270954] zbud: loaded
Sep  6 21:51:07 ubuntu kernel: [   13.271380] VFS: Disk quotas dquot_6.5.2
Sep  6 21:51:07 ubuntu kernel: [   13.271478] VFS: Dquot-cache hash table entries: 512 (order 0, 4096 bytes)
Sep  6 21:51:07 ubuntu kernel: [   13.272836] fuse init (API version 7.23)
Sep  6 21:51:07 ubuntu kernel: [   13.273220] Key type big_key registered
Sep  6 21:51:07 ubuntu kernel: [   13.274347] Key type asymmetric registered
Sep  6 21:51:07 ubuntu kernel: [   13.274358] Asymmetric key parser 'x509' registered
Sep  6 21:51:07 ubuntu kernel: [   13.274483] Block layer SCSI generic (bsg) driver version 0.4 loaded (major 252)
Sep  6 21:51:07 ubuntu kernel: [   13.274608] io scheduler noop registered
Sep  6 21:51:07 ubuntu kernel: [   13.274620] io scheduler deadline registered (default)
Sep  6 21:51:07 ubuntu kernel: [   13.274731] io scheduler cfq registered
Sep  6 21:51:07 ubuntu kernel: [   13.275354] pci_hotplug: PCI Hot Plug PCI Core version: 0.5
Sep  6 21:51:07 ubuntu kernel: [   13.275407] pciehp: PCI Express Hot Plug Controller Driver version: 0.4
Sep  6 21:51:07 ubuntu kernel: [   13.275530] efifb: probing for efifb
Sep  6 21:51:07 ubuntu kernel: [   13.275581] efifb: framebuffer at 0xe0000000, mapped to 0xffffc90004f00000, using 4224k, total 4224k
Sep  6 21:51:07 ubuntu kernel: [   13.275587] efifb: mode is 1366x768x32, linelength=5632, pages=1
Sep  6 21:51:07 ubuntu kernel: [   13.275590] efifb: scrolling: redraw
Sep  6 21:51:07 ubuntu kernel: [   13.275596] efifb: Truecolor: size=8:8:8:8, shift=24:16:8:0
Sep  6 21:51:07 ubuntu kernel: [   13.285610] Console: switching to colour frame buffer device 170x48
Sep  6 21:51:07 ubuntu kernel: [   13.293962] fb0: EFI VGA frame buffer device
Sep  6 21:51:07 ubuntu kernel: [   13.294707] ACPI: AC Adapter [ADP1] (on-line)
Sep  6 21:51:07 ubuntu kernel: [   13.295355] input: Power Button as /devices/LNXSYSTM:00/LNXSYBUS:00/PNP0C0C:00/input/input0
Sep  6 21:51:07 ubuntu kernel: [   13.295364] ACPI: Power Button [PWRB]
Sep  6 21:51:07 ubuntu kernel: [   13.295482] input: Lid Switch as /devices/LNXSYSTM:00/LNXSYBUS:00/PNP0C0D:00/input/input1
Sep  6 21:51:07 ubuntu kernel: [   13.295529] ACPI: Lid Switch [LID0]
Sep  6 21:51:07 ubuntu kernel: [   13.295625] input: Sleep Button as /devices/LNXSYSTM:00/LNXSYBUS:00/PNP0C0E:00/input/input2
Sep  6 21:51:07 ubuntu kernel: [   13.295631] ACPI: Sleep Button [SLPB]
Sep  6 21:51:07 ubuntu kernel: [   13.295725] input: Power Button as /devices/LNXSYSTM:00/LNXPWRBN:00/input/input3
Sep  6 21:51:07 ubuntu kernel: [   13.295731] ACPI: Power Button [PWRF]
Sep  6 21:51:07 ubuntu kernel: [   13.295832] ACPI: acpi_idle registered with cpuidle
Sep  6 21:51:07 ubuntu kernel: [   13.301084] thermal LNXTHERM:00: registered as thermal_zone0
Sep  6 21:51:07 ubuntu kernel: [   13.301093] ACPI: Thermal Zone [TZS0] (67 C)
Sep  6 21:51:07 ubuntu kernel: [   13.301583] thermal LNXTHERM:01: registered as thermal_zone1
Sep  6 21:51:07 ubuntu kernel: [   13.301587] ACPI: Thermal Zone [TZS1] (44 C)
Sep  6 21:51:07 ubuntu kernel: [   13.301693] GHES: HEST is not enabled!
Sep  6 21:51:07 ubuntu kernel: [   13.301990] Serial: 8250/16550 driver, 32 ports, IRQ sharing enabled
Sep  6 21:51:07 ubuntu kernel: [   13.302911] ACPI: Battery Slot [BAT0] (battery present)
Sep  6 21:51:07 ubuntu kernel: [   13.306383] Linux agpgart interface v0.103
Sep  6 21:51:07 ubuntu kernel: [   13.310338] brd: module loaded
Sep  6 21:51:07 ubuntu kernel: [   13.312283] loop: module loaded
Sep  6 21:51:07 ubuntu kernel: [   13.312904] libphy: Fixed MDIO Bus: probed
Sep  6 21:51:07 ubuntu kernel: [   13.312914] tun: Universal TUN/TAP device driver, 1.6
Sep  6 21:51:07 ubuntu kernel: [   13.312918] tun: (C) 1999-2004 Max Krasnyansky <maxk@qualcomm.com>
Sep  6 21:51:07 ubuntu kernel: [   13.313049] PPP generic driver version 2.4.2
Sep  6 21:51:07 ubuntu kernel: [   13.313505] xhci_hcd 0000:00:10.0: xHCI Host Controller
Sep  6 21:51:07 ubuntu kernel: [   13.313523] xhci_hcd 0000:00:10.0: new USB bus registered, assigned bus number 1
Sep  6 21:51:07 ubuntu kernel: [   13.314095] usb usb1: New USB device found, idVendor=1d6b, idProduct=0002
Sep  6 21:51:07 ubuntu kernel: [   13.314104] usb usb1: New USB device strings: Mfr=3, Product=2, SerialNumber=1
Sep  6 21:51:07 ubuntu kernel: [   13.314110] usb usb1: Product: xHCI Host Controller
Sep  6 21:51:07 ubuntu kernel: [   13.314116] usb usb1: Manufacturer: Linux 3.19.0-25-generic xhci-hcd
Sep  6 21:51:07 ubuntu kernel: [   13.314122] usb usb1: SerialNumber: 0000:00:10.0
Sep  6 21:51:07 ubuntu kernel: [   13.314516] hub 1-0:1.0: USB hub found
Sep  6 21:51:07 ubuntu kernel: [   13.314545] hub 1-0:1.0: 2 ports detected
Sep  6 21:51:07 ubuntu kernel: [   13.314826] xhci_hcd 0000:00:10.0: xHCI Host Controller
Sep  6 21:51:07 ubuntu kernel: [   13.314837] xhci_hcd 0000:00:10.0: new USB bus registered, assigned bus number 2
Sep  6 21:51:07 ubuntu kernel: [   13.317208] usb usb2: New USB device found, idVendor=1d6b, idProduct=0003
Sep  6 21:51:07 ubuntu kernel: [   13.317215] usb usb2: New USB device strings: Mfr=3, Product=2, SerialNumber=1
Sep  6 21:51:07 ubuntu kernel: [   13.317221] usb usb2: Product: xHCI Host Controller
Sep  6 21:51:07 ubuntu kernel: [   13.317227] usb usb2: Manufacturer: Linux 3.19.0-25-generic xhci-hcd
Sep  6 21:51:07 ubuntu kernel: [   13.317233] usb usb2: SerialNumber: 0000:00:10.0
Sep  6 21:51:07 ubuntu kernel: [   13.317553] hub 2-0:1.0: USB hub found
Sep  6 21:51:07 ubuntu kernel: [   13.317580] hub 2-0:1.0: 2 ports detected
Sep  6 21:51:07 ubuntu kernel: [   13.317873] ehci_hcd: USB 2.0 'Enhanced' Host Controller (EHCI) Driver
Sep  6 21:51:07 ubuntu kernel: [   13.317892] ehci-pci: EHCI PCI platform driver
Sep  6 21:51:07 ubuntu kernel: [   13.318118] ehci-pci 0000:00:12.2: EHCI Host Controller
Sep  6 21:51:07 ubuntu kernel: [   13.318135] ehci-pci 0000:00:12.2: new USB bus registered, assigned bus number 3
Sep  6 21:51:07 ubuntu kernel: [   13.318147] ehci-pci 0000:00:12.2: applying AMD SB700/SB800/Hudson-2/3 EHCI dummy qh workaround
Sep  6 21:51:07 ubuntu kernel: [   13.318164] ehci-pci 0000:00:12.2: debug port 1
Sep  6 21:51:07 ubuntu kernel: [   13.318240] ehci-pci 0000:00:12.2: irq 17, io mem 0xf094d000
Sep  6 21:51:07 ubuntu kernel: [   13.328803] ehci-pci 0000:00:12.2: USB 2.0 started, EHCI 1.00
Sep  6 21:51:07 ubuntu kernel: [   13.329015] usb usb3: New USB device found, idVendor=1d6b, idProduct=0002
Sep  6 21:51:07 ubuntu kernel: [   13.329024] usb usb3: New USB device strings: Mfr=3, Product=2, SerialNumber=1
Sep  6 21:51:07 ubuntu kernel: [   13.329032] usb usb3: Product: EHCI Host Controller
Sep  6 21:51:07 ubuntu kernel: [   13.329039] usb usb3: Manufacturer: Linux 3.19.0-25-generic ehci_hcd
Sep  6 21:51:07 ubuntu kernel: [   13.329046] usb usb3: SerialNumber: 0000:00:12.2
Sep  6 21:51:07 ubuntu kernel: [   13.329516] hub 3-0:1.0: USB hub found
Sep  6 21:51:07 ubuntu kernel: [   13.329546] hub 3-0:1.0: 4 ports detected
Sep  6 21:51:07 ubuntu kernel: [   13.330280] ehci-pci 0000:00:13.2: EHCI Host Controller
Sep  6 21:51:07 ubuntu kernel: [   13.330302] ehci-pci 0000:00:13.2: new USB bus registered, assigned bus number 4
Sep  6 21:51:07 ubuntu kernel: [   13.330315] ehci-pci 0000:00:13.2: applying AMD SB700/SB800/Hudson-2/3 EHCI dummy qh workaround
Sep  6 21:51:07 ubuntu kernel: [   13.330334] ehci-pci 0000:00:13.2: debug port 1
Sep  6 21:51:07 ubuntu kernel: [   13.330405] ehci-pci 0000:00:13.2: irq 17, io mem 0xf094b000
Sep  6 21:51:07 ubuntu kernel: [   13.340800] ehci-pci 0000:00:13.2: USB 2.0 started, EHCI 1.00
Sep  6 21:51:07 ubuntu kernel: [   13.341040] usb usb4: New USB device found, idVendor=1d6b, idProduct=0002
Sep  6 21:51:07 ubuntu kernel: [   13.341049] usb usb4: New USB device strings: Mfr=3, Product=2, SerialNumber=1
Sep  6 21:51:07 ubuntu kernel: [   13.341056] usb usb4: Product: EHCI Host Controller
Sep  6 21:51:07 ubuntu kernel: [   13.341062] usb usb4: Manufacturer: Linux 3.19.0-25-generic ehci_hcd
Sep  6 21:51:07 ubuntu kernel: [   13.341068] usb usb4: SerialNumber: 0000:00:13.2
Sep  6 21:51:07 ubuntu kernel: [   13.341596] hub 4-0:1.0: USB hub found
Sep  6 21:51:07 ubuntu kernel: [   13.341623] hub 4-0:1.0: 4 ports detected
Sep  6 21:51:07 ubuntu kernel: [   13.342130] ehci-platform: EHCI generic platform driver
Sep  6 21:51:07 ubuntu kernel: [   13.342183] ohci_hcd: USB 1.1 'Open' Host Controller (OHCI) Driver
Sep  6 21:51:07 ubuntu kernel: [   13.342209] ohci-pci: OHCI PCI platform driver
Sep  6 21:51:07 ubuntu kernel: [   13.342488] ohci-pci 0000:00:12.0: OHCI PCI host controller
Sep  6 21:51:07 ubuntu kernel: [   13.342513] ohci-pci 0000:00:12.0: new USB bus registered, assigned bus number 5
Sep  6 21:51:07 ubuntu kernel: [   13.342584] ohci-pci 0000:00:12.0: irq 18, io mem 0xf094e000
Sep  6 21:51:07 ubuntu kernel: [   13.400902] usb usb5: New USB device found, idVendor=1d6b, idProduct=0001
Sep  6 21:51:07 ubuntu kernel: [   13.400913] usb usb5: New USB device strings: Mfr=3, Product=2, SerialNumber=1
Sep  6 21:51:07 ubuntu kernel: [   13.400918] usb usb5: Product: OHCI PCI host controller
Sep  6 21:51:07 ubuntu kernel: [   13.400922] usb usb5: Manufacturer: Linux 3.19.0-25-generic ohci_hcd
Sep  6 21:51:07 ubuntu kernel: [   13.400927] usb usb5: SerialNumber: 0000:00:12.0
Sep  6 21:51:07 ubuntu kernel: [   13.401356] hub 5-0:1.0: USB hub found
Sep  6 21:51:07 ubuntu kernel: [   13.401378] hub 5-0:1.0: 4 ports detected
Sep  6 21:51:07 ubuntu kernel: [   13.401947] ohci-pci 0000:00:13.0: OHCI PCI host controller
Sep  6 21:51:07 ubuntu kernel: [   13.401969] ohci-pci 0000:00:13.0: new USB bus registered, assigned bus number 6
Sep  6 21:51:07 ubuntu kernel: [   13.402022] ohci-pci 0000:00:13.0: irq 18, io mem 0xf094c000
Sep  6 21:51:07 ubuntu kernel: [   13.460905] usb usb6: New USB device found, idVendor=1d6b, idProduct=0001
Sep  6 21:51:07 ubuntu kernel: [   13.460915] usb usb6: New USB device strings: Mfr=3, Product=2, SerialNumber=1
Sep  6 21:51:07 ubuntu kernel: [   13.460921] usb usb6: Product: OHCI PCI host controller
Sep  6 21:51:07 ubuntu kernel: [   13.460925] usb usb6: Manufacturer: Linux 3.19.0-25-generic ohci_hcd
Sep  6 21:51:07 ubuntu kernel: [   13.460930] usb usb6: SerialNumber: 0000:00:13.0
Sep  6 21:51:07 ubuntu kernel: [   13.461388] hub 6-0:1.0: USB hub found
Sep  6 21:51:07 ubuntu kernel: [   13.461410] hub 6-0:1.0: 4 ports detected
Sep  6 21:51:07 ubuntu kernel: [   13.461855] ohci-platform: OHCI generic platform driver
Sep  6 21:51:07 ubuntu kernel: [   13.461902] uhci_hcd: USB Universal Host Controller Interface driver
Sep  6 21:51:07 ubuntu kernel: [   13.462081] i8042: PNP: PS/2 Controller [PNP0303:KBD0,PNP0f13:PS2M] at 0x60,0x64 irq 1,12
Sep  6 21:51:07 ubuntu kernel: [   13.469407] serio: i8042 KBD port at 0x60,0x64 irq 1
Sep  6 21:51:07 ubuntu kernel: [   13.469421] serio: i8042 AUX port at 0x60,0x64 irq 12
Sep  6 21:51:07 ubuntu kernel: [   13.469807] mousedev: PS/2 mouse device common for all mice
Sep  6 21:51:07 ubuntu kernel: [   13.470360] rtc_cmos 00:01: RTC can wake from S4
Sep  6 21:51:07 ubuntu kernel: [   13.470560] rtc_cmos 00:01: rtc core: registered rtc_cmos as rtc0
Sep  6 21:51:07 ubuntu kernel: [   13.470601] rtc_cmos 00:01: alarms up to one month, 114 bytes nvram, hpet irqs
Sep  6 21:51:07 ubuntu kernel: [   13.470646] i2c /dev entries driver
Sep  6 21:51:07 ubuntu kernel: [   13.470842] device-mapper: uevent: version 1.0.3
Sep  6 21:51:07 ubuntu kernel: [   13.471033] device-mapper: ioctl: 4.29.0-ioctl (2014-10-28) initialised: dm-devel@redhat.com
Sep  6 21:51:07 ubuntu kernel: [   13.471077] ledtrig-cpu: registered to indicate activity on CPUs
Sep  6 21:51:07 ubuntu kernel: [   13.471099] EFI Variables Facility v0.08 2004-May-17
Sep  6 21:51:07 ubuntu kernel: [   13.483200] input: AT Translated Set 2 keyboard as /devices/platform/i8042/serio0/input/input4
Sep  6 21:51:07 ubuntu kernel: [   13.522880] PCCT header not found.
Sep  6 21:51:07 ubuntu kernel: [   13.522889] ACPI PCC probe failed.
Sep  6 21:51:07 ubuntu kernel: [   13.523193] TCP: cubic registered
Sep  6 21:51:07 ubuntu kernel: [   13.523475] NET: Registered protocol family 10
Sep  6 21:51:07 ubuntu kernel: [   13.524043] NET: Registered protocol family 17
Sep  6 21:51:07 ubuntu kernel: [   13.524072] Key type dns_resolver registered
Sep  6 21:51:07 ubuntu kernel: [   13.525058] Loading compiled-in X.509 certificates
Sep  6 21:51:07 ubuntu kernel: [   13.527858] Loaded X.509 cert 'Magrathea: Glacier signing key: 6aaa11d18c2d3a40b1b4dbe5bf8ad656ddf51838'
Sep  6 21:51:07 ubuntu kernel: [   13.527902] registered taskstats version 1
Sep  6 21:51:07 ubuntu kernel: [   13.533091] Key type trusted registered
Sep  6 21:51:07 ubuntu kernel: [   13.543360] Key type encrypted registered
Sep  6 21:51:07 ubuntu kernel: [   13.543381] AppArmor: AppArmor sha1 policy hashing enabled
Sep  6 21:51:07 ubuntu kernel: [   13.543395] ima: No TPM chip found, activating TPM-bypass!
Sep  6 21:51:07 ubuntu kernel: [   13.543462] evm: HMAC attrs: 0x1
Sep  6 21:51:07 ubuntu kernel: [   13.544342]   Magic number: 15:484:853
Sep  6 21:51:07 ubuntu kernel: [   13.544640] rtc_cmos 00:01: setting system clock to 2015-09-06 21:50:37 UTC (1441576237)
Sep  6 21:51:07 ubuntu kernel: [   13.545049] acpi-cpufreq: overriding BIOS provided _PSD data
Sep  6 21:51:07 ubuntu kernel: [   13.545519] BIOS EDD facility v0.16 2004-Jun-25, 0 devices found
Sep  6 21:51:07 ubuntu kernel: [   13.545524] EDD information not available.
Sep  6 21:51:07 ubuntu kernel: [   13.545756] PM: Hibernation image not present or could not be loaded.
Sep  6 21:51:07 ubuntu kernel: [   13.546669] Freeing unused kernel memory: 1408K (ffffffff81d27000 - ffffffff81e87000)
Sep  6 21:51:07 ubuntu kernel: [   13.546676] Write protecting the kernel read-only data: 12288k
Sep  6 21:51:07 ubuntu kernel: [   13.547293] Freeing unused kernel memory: 260K (ffff8800027bf000 - ffff880002800000)
Sep  6 21:51:07 ubuntu kernel: [   13.547579] Freeing unused kernel memory: 344K (ffff880002baa000 - ffff880002c00000)
Sep  6 21:51:07 ubuntu kernel: [   13.579517] systemd-udevd[122]: starting version 204
Sep  6 21:51:07 ubuntu kernel: [   13.610202] ACPI: Video Device [VGA] (multi-head: yes  rom: no  post: no)
Sep  6 21:51:07 ubuntu kernel: [   13.631900] acpi device:00: registered as cooling_device4
Sep  6 21:51:07 ubuntu kernel: [   13.632112] input: Video Bus as /devices/LNXSYSTM:00/LNXSYBUS:00/PNP0A08:00/LNXVIDEO:00/input/input7
Sep  6 21:51:07 ubuntu kernel: [   13.634421] wmi: Mapper loaded
Sep  6 21:51:07 ubuntu kernel: [   13.654005] ahci 0000:00:11.0: version 3.0
Sep  6 21:51:07 ubuntu kernel: [   13.654483] ahci 0000:00:11.0: AHCI 0001.0300 32 slots 1 ports 6 Gbps 0x1 impl SATA mode
Sep  6 21:51:07 ubuntu kernel: [   13.654500] ahci 0000:00:11.0: flags: 64bit ncq sntf ilck pm led clo pmp fbs pio slum part
Sep  6 21:51:07 ubuntu kernel: [   13.655366] usb 4-1: new high-speed USB device number 2 using ehci-pci
Sep  6 21:51:07 ubuntu kernel: [   13.657136] sdhci: Secure Digital Host Controller Interface driver
Sep  6 21:51:07 ubuntu kernel: [   13.657145] sdhci: Copyright(c) Pierre Ossman
Sep  6 21:51:07 ubuntu kernel: [   13.658104] usb 3-1: new high-speed USB device number 2 using ehci-pci
Sep  6 21:51:07 ubuntu kernel: [   13.660271] scsi host0: ahci
Sep  6 21:51:07 ubuntu kernel: [   13.660698] ata1: SATA max UDMA/133 abar m1024@0xf094f000 port 0xf094f100 irq 19
Sep  6 21:51:07 ubuntu kernel: [   13.666129] [drm] Initialized drm 1.1.0 20060810
Sep  6 21:51:07 ubuntu kernel: [   13.666316] sdhci-pci 0000:00:14.7: SDHCI controller found [1022:7813] (rev 1)
Sep  6 21:51:07 ubuntu kernel: [   13.666791] sdhci-pci 0000:00:14.7: No vmmc regulator found
Sep  6 21:51:07 ubuntu kernel: [   13.666802] sdhci-pci 0000:00:14.7: No vqmmc regulator found
Sep  6 21:51:07 ubuntu kernel: [   13.669378] mmc0: SDHCI controller on PCI [0000:00:14.7] using ADMA
Sep  6 21:51:07 ubuntu kernel: [   13.773830] [drm] radeon kernel modesetting enabled.
Sep  6 21:51:07 ubuntu kernel: [   13.781906] AMD IOMMUv2 driver by Joerg Roedel <joerg.roedel@amd.com>
Sep  6 21:51:07 ubuntu kernel: [   13.781917] AMD IOMMUv2 functionality not available on this system
Sep  6 21:51:07 ubuntu kernel: [   13.789598] CRAT table not found
Sep  6 21:51:07 ubuntu kernel: [   13.789607] Finished initializing topology ret=0
Sep  6 21:51:07 ubuntu kernel: [   13.789766] kfd kfd: Initialized module
Sep  6 21:51:07 ubuntu kernel: [   13.790266] checking generic (e0000000 420000) vs hw (e0000000 10000000)
Sep  6 21:51:07 ubuntu kernel: [   13.790272] fb: switching to radeondrmfb from EFI VGA
Sep  6 21:51:07 ubuntu kernel: [   13.790327] Console: switching to colour dummy device 80x25
Sep  6 21:51:07 ubuntu kernel: [   13.791283] [drm] initializing kernel modesetting (KABINI 0x1002:0x983D 0x1025:0x080D).
Sep  6 21:51:07 ubuntu kernel: [   13.791312] [drm] register mmio base: 0xF0900000
Sep  6 21:51:07 ubuntu kernel: [   13.791316] [drm] register mmio size: 262144
Sep  6 21:51:07 ubuntu kernel: [   13.791329] [drm] doorbell mmio base: 0xF0000000
Sep  6 21:51:07 ubuntu kernel: [   13.791332] [drm] doorbell mmio size: 8388608
Sep  6 21:51:07 ubuntu kernel: [   13.791344] [drm] ACPI VFCT contains a BIOS for 00:01.0 1002:983d, size 17920
Sep  6 21:51:07 ubuntu kernel: [   13.791367] ATOM BIOS: AMD
Sep  6 21:51:07 ubuntu kernel: [   13.791463] radeon 0000:00:01.0: VRAM: 512M 0x0000000000000000 - 0x000000001FFFFFFF (512M used)
Sep  6 21:51:07 ubuntu kernel: [   13.791470] radeon 0000:00:01.0: GTT: 1024M 0x0000000020000000 - 0x000000005FFFFFFF
Sep  6 21:51:07 ubuntu kernel: [   13.791475] [drm] Detected VRAM RAM=512M, BAR=256M
Sep  6 21:51:07 ubuntu kernel: [   13.791478] [drm] RAM width 128bits DDR
Sep  6 21:51:07 ubuntu kernel: [   13.791612] [TTM] Zone  kernel: Available graphics memory: 2735000 kiB
Sep  6 21:51:07 ubuntu kernel: [   13.791617] [TTM] Zone   dma32: Available graphics memory: 2097152 kiB
Sep  6 21:51:07 ubuntu kernel: [   13.791620] [TTM] Initializing pool allocator
Sep  6 21:51:07 ubuntu kernel: [   13.791634] [TTM] Initializing DMA pool allocator
Sep  6 21:51:07 ubuntu kernel: [   13.791684] [drm] radeon: 512M of VRAM memory ready
Sep  6 21:51:07 ubuntu kernel: [   13.791689] [drm] radeon: 1024M of GTT memory ready.
Sep  6 21:51:07 ubuntu kernel: [   13.791733] [drm] Loading kabini Microcode
Sep  6 21:51:07 ubuntu kernel: [   13.792008] [drm] Internal thermal controller without fan control
Sep  6 21:51:07 ubuntu kernel: [   13.793248] [drm] radeon: dpm initialized
Sep  6 21:51:07 ubuntu kernel: [   13.797611] [drm] Found VCE firmware/feedback version 40.2.2 / 15!
Sep  6 21:51:07 ubuntu kernel: [   13.797636] [drm] GART: num cpu pages 262144, num gpu pages 262144
Sep  6 21:51:07 ubuntu kernel: [   13.820590] [drm] PCIE GART of 1024M enabled (table at 0x000000000078C000).
Sep  6 21:51:07 ubuntu kernel: [   13.820931] radeon 0000:00:01.0: WB enabled
Sep  6 21:51:07 ubuntu kernel: [   13.820966] radeon 0000:00:01.0: fence driver on ring 0 use gpu addr 0x0000000020000c00 and cpu addr 0xffff880194bdfc00
Sep  6 21:51:07 ubuntu kernel: [   13.820975] radeon 0000:00:01.0: fence driver on ring 1 use gpu addr 0x0000000020000c04 and cpu addr 0xffff880194bdfc04
Sep  6 21:51:07 ubuntu kernel: [   13.820982] radeon 0000:00:01.0: fence driver on ring 2 use gpu addr 0x0000000020000c08 and cpu addr 0xffff880194bdfc08
Sep  6 21:51:07 ubuntu kernel: [   13.820989] radeon 0000:00:01.0: fence driver on ring 3 use gpu addr 0x0000000020000c0c and cpu addr 0xffff880194bdfc0c
Sep  6 21:51:07 ubuntu kernel: [   13.820996] radeon 0000:00:01.0: fence driver on ring 4 use gpu addr 0x0000000020000c10 and cpu addr 0xffff880194bdfc10
Sep  6 21:51:07 ubuntu kernel: [   13.821931] radeon 0000:00:01.0: fence driver on ring 5 use gpu addr 0x0000000000076c98 and cpu addr 0xffffc90005436c98
Sep  6 21:51:07 ubuntu kernel: [   13.822059] radeon 0000:00:01.0: fence driver on ring 6 use gpu addr 0x0000000020000c18 and cpu addr 0xffff880194bdfc18
Sep  6 21:51:07 ubuntu kernel: [   13.822067] radeon 0000:00:01.0: fence driver on ring 7 use gpu addr 0x0000000020000c1c and cpu addr 0xffff880194bdfc1c
Sep  6 21:51:07 ubuntu kernel: [   13.822074] [drm] Supports vblank timestamp caching Rev 2 (21.10.2013).
Sep  6 21:51:07 ubuntu kernel: [   13.822078] [drm] Driver supports precise vblank timestamp query.
Sep  6 21:51:07 ubuntu kernel: [   13.822154] radeon 0000:00:01.0: radeon: using MSI.
Sep  6 21:51:07 ubuntu kernel: [   13.822216] [drm] radeon: irq initialized.
Sep  6 21:51:07 ubuntu kernel: [   13.827833] [drm] ring test on 0 succeeded in 3 usecs
Sep  6 21:51:07 ubuntu kernel: [   13.827952] [drm] ring test on 1 succeeded in 3 usecs
Sep  6 21:51:07 ubuntu kernel: [   13.827986] [drm] ring test on 2 succeeded in 3 usecs
Sep  6 21:51:07 ubuntu kernel: [   13.828250] [drm] ring test on 3 succeeded in 4 usecs
Sep  6 21:51:07 ubuntu kernel: [   13.828263] [drm] ring test on 4 succeeded in 4 usecs
Sep  6 21:51:07 ubuntu kernel: [   13.841628] usb 3-1: New USB device found, idVendor=8644, idProduct=800b
Sep  6 21:51:07 ubuntu kernel: [   13.841637] usb 3-1: New USB device strings: Mfr=1, Product=2, SerialNumber=3
Sep  6 21:51:07 ubuntu kernel: [   13.841644] usb 3-1: Product: USB Flash Disk
Sep  6 21:51:07 ubuntu kernel: [   13.841651] usb 3-1: Manufacturer: General
Sep  6 21:51:07 ubuntu kernel: [   13.841656] usb 3-1: SerialNumber: 0000000000001379
Sep  6 21:51:07 ubuntu kernel: [   13.853723] usb-storage 3-1:1.0: USB Mass Storage device detected
Sep  6 21:51:07 ubuntu kernel: [   13.854057] scsi host1: usb-storage 3-1:1.0
Sep  6 21:51:07 ubuntu kernel: [   13.854300] usbcore: registered new interface driver usb-storage
Sep  6 21:51:07 ubuntu kernel: [   13.859025] usbcore: registered new interface driver uas
Sep  6 21:51:07 ubuntu kernel: [   13.874028] [drm] ring test on 5 succeeded in 2 usecs
Sep  6 21:51:07 ubuntu kernel: [   13.893109] usb 4-1: New USB device found, idVendor=064e, idProduct=e330
Sep  6 21:51:07 ubuntu kernel: [   13.893118] usb 4-1: New USB device strings: Mfr=3, Product=1, SerialNumber=0
Sep  6 21:51:07 ubuntu kernel: [   13.893125] usb 4-1: Product: HD WebCam
Sep  6 21:51:07 ubuntu kernel: [   13.893131] usb 4-1: Manufacturer: SuYin
Sep  6 21:51:07 ubuntu kernel: [   13.893920] [drm] UVD initialized successfully.
Sep  6 21:51:07 ubuntu kernel: [   14.003250] [drm] ring test on 6 succeeded in 17 usecs
Sep  6 21:51:07 ubuntu kernel: [   14.003267] [drm] ring test on 7 succeeded in 3 usecs
Sep  6 21:51:07 ubuntu kernel: [   14.003271] [drm] VCE initialized successfully.
Sep  6 21:51:07 ubuntu kernel: [   14.008474] [drm] ib test on ring 0 succeeded in 0 usecs
Sep  6 21:51:07 ubuntu kernel: [   14.152750] ata1: SATA link up 6.0 Gbps (SStatus 133 SControl 300)
Sep  6 21:51:07 ubuntu kernel: [   14.153404] ata1.00: ATA-9: WDC WD5000LPVX-22V0TT0, 01.01A01, max UDMA/133
Sep  6 21:51:07 ubuntu kernel: [   14.153413] ata1.00: 976773168 sectors, multi 0: LBA48 NCQ (depth 31/32), AA
Sep  6 21:51:07 ubuntu kernel: [   14.154064] ata1.00: configured for UDMA/133
Sep  6 21:51:07 ubuntu kernel: [   14.154484] scsi 0:0:0:0: Direct-Access     ATA      WDC WD5000LPVX-2 1A01 PQ: 0 ANSI: 5
Sep  6 21:51:07 ubuntu kernel: [   14.155243] sd 0:0:0:0: [sda] 976773168 512-byte logical blocks: (500 GB/465 GiB)
Sep  6 21:51:07 ubuntu kernel: [   14.155251] sd 0:0:0:0: [sda] 4096-byte physical blocks
Sep  6 21:51:07 ubuntu kernel: [   14.155298] sd 0:0:0:0: Attached scsi generic sg0 type 0
Sep  6 21:51:07 ubuntu kernel: [   14.155452] sd 0:0:0:0: [sda] Write Protect is off
Sep  6 21:51:07 ubuntu kernel: [   14.155462] sd 0:0:0:0: [sda] Mode Sense: 00 3a 00 00
Sep  6 21:51:07 ubuntu kernel: [   14.155559] sd 0:0:0:0: [sda] Write cache: enabled, read cache: enabled, doesn't support DPO or FUA
Sep  6 21:51:07 ubuntu kernel: [   14.260727] tsc: Refined TSC clocksource calibration: 998.128 MHz
Sep  6 21:51:07 ubuntu kernel: [   14.268811] usb 6-2: new full-speed USB device number 2 using ohci-pci
Sep  6 21:51:07 ubuntu kernel: [   14.284618] psmouse serio1: elantech: assuming hardware version 4 (with firmware version 0x461f00)
Sep  6 21:51:07 ubuntu kernel: [   14.301813] psmouse serio1: elantech: Synaptics capabilities query result 0x11, 0x16, 0x0c.
Sep  6 21:51:07 ubuntu kernel: [   14.388909] input: ETPS/2 Elantech Touchpad as /devices/platform/i8042/serio1/input/input8
Sep  6 21:51:07 ubuntu kernel: [   14.429820] usb 6-2: New USB device found, idVendor=0489, idProduct=e05f
Sep  6 21:51:07 ubuntu kernel: [   14.429833] usb 6-2: New USB device strings: Mfr=0, Product=0, SerialNumber=0
Sep  6 21:51:07 ubuntu kernel: [   14.504984] [drm] ib test on ring 1 succeeded in 0 usecs
Sep  6 21:51:07 ubuntu kernel: [   14.696946] usb 6-3: new full-speed USB device number 3 using ohci-pci
Sep  6 21:51:07 ubuntu kernel: [   14.853764] scsi 1:0:0:0: Direct-Access     General  USB Flash Disk   1.0  PQ: 0 ANSI: 2
Sep  6 21:51:07 ubuntu kernel: [   14.854532] sd 1:0:0:0: Attached scsi generic sg1 type 0
Sep  6 21:51:07 ubuntu kernel: [   14.855478] sd 1:0:0:0: [sdb] 3915776 512-byte logical blocks: (2.00 GB/1.86 GiB)
Sep  6 21:51:07 ubuntu kernel: [   14.856480] sd 1:0:0:0: [sdb] Write Protect is off
Sep  6 21:51:07 ubuntu kernel: [   14.856490] sd 1:0:0:0: [sdb] Mode Sense: 03 00 00 00
Sep  6 21:51:07 ubuntu kernel: [   14.857611] sd 1:0:0:0: [sdb] No Caching mode page found
Sep  6 21:51:07 ubuntu kernel: [   14.857625] sd 1:0:0:0: [sdb] Assuming drive cache: write through
Sep  6 21:51:07 ubuntu kernel: [   14.863653]  sdb: sdb1
Sep  6 21:51:07 ubuntu kernel: [   14.867847] sd 1:0:0:0: [sdb] Attached SCSI removable disk
Sep  6 21:51:07 ubuntu kernel: [   14.869786] usb 6-3: New USB device found, idVendor=04f3, idProduct=009e
Sep  6 21:51:07 ubuntu kernel: [   14.869812] usb 6-3: New USB device strings: Mfr=4, Product=14, SerialNumber=0
Sep  6 21:51:07 ubuntu kernel: [   14.869819] usb 6-3: Product: Touchscreen
Sep  6 21:51:07 ubuntu kernel: [   14.869825] usb 6-3: Manufacturer: ELAN
Sep  6 21:51:07 ubuntu kernel: [   14.882450] hidraw: raw HID events driver (C) Jiri Kosina
Sep  6 21:51:07 ubuntu kernel: [   14.900209] usbcore: registered new interface driver usbhid
Sep  6 21:51:07 ubuntu kernel: [   14.900216] usbhid: USB HID core driver
Sep  6 21:51:07 ubuntu kernel: [   15.004821] [drm] ib test on ring 2 succeeded in 0 usecs
Sep  6 21:51:07 ubuntu kernel: [   15.004941] [drm] ib test on ring 3 succeeded in 0 usecs
Sep  6 21:51:07 ubuntu kernel: [   15.005025] [drm] ib test on ring 4 succeeded in 0 usecs
Sep  6 21:51:07 ubuntu kernel: [   15.261277] Switched to clocksource tsc
Sep  6 21:51:07 ubuntu kernel: [   15.525046] [drm] ib test on ring 5 succeeded
Sep  6 21:51:07 ubuntu kernel: [   15.545820] [drm] ib test on ring 6 succeeded
Sep  6 21:51:07 ubuntu kernel: [   15.546611] [drm] ib test on ring 7 succeeded
Sep  6 21:51:07 ubuntu kernel: [   15.554074] [drm] radeon atom DIG backlight initialized
Sep  6 21:51:07 ubuntu kernel: [   15.554082] [drm] Radeon Display Connectors
Sep  6 21:51:07 ubuntu kernel: [   15.554086] [drm] Connector 0:
Sep  6 21:51:07 ubuntu kernel: [   15.554090] [drm]   eDP-1
Sep  6 21:51:07 ubuntu kernel: [   15.554094] [drm]   HPD1
Sep  6 21:51:07 ubuntu kernel: [   15.554100] [drm]   DDC: 0x6530 0x6530 0x6534 0x6534 0x6538 0x6538 0x653c 0x653c
Sep  6 21:51:07 ubuntu kernel: [   15.554103] [drm]   Encoders:
Sep  6 21:51:07 ubuntu kernel: [   15.554107] [drm]     LCD1: INTERNAL_UNIPHY
Sep  6 21:51:07 ubuntu kernel: [   15.554110] [drm] Connector 1:
Sep  6 21:51:07 ubuntu kernel: [   15.554113] [drm]   DP-1
Sep  6 21:51:07 ubuntu kernel: [   15.554116] [drm]   HPD2
Sep  6 21:51:07 ubuntu kernel: [   15.554121] [drm]   DDC: 0x6540 0x6540 0x6544 0x6544 0x6548 0x6548 0x654c 0x654c
Sep  6 21:51:07 ubuntu kernel: [   15.554124] [drm]   Encoders:
Sep  6 21:51:07 ubuntu kernel: [   15.554127] [drm]     DFP1: INTERNAL_UNIPHY
Sep  6 21:51:07 ubuntu kernel: [   15.554130] [drm] Connector 2:
Sep  6 21:51:07 ubuntu kernel: [   15.554133] [drm]   VGA-1
Sep  6 21:51:07 ubuntu kernel: [   15.554138] [drm]   DDC: 0x65c0 0x65c0 0x65c4 0x65c4 0x65c8 0x65c8 0x65cc 0x65cc
Sep  6 21:51:07 ubuntu kernel: [   15.554141] [drm]   Encoders:
Sep  6 21:51:07 ubuntu kernel: [   15.554144] [drm]     CRT1: INTERNAL_KLDSCP_DAC1
Sep  6 21:51:07 ubuntu kernel: [   15.934342]  sda: sda1 sda2 sda3 sda4 sda5 sda6 sda7
Sep  6 21:51:07 ubuntu kernel: [   15.935749] sd 0:0:0:0: [sda] Attached SCSI disk
Sep  6 21:51:07 ubuntu kernel: [   16.291133] [drm] fb mappable at 0xE0990000
Sep  6 21:51:07 ubuntu kernel: [   16.291142] [drm] vram apper at 0xE0000000
Sep  6 21:51:07 ubuntu kernel: [   16.291145] [drm] size 4325376
Sep  6 21:51:07 ubuntu kernel: [   16.291148] [drm] fb depth is 24
Sep  6 21:51:07 ubuntu kernel: [   16.291150] [drm]    pitch is 5632
Sep  6 21:51:07 ubuntu kernel: [   16.291395] fbcon: radeondrmfb (fb0) is primary device
Sep  6 21:51:07 ubuntu kernel: [   16.987612] random: nonblocking pool is initialized
Sep  6 21:51:07 ubuntu kernel: [   17.465690] Console: switching to colour frame buffer device 170x48
Sep  6 21:51:07 ubuntu kernel: [   17.474022] radeon 0000:00:01.0: fb0: radeondrmfb frame buffer device
Sep  6 21:51:07 ubuntu kernel: [   17.474028] radeon 0000:00:01.0: registered panic notifier
Sep  6 21:51:07 ubuntu kernel: [   17.489789] [drm] Initialized radeon 2.40.0 20080528 for 0000:00:01.0 on minor 0
Sep  6 21:51:07 ubuntu kernel: [   19.694646] EXT4-fs (sda6): mounted filesystem with ordered data mode. Opts: (null)
Sep  6 21:51:07 ubuntu kernel: [   21.042470] squashfs: version 4.0 (2009/01/31) Phillip Lougher
Sep  6 21:51:07 ubuntu kernel: [   41.832563] Adding 5705724k swap on /dev/sda7.  Priority:-1 extents:1 across:5705724k FS
Sep  6 21:51:07 ubuntu kernel: [   42.117945] systemd-udevd[1185]: starting version 204
Sep  6 21:51:07 ubuntu kernel: [   42.477537] lp: driver loaded but no devices found
Sep  6 21:51:07 ubuntu kernel: [   42.524123] ppdev: user-space parallel port driver
Sep  6 21:51:07 ubuntu kernel: [   42.781627] Bluetooth: Core ver 2.20
Sep  6 21:51:07 ubuntu kernel: [   42.781678] NET: Registered protocol family 31
Sep  6 21:51:07 ubuntu kernel: [   42.781685] Bluetooth: HCI device and connection manager initialized
Sep  6 21:51:07 ubuntu kernel: [   42.785083] Bluetooth: HCI socket layer initialized
Sep  6 21:51:07 ubuntu kernel: [   42.785102] Bluetooth: L2CAP socket layer initialized
Sep  6 21:51:07 ubuntu kernel: [   42.785127] Bluetooth: SCO socket layer initialized
Sep  6 21:51:07 ubuntu kernel: [   42.860083] Bluetooth: RFCOMM TTY layer initialized
Sep  6 21:51:07 ubuntu kernel: [   42.860110] Bluetooth: RFCOMM socket layer initialized
Sep  6 21:51:07 ubuntu kernel: [   42.860133] Bluetooth: RFCOMM ver 1.11
Sep  6 21:51:07 ubuntu kernel: [   42.878491] Bluetooth: BNEP (Ethernet Emulation) ver 1.3
Sep  6 21:51:07 ubuntu kernel: [   42.878503] Bluetooth: BNEP filters: protocol multicast
Sep  6 21:51:07 ubuntu kernel: [   42.878518] Bluetooth: BNEP socket layer initialized
Sep  6 21:51:07 ubuntu kernel: [   42.934089] shpchp: Standard Hot Plug PCI Controller Driver version: 0.4
Sep  6 21:51:07 ubuntu kernel: [   43.010061] ACPI Warning: SystemIO range 0x0000000000000b00-0x0000000000000b07 conflicts with OpRegion 0x0000000000000b00-0x0000000000000b0f (_SB_.PCI0.SMBS.SMB0) (20141107/utaddress-258)
Sep  6 21:51:07 ubuntu kernel: [   43.010086] ACPI: If an ACPI driver is available for this device, you should use it instead of the native driver
Sep  6 21:51:07 ubuntu kernel: [   43.175751] MCE: In-kernel MCE decoding enabled.
Sep  6 21:51:07 ubuntu kernel: [   43.192578] usbcore: registered new interface driver btusb
Sep  6 21:51:07 ubuntu kernel: [   43.208831] EDAC MC: Ver: 3.0.0
Sep  6 21:51:07 ubuntu kernel: [   43.340443] init: cups main process (1282) killed by HUP signal
Sep  6 21:51:07 ubuntu kernel: [   43.340489] init: cups main process ended, respawning
Sep  6 21:51:07 ubuntu kernel: [   43.449483] AMD64 EDAC driver v3.4.0
Sep  6 21:51:07 ubuntu kernel: [   43.449594] EDAC amd64: DRAM ECC disabled.
Sep  6 21:51:07 ubuntu kernel: [   43.449610] EDAC amd64: ECC disabled in the BIOS or no ECC capability, module will not load.
Sep  6 21:51:07 ubuntu kernel: [   43.449610]  Either enable ECC checking or force module loading by setting 'ecc_enable_override'.
Sep  6 21:51:07 ubuntu kernel: [   43.449610]  (Note that use of the override may cause unknown side effects.)
Sep  6 21:51:07 ubuntu kernel: [   43.471439] usb 6-2: USB disconnect, device number 2
Sep  6 21:51:07 ubuntu kernel: [   43.476431] usbcore: registered new interface driver ath3k
Sep  6 21:51:07 ubuntu kernel: [   43.515864] media: Linux media interface: v0.10
Sep  6 21:51:07 ubuntu kernel: [   43.517356] input: ELAN Touchscreen as /devices/pci0000:00/0000:00:13.0/usb6/6-3/6-3:1.0/0003:04F3:009E.0001/input/input9
Sep  6 21:51:07 ubuntu kernel: [   43.518905] hid-multitouch 0003:04F3:009E.0001: input,hiddev0,hidraw0: USB HID v1.10 Device [ELAN Touchscreen] on usb-0000:00:13.0-3/input0
Sep  6 21:51:07 ubuntu kernel: [   43.570731] Linux video capture interface: v2.00
Sep  6 21:51:07 ubuntu kernel: [   43.627066] cfg80211: Calling CRDA to update world regulatory domain
Sep  6 21:51:07 ubuntu kernel: [   43.640944] AVX version of gcm_enc/dec engaged.
Sep  6 21:51:07 ubuntu kernel: [   43.640958] AES CTR mode by8 optimization enabled
Sep  6 21:51:07 ubuntu kernel: [   43.739001] device-mapper: multipath: version 1.7.0 loaded
Sep  6 21:51:07 ubuntu kernel: [   43.779683] uvcvideo: Found UVC 1.00 device HD WebCam (064e:e330)
Sep  6 21:51:07 ubuntu kernel: [   43.792539] input: HD WebCam as /devices/pci0000:00/0000:00:13.2/usb4/4-1/4-1:1.0/input/input11
Sep  6 21:51:07 ubuntu kernel: [   43.795285] usbcore: registered new interface driver uvcvideo
Sep  6 21:51:07 ubuntu kernel: [   43.795296] USB Video Class driver (1.1.1)
Sep  6 21:51:07 ubuntu kernel: [   43.883237] kvm: disabled by bios
Sep  6 21:51:07 ubuntu kernel: [   43.954367] input: HD-Audio Generic HDMI/DP,pcm=3 as /devices/pci0000:00/0000:00:01.1/sound/card0/input12
Sep  6 21:51:07 ubuntu kernel: [   43.967080] usb 6-2: new full-speed USB device number 4 using ohci-pci
Sep  6 21:51:07 ubuntu kernel: [   43.981582] sound hdaudioC1D0: autoconfig: line_outs=1 (0x14/0x0/0x0/0x0/0x0) type:speaker
Sep  6 21:51:07 ubuntu kernel: [   43.981602] sound hdaudioC1D0:    speaker_outs=0 (0x0/0x0/0x0/0x0/0x0)
Sep  6 21:51:07 ubuntu kernel: [   43.981612] sound hdaudioC1D0:    hp_outs=1 (0x21/0x0/0x0/0x0/0x0)
Sep  6 21:51:07 ubuntu kernel: [   43.981620] sound hdaudioC1D0:    mono: mono_out=0x0
Sep  6 21:51:07 ubuntu kernel: [   43.981626] sound hdaudioC1D0:    inputs:
Sep  6 21:51:07 ubuntu kernel: [   43.981636] sound hdaudioC1D0:      Mic=0x1b
Sep  6 21:51:07 ubuntu kernel: [   43.996436] input: HD-Audio Generic Headphone as /devices/pci0000:00/0000:00:14.2/sound/card1/input13
Sep  6 21:51:08 ubuntu kernel: [   44.084600] ath: phy0: WB335 1-ANT card detected
Sep  6 21:51:08 ubuntu kernel: [   44.093945] ath: phy0: Enable LNA combining
Sep  6 21:51:08 ubuntu kernel: [   44.095121] ath: phy0: ASPM enabled: 0x42
Sep  6 21:51:08 ubuntu kernel: [   44.095131] ath: EEPROM regdomain: 0x6c
Sep  6 21:51:08 ubuntu kernel: [   44.095135] ath: EEPROM indicates we should expect a direct regpair map
Sep  6 21:51:08 ubuntu kernel: [   44.095141] ath: Country alpha2 being used: 00
Sep  6 21:51:08 ubuntu kernel: [   44.095144] ath: Regpair used: 0x6c
Sep  6 21:51:08 ubuntu kernel: [   44.123722] ieee80211 phy0: Selected rate control algorithm 'minstrel_ht'
Sep  6 21:51:08 ubuntu kernel: [   44.125397] ieee80211 phy0: Atheros AR9565 Rev:1 mem=0xffffc90006180000, irq=35
Sep  6 21:51:08 ubuntu failsafe: Failsafe of 120 seconds reached.
Sep  6 21:51:08 ubuntu kernel: [   44.231489] acer_wmi: Acer Laptop ACPI-WMI Extras
Sep  6 21:51:08 ubuntu kernel: [   44.231867] acer_wmi: Function bitmap for Communication Button: 0x801
Sep  6 21:51:08 ubuntu kernel: [   44.247614] acer_wmi: Brightness must be controlled by acpi video driver
Sep  6 21:51:08 ubuntu kernel: [   44.260869] acer_wmi: Enabling Launch Manager failed: 0xe2 - 0x0
Sep  6 21:51:08 ubuntu kernel: [   44.261185] input: Acer WMI hotkeys as /devices/virtual/input/input14
Sep  6 21:51:08 ubuntu kernel: [   44.263901] input: Acer BMA150 accelerometer as /devices/virtual/input/input15
Sep  6 21:51:08 ubuntu kernel: [   44.430239] cfg80211: World regulatory domain updated:
Sep  6 21:51:08 ubuntu kernel: [   44.430256] cfg80211:  DFS Master region: unset
Sep  6 21:51:08 ubuntu kernel: [   44.430262] cfg80211:   (start_freq - end_freq @ bandwidth), (max_antenna_gain, max_eirp), (dfs_cac_time)
Sep  6 21:51:08 ubuntu kernel: [   44.430272] cfg80211:   (2402000 KHz - 2472000 KHz @ 40000 KHz), (300 mBi, 2000 mBm), (N/A)
Sep  6 21:51:08 ubuntu kernel: [   44.430281] cfg80211:   (2457000 KHz - 2482000 KHz @ 40000 KHz), (300 mBi, 2000 mBm), (N/A)
Sep  6 21:51:08 ubuntu kernel: [   44.430290] cfg80211:   (2474000 KHz - 2494000 KHz @ 20000 KHz), (300 mBi, 2000 mBm), (N/A)
Sep  6 21:51:08 ubuntu kernel: [   44.430297] cfg80211:   (5170000 KHz - 5250000 KHz @ 40000 KHz), (300 mBi, 2000 mBm), (N/A)
Sep  6 21:51:08 ubuntu kernel: [   44.430305] cfg80211:   (5735000 KHz - 5835000 KHz @ 40000 KHz), (300 mBi, 2000 mBm), (N/A)
Sep  6 21:51:08 ubuntu kernel: [   44.492634] init: failsafe main process (1471) killed by TERM signal
Sep  6 21:51:08 ubuntu ModemManager[1550]: <info>  ModemManager (version 1.0.0) starting...
Sep  6 21:51:09 ubuntu NetworkManager[1691]: <info> NetworkManager (version 0.9.8.8) is starting...
Sep  6 21:51:09 ubuntu NetworkManager[1691]: <info> Read config file /etc/NetworkManager/NetworkManager.conf
Sep  6 21:51:09 ubuntu NetworkManager[1691]: <info> WEXT support is enabled
Sep  6 21:51:09 ubuntu cron[1662]: (CRON) INFO (pidfile fd = 3)
Sep  6 21:51:09 ubuntu cron[1830]: (CRON) STARTUP (fork ok)
Sep  6 21:51:09 ubuntu cron[1830]: (CRON) INFO (Running @reboot jobs)
Sep  6 21:51:09 ubuntu kernel: [   45.737582] init: alsa-restore main process (1826) terminated with status 99
Sep  6 21:51:09 ubuntu NetworkManager[1691]: <info> VPN: loaded org.freedesktop.NetworkManager.pptp
Sep  6 21:51:09 ubuntu NetworkManager[1691]: <info> DNS: loaded plugin dnsmasq
Sep  6 21:51:09 ubuntu dbus[1193]: [system] Activating service name='org.freedesktop.PolicyKit1' (using servicehelper)
Sep  6 21:51:10 ubuntu polkitd[1858]: started daemon version 0.105 using authority implementation `local' version `0.105'
Sep  6 21:51:10 ubuntu dbus[1193]: [system] Successfully activated service 'org.freedesktop.PolicyKit1'
Sep  6 21:51:10 ubuntu NetworkManager[1691]:    SCPlugin-Ifupdown: init!
Sep  6 21:51:10 ubuntu NetworkManager[1691]:    SCPlugin-Ifupdown: update_system_hostname
Sep  6 21:51:10 ubuntu NetworkManager[1691]:       interface-parser: parsing file /etc/network/interfaces
Sep  6 21:51:10 ubuntu NetworkManager[1691]:       interface-parser: finished parsing file /etc/network/interfaces
Sep  6 21:51:10 ubuntu NetworkManager[1691]:    SCPluginIfupdown: management mode: unmanaged
Sep  6 21:51:10 ubuntu NetworkManager[1691]:    SCPlugin-Ifupdown: devices added (path: /sys/devices/pci0000:00/0000:00:02.3/0000:01:00.0/net/wlan0, iface: wlan0)
Sep  6 21:51:10 ubuntu NetworkManager[1691]:    SCPlugin-Ifupdown: device added (path: /sys/devices/pci0000:00/0000:00:02.3/0000:01:00.0/net/wlan0, iface: wlan0): no ifupdown configuration found.
Sep  6 21:51:10 ubuntu NetworkManager[1691]:    SCPlugin-Ifupdown: devices added (path: /sys/devices/virtual/net/lo, iface: lo)
Sep  6 21:51:10 ubuntu NetworkManager[1691]:    SCPlugin-Ifupdown: device added (path: /sys/devices/virtual/net/lo, iface: lo): no ifupdown configuration found.
Sep  6 21:51:10 ubuntu NetworkManager[1691]:    SCPlugin-Ifupdown: end _init.
Sep  6 21:51:10 ubuntu NetworkManager[1691]: <info> Loaded plugin ifupdown: (C) 2008 Canonical Ltd.  To report bugs please use the NetworkManager mailing list.
Sep  6 21:51:10 ubuntu NetworkManager[1691]: <info> Loaded plugin keyfile: (c) 2007 - 2010 Red Hat, Inc.  To report bugs please use the NetworkManager mailing list.
Sep  6 21:51:10 ubuntu NetworkManager[1691]:    SCPlugin-Ofono: Acquired D-Bus service com.canonical.NMOfono
Sep  6 21:51:10 ubuntu NetworkManager[1691]:    SCPlugin-Ofono: init!
Sep  6 21:51:10 ubuntu NetworkManager[1691]:    SCPlugin-Ofono: end _init.
Sep  6 21:51:10 ubuntu NetworkManager[1691]: <info> Loaded plugin (null): (null)
Sep  6 21:51:10 ubuntu NetworkManager[1691]:    Ifupdown: get unmanaged devices count: 0
Sep  6 21:51:10 ubuntu NetworkManager[1691]:    SCPlugin-Ifupdown: (38831888) ... get_connections.
Sep  6 21:51:10 ubuntu NetworkManager[1691]:    SCPlugin-Ifupdown: (38831888) ... get_connections (managed=false): return empty list.
Sep  6 21:51:10 ubuntu NetworkManager[1691]:    SCPlugin-Ofono: (38641488) ... get_connections.
Sep  6 21:51:10 ubuntu NetworkManager[1691]:    SCPlugin-Ofono: (38641488) connections count: 0
Sep  6 21:51:10 ubuntu NetworkManager[1691]:    Ifupdown: get unmanaged devices count: 0
Sep  6 21:51:10 ubuntu NetworkManager[1691]: <info> monitoring kernel firmware directory '/lib/firmware'.
Sep  6 21:51:10 ubuntu NetworkManager[1691]: <info> rfkill0: found WiFi radio killswitch (at /sys/devices/pci0000:00/0000:00:02.3/0000:01:00.0/ieee80211/phy0/rfkill0) (driver ath9k)
Sep  6 21:51:10 ubuntu NetworkManager[1691]: <info> rfkill1: found WiFi radio killswitch (at /sys/devices/platform/acer-wmi/rfkill/rfkill1) (platform driver acer-wmi)
Sep  6 21:51:10 ubuntu acpid: starting up with netlink and the input layer
Sep  6 21:51:10 ubuntu NetworkManager[1691]: <info> WiFi enabled by radio killswitch; enabled by state file
Sep  6 21:51:10 ubuntu NetworkManager[1691]: <info> WWAN enabled by radio killswitch; enabled by state file
Sep  6 21:51:10 ubuntu NetworkManager[1691]: <info> WiMAX enabled by radio killswitch; enabled by state file
Sep  6 21:51:10 ubuntu NetworkManager[1691]: <info> Networking is enabled by state file
Sep  6 21:51:10 ubuntu acpid: 9 rules loaded
Sep  6 21:51:10 ubuntu acpid: waiting for events: event logging is off
Sep  6 21:51:10 ubuntu NetworkManager[1691]: <info> (wlan0): using nl80211 for WiFi device control
Sep  6 21:51:10 ubuntu NetworkManager[1691]: <info> (wlan0): driver supports Access Point (AP) mode
Sep  6 21:51:10 ubuntu NetworkManager[1691]: <info> (wlan0): new 802.11 WiFi device (driver: 'ath9k' ifindex: 2)
Sep  6 21:51:10 ubuntu NetworkManager[1691]: <info> (wlan0): exported as /org/freedesktop/NetworkManager/Devices/0
Sep  6 21:51:10 ubuntu NetworkManager[1691]: <info> (wlan0): device state change: unmanaged -> unavailable (reason 'managed') [10 20 2]
Sep  6 21:51:10 ubuntu NetworkManager[1691]: <info> (wlan0): bringing up device.
Sep  6 21:51:10 ubuntu NetworkManager[1691]: <info> (wlan0): preparing device.
Sep  6 21:51:10 ubuntu NetworkManager[1691]: <info> (wlan0): deactivating device (reason 'managed') [2]
Sep  6 21:51:10 ubuntu kernel: [   46.399123] IPv6: ADDRCONF(NETDEV_UP): wlan0: link is not ready
Sep  6 21:51:10 ubuntu dbus[1193]: [system] Activating service name='fi.w1.wpa_supplicant1' (using servicehelper)
Sep  6 21:51:10 ubuntu NetworkManager[1691]: <warn> /sys/devices/virtual/net/lo: couldn't determine device driver; ignoring...
Sep  6 21:51:10 ubuntu NetworkManager[1691]: <warn> /sys/devices/virtual/net/lo: couldn't determine device driver; ignoring...
Sep  6 21:51:10 ubuntu NetworkManager[1691]: <info> urfkill disappeared from the bus
Sep  6 21:51:10 ubuntu NetworkManager[1691]: <info> ModemManager available in the bus
Sep  6 21:51:10 ubuntu dbus[1193]: [system] Successfully activated service 'fi.w1.wpa_supplicant1'
Sep  6 21:51:10 ubuntu NetworkManager[1691]: <info> wpa_supplicant started
Sep  6 21:51:10 ubuntu wpa_supplicant[1966]: Successfully initialized wpa_supplicant
Sep  6 21:51:10 ubuntu NetworkManager[1691]: <info> (wlan0) supports 4 scan SSIDs
Sep  6 21:51:10 ubuntu NetworkManager[1691]: <warn> Trying to remove a non-existant call id.
Sep  6 21:51:10 ubuntu NetworkManager[1691]: <info> (wlan0): supplicant interface state: starting -> ready
Sep  6 21:51:10 ubuntu NetworkManager[1691]: <info> (wlan0): device state change: unavailable -> disconnected (reason 'supplicant-available') [20 30 42]
Sep  6 21:51:10 ubuntu NetworkManager[1691]: <info> (wlan0): supplicant interface state: ready -> disconnected
Sep  6 21:51:10 ubuntu NetworkManager[1691]: <info> (wlan0) supports 4 scan SSIDs
Sep  6 21:51:10 ubuntu wpa_supplicant[2026]: wlan0: CTRL-EVENT-SCAN-STARTED
Sep  6 21:51:10 ubuntu dbus[1193]: [system] Activating service name='org.freedesktop.Accounts' (using servicehelper)
Sep  6 21:51:11 ubuntu accounts-daemon[2082]: started daemon version 0.6.35
Sep  6 21:51:11 ubuntu dbus[1193]: [system] Successfully activated service 'org.freedesktop.Accounts'
Sep  6 21:51:11 ubuntu ntpdate[2169]: Can't find host ntp.ubuntu.com: Name or service not known (-2)
Sep  6 21:51:11 ubuntu ntpdate[2169]: no servers can be used, exiting
Sep  6 21:51:11 ubuntu NetworkManager[1691]: <info> (wlan0): supplicant interface state: disconnected -> inactive
Sep  6 21:51:12 ubuntu ModemManager[1550]: <warn>  Couldn't find support for device at '/sys/devices/pci0000:00/0000:00:02.3/0000:01:00.0': not supported by any plugin
Sep  6 21:51:13 ubuntu kernel: [   49.128460] usb 6-2: New USB device found, idVendor=0489, idProduct=e05f
Sep  6 21:51:13 ubuntu kernel: [   49.128475] usb 6-2: New USB device strings: Mfr=0, Product=0, SerialNumber=0
Sep  6 21:51:13 ubuntu bluetoothd[1239]: Unknown command complete for opcode 19
Sep  6 21:51:13 ubuntu bluetoothd[1239]: Adapter /org/bluez/1239/hci0 has been enabled
Sep  6 21:51:14 ubuntu kernel: [   50.264433] init: plymouth-upstart-bridge main process ended, respawning
Sep  6 21:51:14 ubuntu kernel: [   50.277451] init: plymouth-upstart-bridge main process (2238) terminated with status 1
Sep  6 21:51:14 ubuntu kernel: [   50.277492] init: plymouth-upstart-bridge main process ended, respawning
Sep  6 21:51:20 ubuntu NetworkManager[1691]: <info> WiFi hardware radio set enabled
Sep  6 21:51:26 ubuntu dbus[1193]: [system] Activating service name='org.freedesktop.UPower' (using servicehelper)
Sep  6 21:51:26 ubuntu dbus[1193]: [system] Successfully activated service 'org.freedesktop.UPower'
Sep  6 21:51:26 ubuntu dbus[1193]: [system] Activating service name='org.freedesktop.RealtimeKit1' (using servicehelper)
Sep  6 21:51:26 ubuntu dbus[1193]: [system] Successfully activated service 'org.freedesktop.RealtimeKit1'
Sep  6 21:51:26 ubuntu rtkit-daemon[2753]: Successfully called chroot.
Sep  6 21:51:26 ubuntu rtkit-daemon[2753]: Successfully dropped privileges.
Sep  6 21:51:26 ubuntu rtkit-daemon[2753]: Successfully limited resources.
Sep  6 21:51:26 ubuntu rtkit-daemon[2753]: Running.
Sep  6 21:51:26 ubuntu rtkit-daemon[2753]: Canary thread running.
Sep  6 21:51:26 ubuntu rtkit-daemon[2753]: Watchdog thread running.
Sep  6 21:51:26 ubuntu rtkit-daemon[2753]: Successfully made thread 2751 of process 2751 (n/a) owned by '999' high priority at nice level -11.
Sep  6 21:51:26 ubuntu rtkit-daemon[2753]: Supervising 1 threads of 1 processes of 1 users.
Sep  6 21:51:27 ubuntu dbus[1193]: [system] Activating service name='org.freedesktop.systemd1' (using servicehelper)
Sep  6 21:51:27 ubuntu rtkit-daemon[2753]: Successfully made thread 2808 of process 2751 (n/a) owned by '999' RT at priority 5.
Sep  6 21:51:27 ubuntu rtkit-daemon[2753]: Supervising 2 threads of 1 processes of 1 users.
Sep  6 21:51:27 ubuntu dbus[1193]: [system] Successfully activated service 'org.freedesktop.systemd1'
Sep  6 21:51:27 ubuntu rtkit-daemon[2753]: Successfully made thread 2845 of process 2751 (n/a) owned by '999' RT at priority 5.
Sep  6 21:51:27 ubuntu rtkit-daemon[2753]: Supervising 3 threads of 1 processes of 1 users.
Sep  6 21:51:27 ubuntu rtkit-daemon[2753]: Successfully made thread 2852 of process 2751 (n/a) owned by '999' RT at priority 5.
Sep  6 21:51:27 ubuntu rtkit-daemon[2753]: Supervising 4 threads of 1 processes of 1 users.
Sep  6 21:51:27 ubuntu bluetoothd[1239]: Endpoint registered: sender=:1.44 path=/MediaEndpoint/HFPAG
Sep  6 21:51:27 ubuntu bluetoothd[1239]: Endpoint registered: sender=:1.44 path=/MediaEndpoint/HFPHS
Sep  6 21:51:27 ubuntu bluetoothd[1239]: Endpoint registered: sender=:1.44 path=/MediaEndpoint/A2DPSource
Sep  6 21:51:27 ubuntu bluetoothd[1239]: Endpoint registered: sender=:1.44 path=/MediaEndpoint/A2DPSink
Sep  6 21:51:27 ubuntu rtkit-daemon[2753]: Successfully made thread 2889 of process 2889 (n/a) owned by '999' high priority at nice level -11.
Sep  6 21:51:27 ubuntu rtkit-daemon[2753]: Supervising 5 threads of 2 processes of 1 users.
Sep  6 21:51:27 ubuntu pulseaudio[2889]: [pulseaudio] pid.c: Daemon already running.
Sep  6 21:51:28 ubuntu dbus[1193]: [system] Activating service name='org.freedesktop.ColorManager' (using servicehelper)
Sep  6 21:51:28 ubuntu colord: Using config file /etc/colord.conf
Sep  6 21:51:29 ubuntu colord: Using mapping database file /var/lib/colord/mapping.db
Sep  6 21:51:29 ubuntu colord: Using device database file /var/lib/colord/storage.db
Sep  6 21:51:29 ubuntu colord: loaded plugin libcd_plugin_camera.so
Sep  6 21:51:29 ubuntu colord: plugin /usr/lib/x86_64-linux-gnu/colord-plugins/libcd_plugin_sane.so not loaded: plugin refused to load
Sep  6 21:51:29 ubuntu colord: loaded plugin libcd_plugin_scanner.so
Sep  6 21:51:29 ubuntu colord: Daemon ready for requests
Sep  6 21:51:29 ubuntu dbus[1193]: [system] Successfully activated service 'org.freedesktop.ColorManager'
Sep  6 21:51:29 ubuntu colord: Profile added: icc-c3e6382fa9b2d31b01b736f6f97aac3a
Sep  6 21:51:29 ubuntu colord: Profile added: icc-3a34aa6c3d1fb1ef63ff41e04ee00979
Sep  6 21:51:29 ubuntu colord: Profile added: icc-d6490883a866e4059370e1de1d840283
Sep  6 21:51:29 ubuntu colord: Profile added: icc-526fbc9bdf0d7156c553998d47a3b5fc
Sep  6 21:51:29 ubuntu colord: Profile added: icc-6a245ab2d8892e2e56232af93cd48b81
Sep  6 21:51:29 ubuntu colord: Profile added: icc-a1d13bd5309e0f06ceda6f0d75367823
Sep  6 21:51:29 ubuntu colord: Profile added: icc-a10be98be58460669fcdc6946939b7cf
Sep  6 21:51:29 ubuntu colord: Profile added: icc-523e494bc2f53c53d51d0758b07f4879
Sep  6 21:51:29 ubuntu colord: Profile added: icc-f64a1f19ce07290b35a752b00217b684
Sep  6 21:51:29 ubuntu colord: Profile added: icc-fb0ac62618f016ed9b92ce239258efa8
Sep  6 21:51:29 ubuntu colord: Profile added: icc-d4a7a2bd8ddaacf10e275e3db31d72b8
Sep  6 21:51:29 ubuntu colord: Profile added: icc-3bd2261b1125a0fd9ebf827a2d1bed84
Sep  6 21:51:29 ubuntu colord: Profile added: icc-0720e7cdbc792b77c0740c39f325ef9e
Sep  6 21:51:29 ubuntu colord: Profile added: icc-df7c0067b1eb9bcc9fc9b33bc3a797eb
Sep  6 21:51:29 ubuntu dbus[1193]: [system] Activating service name='org.freedesktop.locale1' (using servicehelper)
Sep  6 21:51:29 ubuntu colord: Profile added: icc-57f0d896250f6f98f77ca1b0d19019c0
Sep  6 21:51:29 ubuntu dbus[1193]: [system] Successfully activated service 'org.freedesktop.locale1'
Sep  6 21:51:29 ubuntu colord: Profile added: icc-2f1f11ecd613fe5551420fcaf5b11ff8
Sep  6 21:51:29 ubuntu colord: Profile added: icc-72f5b1cba915b68ea75cc843e270df5a
Sep  6 21:51:29 ubuntu colord: Profile added: icc-0383c34650771ce95ef93fe916867725
Sep  6 21:51:29 ubuntu colord: Profile added: icc-6ad6d63767ce0393245528ada92f1cb2
Sep  6 21:51:29 ubuntu colord: Profile added: icc-c227f46f246694ba9971f270cb61a0c1
Sep  6 21:51:29 ubuntu colord: Profile added: icc-b0701c2ccf059287d0b067464df8bda9
Sep  6 21:51:29 ubuntu colord: Profile added: icc-ea421e3a65cfa796e2732ce36086e327
Sep  6 21:51:29 ubuntu colord: Profile added: icc-353a6bcabda00f04b6988f89126ce6f5
Sep  6 21:51:29 ubuntu colord: Profile added: icc-654b99c87e67edb1c1cfb0dcb7fa9d04
Sep  6 21:51:29 ubuntu colord: Device added: xrandr-AU Optronics
Sep  6 21:51:30 ubuntu colord: Automatic metadata add icc-9b200edea9e2634c1d6eef6e8a5238d8 to xrandr-AU Optronics
Sep  6 21:51:30 ubuntu colord: Profile added: icc-9b200edea9e2634c1d6eef6e8a5238d8
Sep  6 21:51:30 ubuntu dbus[1193]: [system] Activating service name='org.freedesktop.UDisks2' (using servicehelper)
Sep  6 21:51:30 ubuntu udisksd[3007]: udisks daemon version 2.1.3 starting
Sep  6 21:51:30 ubuntu dbus[1193]: [system] Successfully activated service 'org.freedesktop.UDisks2'
Sep  6 21:51:30 ubuntu udisksd[3007]: Acquired the name org.freedesktop.UDisks2 on the system message bus
Sep  6 21:51:54 ubuntu NetworkManager[1691]: <info> Activation (wlan0) starting connection 'Nutella'
Sep  6 21:51:54 ubuntu NetworkManager[1691]: <info> (wlan0): device state change: disconnected -> prepare (reason 'none') [30 40 0]
Sep  6 21:51:54 ubuntu NetworkManager[1691]: <info> NetworkManager state is now CONNECTING
Sep  6 21:51:54 ubuntu NetworkManager[1691]: <info> Activation (wlan0) Stage 1 of 5 (Device Prepare) scheduled...
Sep  6 21:51:54 ubuntu NetworkManager[1691]: <info> Activation (wlan0) Stage 1 of 5 (Device Prepare) started...
Sep  6 21:51:54 ubuntu NetworkManager[1691]: <info> Activation (wlan0) Stage 2 of 5 (Device Configure) scheduled...
Sep  6 21:51:54 ubuntu NetworkManager[1691]: <info> Activation (wlan0) Stage 1 of 5 (Device Prepare) complete.
Sep  6 21:51:54 ubuntu NetworkManager[1691]: <info> Activation (wlan0) Stage 2 of 5 (Device Configure) starting...
Sep  6 21:51:54 ubuntu NetworkManager[1691]: <info> (wlan0): device state change: prepare -> config (reason 'none') [40 50 0]
Sep  6 21:51:54 ubuntu NetworkManager[1691]: <info> Activation (wlan0/wireless): access point 'Nutella' has security, but secrets are required.
Sep  6 21:51:54 ubuntu NetworkManager[1691]: <info> (wlan0): device state change: config -> need-auth (reason 'none') [50 60 0]
Sep  6 21:51:54 ubuntu NetworkManager[1691]: <info> Activation (wlan0) Stage 2 of 5 (Device Configure) complete.
Sep  6 21:51:55 ubuntu dbus[1193]: [system] Activating service name='org.freedesktop.hostname1' (using servicehelper)
Sep  6 21:51:55 ubuntu kernel: [   91.735685] systemd-hostnamed[3143]: Warning: nss-myhostname is not installed. Changing the local hostname might make it unresolveable. Please install nss-myhostname!
Sep  6 21:51:55 ubuntu dbus[1193]: [system] Successfully activated service 'org.freedesktop.hostname1'
Sep  6 21:52:03 ubuntu NetworkManager[1691]: get_secret_flags: assertion 'is_secret_prop (setting, secret_name, error)' failed
Sep  6 21:52:03 ubuntu NetworkManager[1691]: <info> Activation (wlan0) Stage 1 of 5 (Device Prepare) scheduled...
Sep  6 21:52:03 ubuntu NetworkManager[1691]: <info> Activation (wlan0) Stage 1 of 5 (Device Prepare) started...
Sep  6 21:52:03 ubuntu NetworkManager[1691]: <info> (wlan0): device state change: need-auth -> prepare (reason 'none') [60 40 0]
Sep  6 21:52:03 ubuntu NetworkManager[1691]: <info> Activation (wlan0) Stage 2 of 5 (Device Configure) scheduled...
Sep  6 21:52:03 ubuntu NetworkManager[1691]: <info> Activation (wlan0) Stage 1 of 5 (Device Prepare) complete.
Sep  6 21:52:03 ubuntu NetworkManager[1691]: <info> Activation (wlan0) Stage 2 of 5 (Device Configure) starting...
Sep  6 21:52:03 ubuntu NetworkManager[1691]: <info> (wlan0): device state change: prepare -> config (reason 'none') [40 50 0]
Sep  6 21:52:03 ubuntu NetworkManager[1691]: <info> Activation (wlan0/wireless): connection 'Nutella' has security, and secrets exist.  No new secrets needed.
Sep  6 21:52:03 ubuntu NetworkManager[1691]: <info> Config: added 'ssid' value 'Nutella'
Sep  6 21:52:03 ubuntu NetworkManager[1691]: <info> Config: added 'scan_ssid' value '1'
Sep  6 21:52:03 ubuntu NetworkManager[1691]: <info> Config: added 'key_mgmt' value 'WPA-PSK'
Sep  6 21:52:03 ubuntu NetworkManager[1691]: <info> Config: added 'auth_alg' value 'OPEN'
Sep  6 21:52:03 ubuntu NetworkManager[1691]: <info> Config: added 'psk' value '<omitted>'
Sep  6 21:52:03 ubuntu NetworkManager[1691]: <info> Activation (wlan0) Stage 2 of 5 (Device Configure) complete.
Sep  6 21:52:03 ubuntu NetworkManager[1691]: <info> Config: set interface ap_scan to 1
Sep  6 21:52:03 ubuntu NetworkManager[1691]: <info> (wlan0): supplicant interface state: inactive -> scanning
Sep  6 21:52:03 ubuntu wpa_supplicant[2026]: message repeated 3 times: [ wlan0: CTRL-EVENT-SCAN-STARTED ]
Sep  6 21:52:04 ubuntu wpa_supplicant[2026]: wlan0: SME: Trying to authenticate with 14:0c:76:72:18:d7 (SSID='Nutella' freq=2447 MHz)
Sep  6 21:52:04 ubuntu kernel: [  100.188514] wlan0: authenticate with 14:0c:76:72:18:d7
Sep  6 21:52:04 ubuntu NetworkManager[1691]: <info> (wlan0): supplicant interface state: scanning -> authenticating
Sep  6 21:52:04 ubuntu kernel: [  100.205667] wlan0: send auth to 14:0c:76:72:18:d7 (try 1/3)
Sep  6 21:52:04 ubuntu wpa_supplicant[2026]: wlan0: Trying to associate with 14:0c:76:72:18:d7 (SSID='Nutella' freq=2447 MHz)
Sep  6 21:52:04 ubuntu kernel: [  100.207644] wlan0: authenticated
Sep  6 21:52:04 ubuntu kernel: [  100.211348] wlan0: associate with 14:0c:76:72:18:d7 (try 1/3)
Sep  6 21:52:04 ubuntu NetworkManager[1691]: <info> (wlan0): supplicant interface state: authenticating -> associating
Sep  6 21:52:04 ubuntu kernel: [  100.214901] wlan0: RX AssocResp from 14:0c:76:72:18:d7 (capab=0x411 status=0 aid=2)
Sep  6 21:52:04 ubuntu kernel: [  100.215043] wlan0: associated
Sep  6 21:52:04 ubuntu kernel: [  100.215073] IPv6: ADDRCONF(NETDEV_CHANGE): wlan0: link becomes ready
Sep  6 21:52:04 ubuntu wpa_supplicant[2026]: wlan0: Associated with 14:0c:76:72:18:d7
Sep  6 21:52:04 ubuntu wpa_supplicant[2026]: wlan0: WPA: Key negotiation completed with 14:0c:76:72:18:d7 [PTK=CCMP GTK=CCMP]
Sep  6 21:52:04 ubuntu wpa_supplicant[2026]: wlan0: CTRL-EVENT-CONNECTED - Connection to 14:0c:76:72:18:d7 completed [id=0 id_str=]
Sep  6 21:52:04 ubuntu NetworkManager[1691]: <info> (wlan0): supplicant interface state: associating -> completed
Sep  6 21:52:04 ubuntu NetworkManager[1691]: <info> Activation (wlan0/wireless) Stage 2 of 5 (Device Configure) successful.  Connected to wireless network 'Nutella'.
Sep  6 21:52:04 ubuntu NetworkManager[1691]: <info> Activation (wlan0) Stage 3 of 5 (IP Configure Start) scheduled.
Sep  6 21:52:04 ubuntu NetworkManager[1691]: <info> Activation (wlan0) Stage 3 of 5 (IP Configure Start) started...
Sep  6 21:52:04 ubuntu NetworkManager[1691]: <info> (wlan0): device state change: config -> ip-config (reason 'none') [50 70 0]
Sep  6 21:52:04 ubuntu NetworkManager[1691]: <info> Activation (wlan0) Beginning DHCPv4 transaction (timeout in 45 seconds)
Sep  6 21:52:04 ubuntu NetworkManager[1691]: <info> dhclient started with pid 3165
Sep  6 21:52:04 ubuntu NetworkManager[1691]: <info> Activation (wlan0) Beginning IP6 addrconf.
Sep  6 21:52:04 ubuntu NetworkManager[1691]: <info> Activation (wlan0) Stage 3 of 5 (IP Configure Start) complete.
Sep  6 21:52:04 ubuntu dhclient: Internet Systems Consortium DHCP Client 4.2.4
Sep  6 21:52:04 ubuntu dhclient: Copyright 2004-2012 Internet Systems Consortium.
Sep  6 21:52:04 ubuntu dhclient: All rights reserved.
Sep  6 21:52:04 ubuntu dhclient: For info, please visit https://www.isc.org/software/dhcp/
Sep  6 21:52:04 ubuntu dhclient:
Sep  6 21:52:04 ubuntu NetworkManager[1691]: <info> (wlan0): DHCPv4 state changed nbi -> preinit
Sep  6 21:52:04 ubuntu dhclient: Listening on LPF/wlan0/bc:85:56:d9:23:f7
Sep  6 21:52:04 ubuntu dhclient: Sending on   LPF/wlan0/bc:85:56:d9:23:f7
Sep  6 21:52:04 ubuntu dhclient: Sending on   Socket/fallback
Sep  6 21:52:04 ubuntu dhclient: DHCPDISCOVER on wlan0 to 255.255.255.255 port 67 interval 3 (xid=0xc97d6045)
Sep  6 21:52:04 ubuntu dhclient: DHCPREQUEST of 192.168.1.5 on wlan0 to 255.255.255.255 port 67 (xid=0x45607dc9)
Sep  6 21:52:04 ubuntu dhclient: DHCPOFFER of 192.168.1.5 from 192.168.1.254
Sep  6 21:52:06 ubuntu dhclient: DHCPACK of 192.168.1.5 from 192.168.1.254
Sep  6 21:52:06 ubuntu dhclient: bound to 192.168.1.5 -- renewal in 17374 seconds.
Sep  6 21:52:06 ubuntu NetworkManager[1691]: <info> (wlan0): DHCPv4 state changed preinit -> bound
Sep  6 21:52:06 ubuntu NetworkManager[1691]: <info>   address 192.168.1.5
Sep  6 21:52:06 ubuntu NetworkManager[1691]: <info>   prefix 24 (255.255.255.0)
Sep  6 21:52:06 ubuntu NetworkManager[1691]: <info>   gateway 192.168.1.254
Sep  6 21:52:06 ubuntu NetworkManager[1691]: <info>   nameserver '8.8.8.8'
Sep  6 21:52:06 ubuntu NetworkManager[1691]: <info>   nameserver '8.8.4.4'
Sep  6 21:52:06 ubuntu NetworkManager[1691]: <info>   nameserver '208.67.222.222'
Sep  6 21:52:06 ubuntu NetworkManager[1691]: <info>   nameserver '208.67.220.220'
Sep  6 21:52:06 ubuntu NetworkManager[1691]: <info> Activation (wlan0) Stage 5 of 5 (IPv4 Configure Commit) scheduled...
Sep  6 21:52:06 ubuntu NetworkManager[1691]: <info> Activation (wlan0) Stage 5 of 5 (IPv4 Commit) started...
Sep  6 21:52:07 ubuntu NetworkManager[1691]: <info> (wlan0): device state change: ip-config -> secondaries (reason 'none') [70 90 0]
Sep  6 21:52:07 ubuntu NetworkManager[1691]: <info> Activation (wlan0) Stage 5 of 5 (IPv4 Commit) complete.
Sep  6 21:52:07 ubuntu NetworkManager[1691]: <info> (wlan0): device state change: secondaries -> activated (reason 'none') [90 100 0]
Sep  6 21:52:07 ubuntu NetworkManager[1691]: <info> NetworkManager state is now CONNECTED_GLOBAL
Sep  6 21:52:07 ubuntu NetworkManager[1691]: <info> Policy set 'Nutella' (wlan0) as default for IPv4 routing and DNS.
Sep  6 21:52:07 ubuntu NetworkManager[1691]: <info> DNS: starting dnsmasq...
Sep  6 21:52:07 ubuntu NetworkManager[1691]: <warn> dnsmasq not available on the bus, can't update servers.
Sep  6 21:52:07 ubuntu NetworkManager[1691]: <error> [1441576327.426897] [nm-dns-dnsmasq.c:396] update(): dnsmasq owner not found on bus: Could not get owner of name 'org.freedesktop.NetworkManager.dnsmasq': no such name
Sep  6 21:52:07 ubuntu NetworkManager[1691]: <warn> DNS: plugin dnsmasq update failed
Sep  6 21:52:07 ubuntu NetworkManager[1691]: <info> Writing DNS information to /sbin/resolvconf
Sep  6 21:52:07 ubuntu dnsmasq[3175]: started, version 2.68 cache disabled
Sep  6 21:52:07 ubuntu dnsmasq[3175]: compile time options: IPv6 GNU-getopt DBus i18n IDN DHCP DHCPv6 no-Lua TFTP conntrack ipset auth
Sep  6 21:52:07 ubuntu dnsmasq[3175]: DBus support enabled: connected to system bus
Sep  6 21:52:07 ubuntu dnsmasq[3175]: warning: no upstream servers configured
Sep  6 21:52:07 ubuntu NetworkManager[1691]: <info> Activation (wlan0) successful, device activated.
Sep  6 21:52:07 ubuntu dbus[1193]: [system] Activating service name='org.freedesktop.nm_dispatcher' (using servicehelper)
Sep  6 21:52:07 ubuntu NetworkManager[1691]: <warn> dnsmasq appeared on DBus: :1.64
Sep  6 21:52:07 ubuntu NetworkManager[1691]: <info> Writing DNS information to /sbin/resolvconf
Sep  6 21:52:07 ubuntu dnsmasq[3175]: setting upstream servers from DBus
Sep  6 21:52:07 ubuntu dnsmasq[3175]: using nameserver 208.67.220.220#53
Sep  6 21:52:07 ubuntu dnsmasq[3175]: using nameserver 208.67.222.222#53
Sep  6 21:52:07 ubuntu dnsmasq[3175]: using nameserver 8.8.4.4#53
Sep  6 21:52:07 ubuntu dnsmasq[3175]: using nameserver 8.8.8.8#53
Sep  6 21:52:07 ubuntu dbus[1193]: [system] Successfully activated service 'org.freedesktop.nm_dispatcher'
Sep  6 21:52:07 ubuntu NetworkManager[1691]: <info> Activation (wlan0) Stage 5 of 5 (IPv6 Commit) scheduled...
Sep  6 21:52:07 ubuntu NetworkManager[1691]: <info> Activation (wlan0) Stage 5 of 5 (IPv6 Commit) started...
Sep  6 21:52:08 ubuntu NetworkManager[1691]: <info> Policy set 'Nutella' (wlan0) as default for IPv6 routing and DNS.
Sep  6 21:52:08 ubuntu NetworkManager[1691]: <info> Writing DNS information to /sbin/resolvconf
Sep  6 21:52:08 ubuntu dnsmasq[3175]: setting upstream servers from DBus
Sep  6 21:52:08 ubuntu dnsmasq[3175]: using nameserver 208.67.220.220#53
Sep  6 21:52:08 ubuntu dnsmasq[3175]: using nameserver 2a01:e00::1#53
Sep  6 21:52:08 ubuntu dnsmasq[3175]: using nameserver 8.8.8.8#53
Sep  6 21:52:08 ubuntu dnsmasq[3175]: using nameserver 8.8.4.4#53
Sep  6 21:52:08 ubuntu dnsmasq[3175]: using nameserver 208.67.222.222#53
Sep  6 21:52:08 ubuntu NetworkManager[1691]: <info> Activation (wlan0) Stage 5 of 5 (IPv6 Commit) complete.
Sep  6 19:52:14 ubuntu ntpdate[3276]: step time server 2001:67c:1560:8003::c7 offset -7199.923465 sec
Sep  6 19:52:51 ubuntu wpa_supplicant[2026]: wlan0: CTRL-EVENT-SCAN-STARTED
Sep  6 19:59:43 ubuntu wpa_supplicant[2026]: message repeated 5 times: [ wlan0: CTRL-EVENT-SCAN-STARTED ]
Sep  6 19:59:47 ubuntu wpa_supplicant[2026]: wlan0: WPA: Group rekeying completed with 14:0c:76:72:18:d7 [GTK=CCMP]
Sep  6 20:01:06 ubuntu dbus[1193]: [system] Activating service name='org.freedesktop.hostname1' (using servicehelper)
Sep  6 20:01:06 ubuntu kernel: [  642.530020] systemd-hostnamed[4247]: Warning: nss-myhostname is not installed. Changing the local hostname might make it unresolveable. Please install nss-myhostname!
Sep  6 20:01:06 ubuntu dbus[1193]: [system] Successfully activated service 'org.freedesktop.hostname1'
Sep  6 20:01:12 ubuntu kernel: [  648.094957] SGI XFS with ACLs, security attributes, realtime, no debug enabled
Sep  6 20:01:12 ubuntu kernel: [  648.137917] JFS: nTxBlock = 8192, nTxLock = 65536
Sep  6 20:01:12 ubuntu kernel: [  648.194255] ntfs: driver 2.1.31 [Flags: R/O MODULE].
Sep  6 20:01:12 ubuntu kernel: [  648.287772] QNX4 filesystem 0.2.3 registered.
Sep  6 20:01:12 ubuntu kernel: [  648.533832] raid6: using ssse3x2 recovery algorithm
Sep  6 20:01:12 ubuntu kernel: [  648.547329] xor: automatically using best checksumming function:
Sep  6 20:01:12 ubuntu kernel: [  648.671429] Btrfs loaded
Sep  6 20:01:12 ubuntu os-prober: File descriptor 9 (/proc/4233/mounts) leaked on lvs invocation. Parent PID 6087: log-output
Sep  6 20:01:13 ubuntu os-prober:   No volume groups found
Sep  6 20:01:13 ubuntu os-prober: debug: running /usr/lib/os-probes/50mounted-tests on /dev/sda1
Sep  6 20:01:14 ubuntu 50mounted-tests: debug: mounted using GRUB ntfs filesystem driver
Sep  6 20:01:14 ubuntu 50mounted-tests: debug: running subtest /usr/lib/os-probes/mounted/05efi
Sep  6 20:01:14 ubuntu 05efi: debug: /dev/sda1 is ntfs partition: exiting
Sep  6 20:01:14 ubuntu 50mounted-tests: debug: running subtest /usr/lib/os-probes/mounted/10freedos
Sep  6 20:01:14 ubuntu 10freedos: debug: /dev/sda1 is not a FAT partition: exiting
Sep  6 20:01:14 ubuntu 50mounted-tests: debug: running subtest /usr/lib/os-probes/mounted/10qnx
Sep  6 20:01:14 ubuntu 10qnx: debug: /dev/sda1 is not a QNX4 partition: exiting
Sep  6 20:01:14 ubuntu 50mounted-tests: debug: running subtest /usr/lib/os-probes/mounted/20macosx
Sep  6 20:01:14 ubuntu macosx-prober: debug: /dev/sda1 is not an HFS+ partition: exiting
Sep  6 20:01:14 ubuntu 50mounted-tests: debug: running subtest /usr/lib/os-probes/mounted/20microsoft
Sep  6 20:01:14 ubuntu 20microsoft: debug: Skipping legacy bootloaders on UEFI system
Sep  6 20:01:14 ubuntu 50mounted-tests: debug: running subtest /usr/lib/os-probes/mounted/30utility
Sep  6 20:01:14 ubuntu 30utility: debug: /dev/sda1 is not a FAT partition: exiting
Sep  6 20:01:14 ubuntu 50mounted-tests: debug: running subtest /usr/lib/os-probes/mounted/40lsb
Sep  6 20:01:14 ubuntu 50mounted-tests: debug: running subtest /usr/lib/os-probes/mounted/70hurd
Sep  6 20:01:14 ubuntu 50mounted-tests: debug: running subtest /usr/lib/os-probes/mounted/80minix
Sep  6 20:01:14 ubuntu 50mounted-tests: debug: running subtest /usr/lib/os-probes/mounted/83haiku
Sep  6 20:01:14 ubuntu 83haiku: debug: /dev/sda1 is not a BeFS partition: exiting
Sep  6 20:01:14 ubuntu 50mounted-tests: debug: running subtest /usr/lib/os-probes/mounted/90linux-distro
Sep  6 20:01:14 ubuntu 50mounted-tests: debug: running subtest /usr/lib/os-probes/mounted/90solaris
Sep  6 20:01:14 ubuntu 50mounted-tests: debug: running subtest /usr/lib/os-probes/mounted/efi
Sep  6 20:01:14 ubuntu os-prober: debug: running /usr/lib/os-probes/50mounted-tests on /dev/sda2
Sep  6 20:01:15 ubuntu 50mounted-tests: debug: mounted using GRUB fat filesystem driver
Sep  6 20:01:15 ubuntu 50mounted-tests: debug: running subtest /usr/lib/os-probes/mounted/05efi
Sep  6 20:01:15 ubuntu 05efi: debug: /dev/sda2 is a FAT partition (mounted by GRUB)
Sep  6 20:01:15 ubuntu 05efi: debug: /dev/sda2 partition scheme is gpt
Sep  6 20:01:15 ubuntu 05efi: debug: /dev/sda2 partition type is c12a7328-f81f-11d2-ba4b-00a0c93ec93b
Sep  6 20:01:15 ubuntu 05efi: debug: running subtest /usr/lib/os-probes/mounted/efi/10elilo
Sep  6 20:01:16 ubuntu 05efi: debug: running subtest /usr/lib/os-probes/mounted/efi/20microsoft
Sep  6 20:01:16 ubuntu 20microsoft: result: Microsoft/Boot/bootmgfw.efi:Windows Boot Manager:Windows
Sep  6 20:01:16 ubuntu 05efi: debug: bootloader Microsoft/Boot/bootmgfw.efi:Windows Boot Manager:Windows found by subtest /usr/lib/os-probes/mounted/efi/20microsoft
Sep  6 20:01:16 ubuntu 05efi: result: /dev/sda2@/efi/Microsoft/Boot/bootmgfw.efi:Windows Boot Manager:Windows:efi
Sep  6 20:01:16 ubuntu 50mounted-tests: debug: os found by subtest /usr/lib/os-probes/mounted/05efi
Sep  6 20:01:16 ubuntu os-prober: debug: os detected by /usr/lib/os-probes/50mounted-tests
Sep  6 20:01:16 ubuntu os-prober: debug: running /usr/lib/os-probes/50mounted-tests on /dev/sda3
Sep  6 20:01:16 ubuntu 50mounted-tests: debug: /dev/sda3 type not recognised; skipping
Sep  6 20:01:16 ubuntu os-prober: debug: os detected by /usr/lib/os-probes/50mounted-tests
Sep  6 20:01:16 ubuntu os-prober: debug: running /usr/lib/os-probes/50mounted-tests on /dev/sda4
Sep  6 20:01:17 ubuntu 50mounted-tests: debug: mounted using GRUB ntfs filesystem driver
Sep  6 20:01:17 ubuntu 50mounted-tests: debug: running subtest /usr/lib/os-probes/mounted/05efi
Sep  6 20:01:17 ubuntu 05efi: debug: /dev/sda4 is ntfs partition: exiting
Sep  6 20:01:17 ubuntu 50mounted-tests: debug: running subtest /usr/lib/os-probes/mounted/10freedos
Sep  6 20:01:17 ubuntu 10freedos: debug: /dev/sda4 is not a FAT partition: exiting
Sep  6 20:01:17 ubuntu 50mounted-tests: debug: running subtest /usr/lib/os-probes/mounted/10qnx
Sep  6 20:01:17 ubuntu 10qnx: debug: /dev/sda4 is not a QNX4 partition: exiting
Sep  6 20:01:17 ubuntu 50mounted-tests: debug: running subtest /usr/lib/os-probes/mounted/20macosx
Sep  6 20:01:17 ubuntu macosx-prober: debug: /dev/sda4 is not an HFS+ partition: exiting
Sep  6 20:01:17 ubuntu 50mounted-tests: debug: running subtest /usr/lib/os-probes/mounted/20microsoft
Sep  6 20:01:17 ubuntu 20microsoft: debug: Skipping legacy bootloaders on UEFI system
Sep  6 20:01:17 ubuntu 50mounted-tests: debug: running subtest /usr/lib/os-probes/mounted/30utility
Sep  6 20:01:17 ubuntu 30utility: debug: /dev/sda4 is not a FAT partition: exiting
Sep  6 20:01:17 ubuntu 50mounted-tests: debug: running subtest /usr/lib/os-probes/mounted/40lsb
Sep  6 20:01:17 ubuntu 50mounted-tests: debug: running subtest /usr/lib/os-probes/mounted/70hurd
Sep  6 20:01:17 ubuntu 50mounted-tests: debug: running subtest /usr/lib/os-probes/mounted/80minix
Sep  6 20:01:17 ubuntu 50mounted-tests: debug: running subtest /usr/lib/os-probes/mounted/83haiku
Sep  6 20:01:17 ubuntu 83haiku: debug: /dev/sda4 is not a BeFS partition: exiting
Sep  6 20:01:17 ubuntu 50mounted-tests: debug: running subtest /usr/lib/os-probes/mounted/90linux-distro
Sep  6 20:01:17 ubuntu 50mounted-tests: debug: running subtest /usr/lib/os-probes/mounted/90solaris
Sep  6 20:01:17 ubuntu 50mounted-tests: debug: running subtest /usr/lib/os-probes/mounted/efi
Sep  6 20:01:17 ubuntu os-prober: debug: running /usr/lib/os-probes/50mounted-tests on /dev/sda5
Sep  6 20:01:18 ubuntu 50mounted-tests: debug: mounted using GRUB ntfs filesystem driver
Sep  6 20:01:18 ubuntu 50mounted-tests: debug: running subtSET@_progressbar1.pulse()
est /usr/lib/os-probes/mounted/05efi
Sep  6 20:01:18 ubuntu 05efi: debug: /dev/sda5 is ntfs partition: exiting
Sep  6 20:01:18 ubuntu 50mounted-tests: debug: running subtest /usr/lib/os-probes/mounted/10freedos
Sep  6 20:01:18 ubuntu 10freedos: debug: /dev/sda5 is not a FAT partition: exiting
Sep  6 20:01:18 ubuntu 50mounted-tests: debug: running subtest /usr/lib/os-probes/mounted/10qnx
Sep  6 20:01:18 ubuntu 10qnx: debug: /dev/sda5 is not a QNX4 partition: exiting
Sep  6 20:01:18 ubuntu 50mounted-tests: debug: running subtest /usr/lib/os-probes/mounted/20macosx
Sep  6 20:01:18 ubuntu macosx-prober: debug: /dev/sda5 is not an HFS+ partition: exiting
Sep  6 20:01:18 ubuntu 50mounted-tests: debug: running subtest /usr/lib/os-probes/mounted/20microsoft
Sep  6 20:01:18 ubuntu 20microsoft: debug: Skipping legacy bootloaders on UEFI system
Sep  6 20:01:18 ubuntu 50mounted-tests: debug: running subtest /usr/lib/os-probes/mounted/30utility
Sep  6 20:01:18 ubuntu 30utility: debug: /dev/sda5 is not a FAT partition: exiting
Sep  6 20:01:18 ubuntu 50mounted-tests: debug: running subtest /usr/lib/os-probes/mounted/40lsb
Sep  6 20:01:18 ubuntu 50mounted-tests: debug: running subtest /usr/lib/os-probes/mounted/70hurd
Sep  6 20:01:18 ubuntu 50mounted-tests: debug: running subtest /usr/lib/os-probes/mounted/80minix
Sep  6 20:01:18 ubuntu 50mounted-tests: debug: running subtest /usr/lib/os-probes/mounted/83haiku
Sep  6 20:01:18 ubuntu 83haiku: debug: /dev/sda5 is not a BeFS partition: exiting
Sep  6 20:01:18 ubuntu 50mounted-tests: debug: running subtest /usr/lib/os-probes/mounted/90linux-distro
Sep  6 20:01:18 ubuntu 50mounted-tests: debug: running subtest /usr/lib/os-probes/mounted/90solaris
Sep  6 20:01:18 ubuntu 50mounted-tests: debug: running subtest /usr/lib/os-probes/mounted/efi
Sep  6 20:01:18 ubuntu os-prober: debug: running /usr/lib/os-probes/50mounted-tests on /dev/sda6
Sep  6 20:01:19 ubuntu 50mounted-tests: debug: mounted using GRUB ext2 filesystem driver
Sep  6 20:01:19 ubuntu 50mounted-tests: debug: running subtest /usr/lib/os-probes/mounted/05efi
Sep  6 20:01:19 ubuntu 05efi: debug: /dev/sda6 is ext2 partition: exiting
Sep  6 20:01:19 ubuntu 50mounted-tests: debug: running subtest /usr/lib/os-probes/mounted/10freedos
Sep  6 20:01:19 ubuntu 10freedos: debug: /dev/sda6 is not a FAT partition: exiting
Sep  6 20:01:19 ubuntu 50mounted-tests: debug: running subtest /usr/lib/os-probes/mounted/10qnx
Sep  6 20:01:19 ubuntu 10qnx: debug: /dev/sda6 is not a QNX4 partition: exiting
Sep  6 20:01:19 ubuntu 50mounted-tests: debug: running subtest /usr/lib/os-probes/mounted/20macosx
Sep  6 20:01:19 ubuntu macosx-prober: debug: /dev/sda6 is not an HFS+ partition: exiting
Sep  6 20:01:19 ubuntu 50mounted-tests: debug: running subtest /usr/lib/os-probes/mounted/20microsoft
Sep  6 20:01:19 ubuntu 20microsoft: debug: Skipping legacy bootloaders on UEFI system
Sep  6 20:01:19 ubuntu 50mounted-tests: debug: running subtest /usr/lib/os-probes/mounted/30utility
Sep  6 20:01:19 ubuntu 30utility: debug: /dev/sda6 is not a FAT partition: exiting
Sep  6 20:01:19 ubuntu 50mounted-tests: debug: running subtest /usr/lib/os-probes/mounted/40lsb
Sep  6 20:01:19 ubuntu 40lsb: result: /dev/sda6:Ubuntu 14.04.3 LTS (14.04):Ubuntu:linux
Sep  6 20:01:19 ubuntu 50mounted-tests: debug: os found by subtest /usr/lib/os-probes/mounted/40lsb
Sep  6 20:01:19 ubuntu os-prober: debug: os detected by /usr/lib/os-probes/50mounted-tests
Sep  6 20:01:19 ubuntu os-prober: debug: /dev/sda7: is active swap
Sep  6 20:01:19 ubuntu os-prober: debug: running /usr/lib/os-probes/mounted/05efi on mounted /dev/sdb1
Sep  6 20:01:19 ubuntu 05efi: debug: /dev/sdb1 is a FAT32 partition
Sep  6 20:01:19 ubuntu 05efi: debug: /dev/sdb1 partition scheme is dos
Sep  6 20:01:19 ubuntu 05efi: debug: /dev/sdb1 partition type is 0xc
Sep  6 20:01:19 ubuntu 05efi: debug: /dev/sdb1 is not a ESP partition: exiting
Sep  6 20:01:19 ubuntu os-prober: debug: running /usr/lib/os-probes/mounted/10freedos on mounted /dev/sdb1
Sep  6 20:01:19 ubuntu 10freedos: debug: /dev/sdb1 is a FAT32 partition
Sep  6 20:01:19 ubuntu os-prober: debug: running /usr/lib/os-probes/mounted/10qnx on mounted /dev/sdb1
Sep  6 20:01:19 ubuntu 10qnx: debug: /dev/sdb1 is not a QNX4 partition: exiting
Sep  6 20:01:19 ubuntu os-prober: debug: running /usr/lib/os-probes/mounted/20macosx on mounted /dev/sdb1
Sep  6 20:01:19 ubuntu macosx-prober: debug: /dev/sdb1 is not an HFS+ partition: exiting
Sep  6 20:01:19 ubuntu os-prober: debug: running /usr/lib/os-probes/mounted/20microsoft on mounted /dev/sdb1
Sep  6 20:01:19 ubuntu 20microsoft: debug: Skipping legacy bootloaders on UEFI system
Sep  6 20:01:19 ubuntu os-prober: debug: running /usr/lib/os-probes/mounted/30utility on mounted /dev/sdb1
Sep  6 20:01:19 ubuntu 30utility: debug: /dev/sdb1 is a FAT32 partition
Sep  6 20:01:19 ubuntu os-prober: debug: running /usr/lib/os-probes/mounted/40lsb on mounted /dev/sdb1
Sep  6 20:01:19 ubuntu os-prober: debug: running /usr/lib/os-probes/mounted/70hurd on mounted /dev/sdb1
Sep  6 20:01:19 ubuntu os-prober: debug: running /usr/lib/os-probes/mounted/80minix on mounted /dev/sdb1
Sep  6 20:01:19 ubuntu os-prober: debug: running /usr/lib/os-probes/mounted/83haiku on mounted /dev/sdb1
Sep  6 20:01:19 ubuntu 83haiku: debug: /dev/sdb1 is not a BeFS partition: exiting
Sep  6 20:01:19 ubuntu os-prober: debug: running /usr/lib/os-probes/mounted/90linux-distro on mounted /dev/sdb1
Sep  6 20:01:19 ubuntu os-prober: debug: running /usr/lib/os-probes/mounted/90solaris on mounted /dev/sdb1
Sep  6 20:01:21 ubuntu ntfs-3g[6880]: Version 2013.1.13AR.1 external FUSE 29
Sep  6 20:01:21 ubuntu ntfs-3g[6880]: Mounted /dev/sda1 (Read-Write, label "Recovery", NTFS 3.1)
Sep  6 20:01:21 ubuntu ntfs-3g[6880]: Cmdline options: rw
Sep  6 20:01:21 ubuntu ntfs-3g[6880]: Mount options: rw,allow_other,nonempty,relatime,fsname=/dev/sda1,blkdev,blksize=4096
Sep  6 20:01:21 ubuntu ntfs-3g[6880]: Ownership and permissions disabled, configuration type 7
Sep  6 20:01:22 ubuntu ntfs-3g[6897]: Version 2013.1.13AR.1 external FUSE 29
Sep  6 20:01:22 ubuntu ntfs-3g[6897]: Mounted /dev/sda4 (Read-Write, label "Acer", NTFS 3.1)
Sep  6 20:01:22 ubuntu ntfs-3g[6897]: Cmdline options: rw
Sep  6 20:01:22 ubuntu ntfs-3g[6897]: Mount options: rw,allow_other,nonempty,relatime,fsname=/dev/sda4,blkdev,blksize=4096
Sep  6 20:01:22 ubuntu ntfs-3g[6897]: Ownership and permissions disabled, configuration type 7
Sep  6 20:01:22 ubuntu ntfs-3g[6907]: Version 2013.1.13AR.1 external FUSE 29
Sep  6 20:01:22 ubuntu ntfs-3g[6907]: Mounted /dev/sda5 (Read-Write, label "Push Button Reset", NTFS 3.1)
Sep  6 20:01:22 ubuntu ntfs-3g[6907]: Cmdline options: rw
Sep  6 20:01:22 ubuntu ntfs-3g[6907]: Mount options: rw,allow_other,nonempty,relatime,fsname=/dev/sda5,blkdev,blksize=4096
Sep  6 20:01:22 ubuntu ntfs-3g[6907]: Ownership and permissions disabled, configuration type 7
Sep  6 20:01:22 ubuntu kernel: [  658.515694] EXT4-fs (sda6): mounted filesystem with ordered data mode. Opts: (null)
Sep  6 20:01:39 ubuntu ntfs-3g[6897]: Unmounting /dev/sda4 (Acer)
Sep  6 20:01:43 ubuntu wpa_supplicant[2026]: wlan0: CTRL-EVENT-SCAN-STARTED
Sep  6 20:01:56 ubuntu kernel: [  692.744306] FAT-fs (sda2): Corrupted directory (i_pos 131203)
Sep  6 20:02:10 ubuntu kernel: [  706.258795] FAT-fs (sda2): Corrupted directory (i_pos 131203)
Sep  6 20:02:23 ubuntu kernel: [  719.153220] FAT-fs (sda2): Corrupted directory (i_pos 131203)



Error: no grub*.efi generated. Please report this message to boot.repair@gmail.com

Add /mnt/boot-sav/sda6/boot/efi efi entries in /mnt/boot-sav/sda6/etc/grub.d/25_custom
Adding custom /mnt/boot-sav/sda6/boot/efi/EFI/Microsoft/Boot/bootmgfw.efi
Adding custom /mnt/boot-sav/sda6/boot/efi/EFI/Boot/bootx64.efi
sda2/bootx64.efi already added
sda2/bootmgfw.efi already added
Adding custom /mnt/boot-sav/sda6/boot/efi/EFI/OEM/Boot/bootmgfw.efi

---- Grub-install verbose
/usr/sbin/grub-install: 2: /usr/sbin/grub-install: Syntax error: "(" unexpected
--------
/usr/sbin/grub-install --efi-directory=/boot/efi --target=x86_64-efi --uefi-secure-boot : exit code of grub-install :2
---- End of grub-install verbose


chroot /mnt/boot-sav/sda6 efibootmgr -v
BootCurrent: 0001
Timeout: 2 seconds
BootOrder: 2001,0000,2002,2003
Boot0000* Windows Boot Manager	HD(2,c8800,96000,7e77172a-71dd-41ec-a543-f688fe140041)File(EFIMicrosoftBootbootmgfw.efi)WINDOWS.........x...B.C.D.O.B.J.E.C.T.=.{.9.d.e.a.8.6.2.c.-.5.c.d.d.-.4.e.7.0.-.a.c.c.1.-.f.3.2.b.3.4.4.d.4.7.9.5.}....................
Boot0001* USB HDD: General USB Flash Disk	ACPI(a0341d0,0)PCI(12,2)USB(0,0)HD(1,3e,3bb5aa,00000000)RC
Boot0002* WDC WD5000LPVX-22V0TT0          	BIOS(2,500,00)................-.d.......d.A.d........................................
Boot0003* General USB Flash Disk  	BIOS(6,500,00).......................................................................
Boot2001* EFI USB Device	RC
Boot2002* EFI DVD/CDROM	RC
Boot2003* EFI Network	RC

chroot /mnt/boot-sav/sda6 update-grub
Generating grub configuration file ...
Found linux image: /boot/vmlinuz-3.19.0-26-generic
Found initrd image: /boot/initrd.img-3.19.0-26-generic
Found linux image: /boot/vmlinuz-3.19.0-25-generic
Found initrd image: /boot/initrd.img-3.19.0-25-generic
Found Windows Boot Manager on /dev/sda2@/EFI/Microsoft/Boot/bootmgfw.efi
Adding boot menu entry for EFI firmware configuration
Unhide GRUB boot menu in sda6/boot/grub/grub.cfg

An error occurred during the repair.

Locked-ESP detected. You may want to retry after creating a /boot/efi partition (FAT32, 100MB~250MB, start of the disk, boot flag). This can be performed via tools such as gParted. Then select this partition via the [Separate /boot/efi partition:] option of [Boot Repair].

Tu devrais t'inspirer de la solution trouvée dans cette discussion : http://forum.ubuntu-fr.org/viewtopic.php?pid=19588281
C'est à dire, réaliser une liveUSB de Super Grub2 Disk en utilisant ta LiveUSB Ubuntu suivant cette procédure : http://forum.ubuntu-fr.org/viewtopic.ph … #p18288531
Puis après avoir démarré ton Ubuntu installé, faire une réparation du démarrage depuis cette session Ubuntu en passant la commande "sudo grub-install ".

Dernière modification par malbo (Le 07/09/2015, à 12:57)

Hors ligne

#4 Le 07/09/2015, à 13:33

jedineofr

Re : [RESOLU]Dual-Boot Windows 8.1 et Ubuntu 14.04 sur Acer Aspire V5-122P

PROBLEME RESOLU big_smile

Voici ce que j'ai fait, grace à l'aide de malbo. Si ca peut aider du monde ... Je vais aussi faire un lien a partir de la documentation sur ce PC portable

1) Installer Ubuntu en UEFI (donc version international)
2) Créer une clé USB modifié avec le Super Grub et la technique du coucou (http://forum.ubuntu-fr.org/viewtopic.ph … #p18288531)
3) Lancer mon ubuntu installé (et non la version live, erreur que j'ai faite roll)
4) Réinstaller Grub => pas de changement au boot, directement windows
5) Retour sur ubuntu installé par clé modifié et se servir de boot-repair => pas de changement au boot
6) Dans le bios, mettre le disque dur AVANT le Windows Boot Manager big_smile

Remarque : il est possible que des étapes ne soient pas nécessaires, genre le 5 ...

Dernière modification par jedineofr (Le 07/09/2015, à 13:34)


Ubuntu 8.04 LTS -> Ubuntu 8.10 -> Ubuntu 9.04 -> Ubuntu 9.10 -> Ubuntu 10.04 LTS -> Lubuntu 14.04 > Ubuntu 14.04 > Ubuntu 16.04 LTS > Ubuntu 18.04 LTS
Asus ROG G551JW-DM379T (i7 4750HQ, GTX960M, Dd 1 To + SSD 24 Go, Windows 10 + Ubuntu)

Hors ligne

#5 Le 07/09/2015, à 14:46

malbo

Re : [RESOLU]Dual-Boot Windows 8.1 et Ubuntu 14.04 sur Acer Aspire V5-122P

C'est formidable.
Ton cas était donc réellement très semblable à celui de piejearoso (ici) et la procédure de réparation qui avait fonctionné pour lui a bien fonctionné pour toi.
Effectivement, ton étape 5 (Retour sur ubuntu installé par clé modifié et se servir de boot-repair => pas de changement au boot) est inutile. Toutefois, elle montre que Boot-Repair est incapable de réparer ce genre de problème. On le savait déjà depuis ton post #1 puisque tu avais déjà tenté de réparer avec Boot-Repair sans succès. De même que piejearoso avait tenté lui aussi de réparer avec Boot-Repair, mais ça n'avait pas marché non plus.

EDIT : je viens de compléter la ligne pour le modèle Aspire V5-122P(*) dans la Liste des portables « Acer » pour y ajouter ton expérience.
Si tu pouvais démarrer une session de ton Ubuntu installé, faire un Boot-Info depuis cette session et donner l'URL de ce Boot-Info dans ta réponse, ça permettrait d'avoir l'état final après réparation.

(*) EDIT le 2 août 2016 : pour mémoire, le modèle de jedineofr est équipé du processeur AMD A6-1450 (on peut le voir dans le Boot-info du post #3) et la doc de ce composant A6-1450 (paragraphe "Integrated peripherals / components") indique que la partie graphique HD 8250 est intégrée sur la même puce que le processeur.

Dernière modification par malbo (Le 02/08/2016, à 12:51)

Hors ligne

#6 Le 07/09/2015, à 15:07

Bougron

Re : [RESOLU]Dual-Boot Windows 8.1 et Ubuntu 14.04 sur Acer Aspire V5-122P

Bonjour.
C'est bien que tu aies réussi.
Cela serait un plus si tu faisais un boot-info.
Cependant, pour l'action 6 tu devrais détailler un peu   si c'est possible.
Il est possible que la réinstallation du grub aie fait exactement ce que boot-repair demandait de faire!

Hors ligne

#7 Le 18/01/2017, à 22:18

lucsky44

Re : [RESOLU]Dual-Boot Windows 8.1 et Ubuntu 14.04 sur Acer Aspire V5-122P

Bonjour jedineofr,
Je ne sais pas si c'est une bonne idée de ma part de t'écrire sur ce post, mais après tout puisqu'on a la même machine, peut-être pourras-tu me répondre.
Je tourne sous ubuntu depuis qq mois et tout marche impec excepté... la sortie vidéo (prise mini display port).
J'ai essayé de voir si je trouvais qq chose côté pilote, sans succès.
Merci de ton retour d'expérience.

Hors ligne

#8 Le 18/01/2017, à 22:42

jedineofr

Re : [RESOLU]Dual-Boot Windows 8.1 et Ubuntu 14.04 sur Acer Aspire V5-122P

Bonsoir.

Idem que toi, impossible de faire marcher la sortie vidéo alors que tout marche nickel sur Windows.
A noter que la sortie a été testé sur une TV et non pas un vrai écran. Je ne sais pas si ca change quelquechose ...


Ubuntu 8.04 LTS -> Ubuntu 8.10 -> Ubuntu 9.04 -> Ubuntu 9.10 -> Ubuntu 10.04 LTS -> Lubuntu 14.04 > Ubuntu 14.04 > Ubuntu 16.04 LTS > Ubuntu 18.04 LTS
Asus ROG G551JW-DM379T (i7 4750HQ, GTX960M, Dd 1 To + SSD 24 Go, Windows 10 + Ubuntu)

Hors ligne